Download as pdf or txt
Download as pdf or txt
You are on page 1of 348

*Methotrexate cause liver damage and elevate liver enzymes

19 y male with history of headach for 5 days and neck stiffness for two days
With low grade fever for one month CSF analysis Turbid Glucose normal range
WBCs neut: 27 Lymph: 87 or 78 Protein not sure Diagnosis:

A bacterial meningitis
B viral meningioencephlitis
C TB meningitis
D fungal meningitis

*not B because glucose is normal

Adult soldier with bilateral inguinal swelling with - positive cough impulse what is the
most appropriate management
A - Laparoscopic repair
B - herniotomy
C - Open hernia repair and mesh
D – observation

Normal pregnant women at 12 weeks, Indication of sever preeclampsia :


Increase cr
Increase urea
Increase Na level
No platelets in choice

ady 29 week pregnant Came with bleeding and Vesicle Structure came out what is your
Dx ?
- partial mole
- complete mole
- threatened abortion
- missed abortion
RA case on Ibuprofen and Methotrexate for 6 months, came with exacerbations. What
to add?
A- adalimumab
B- Azathioprine
C- Cyclophosphamide
D- Hydroxychloroquine

A female presented with back pain and fever, she was given NSAIDs and advised for bed
rest at home, then she presented with inability to move her lower limbs, O/E there was
tenderness on the midback.
Spinal MRI: Diffuse densities on T6,

What is the most appropriate next step?


A- Reassurance
B- Specimen from T6 vertebra
C- Brucella titer ????
D- Bone marrow aspiration for culture

Pregnant unbooked presented with vaginal bleeding no abdominal pain, fundal height 34.
have no transportation method to hospital. NEXT step:
⁃ US
⁃ Admit to ward
Cervical exam
12 year old boy injured with supracondylar fracture and distal radial pulse absent , which
of the following is the appropriate next step in management?
A-K wire
B-intramedullary nail
C-surgical exploration
D-Closed reduction

*k wire with urgent reduction, but if color is pale and pulse is absent then C-surgical
exploration

70 y old male K/O HTN, DM, hyperlipidemia on madication ( lesinopril, amlodipine,


statin)
came to reguler check and by examination you find congestive heart with bilaterly lung
cripitation and mild LL edema
BB 155/ 85
HR 110
Spo2 94%
What drug to add to his medication?
A) spironolactone
B) frusemide
C) doxazosin
D) atenolol

Child with Sx of tracheomalacia


How to confirm the diagnosis?
A) chest XR
B) bronchoscopy
C) Fluoroscopy

Patient diabetic admitted to hospital on Metformin , which is best during hospitalisation :


Metformin
Sliding scale insulin
Pre and post prandial insulin

8- Stab wound to the neck and injury to Zone 3. There was active bleeding. They did not
mention if stable or not. There was no Vitals. What is the most proper initial
management?
A. Open repair
B. Endovascular repair
C. Artery ligation

A 54 yo female medically free comes for routine checkup ,Her cardiac exam revealed
grade 4 pansystolic murmur heard best at the apex and radiated to the axilla,she is
asymptomatic and the rest of her exam is normal,best next step to confirm the diagnosis?

A. TTE

B. TEE

C. ASO TITER

D. CHEST X ray
Man came with scalp open wound, after 6h assault, what wound management?
A-secondary closure
B-Debridment with 1ry closure
C-Debridment with granulation
D-Leave it for granulation! I think wrote like this

Spina bifida /what is the best diagnostic method?


MRI

A 65-year-old male comes to the clinic with a mild intermittent urinary flow
reduction Rectal examination, urinalysis and prostate specific antigen studies are
normal (see report).Ultrasound prostate: Enlarged median lobe. Which of the
following is the best way to investigate?

A. Annual renal function monitoring


B. Periodic PSA measurement
C. Beta-blocker therapy
D. Cystoscopy
Patient known hypertensive on lisinopril and amlodipine, recently diagnosed as TB
What you will do
1.discountinue lisinopril
2.discontinue amlodipine

*cuz of coughing

in order to eliminate dengue fever from saudi arabia with which you should start
- central
- East
- West
65 y male heavy smoker medicaly and surgycally free
What screen for him
A) DM
B) HTN
C)AAA

* pt came form southern east Asia with his of 2 weeks SOB cough
O/E Febrile + Rt upper lobe crepitation
Xray ..upper Rt lobe opacity and cavitation..
Best action :
- ceftrixone
- sputum for AAFB
- INH ,rifampcin , ethambutol , Pyrizinamide

* test for pulmonary TB, the AFB culture is done on phlegm (sputum) coughed up

3 y old child diagnosed with malaria


What is type of malaria?
A) P falciparum,
B)P vivax,
C)P ovale
D) P malariae

*if the Q about most common then it’s B-Vivax

★SLE
● LABs
➔ Most sensitive lab > ANA
➔ Most specific lab > anti-dsDNA,
anti-smith
● Management.
⁦ ⁩ARTHRITIS :
➔ Arthritis & malar rash: hydroxychloroquine
➔ Extremely severe flare up of polyarthritis: IV methylprednisolone
⁦ ⁩RENAL Involvement
➔ SLE with mild nephritis: corticosteroids
➔ SLE with severe nephritis (rapidly
progressive, diffuse proliferative, or severe proteinuria and active urine sediment) :
corticosteroid + cyclophosphamide
⁦ ⁩CNS involvement:
➔ SLE with central nervous system manifestations (seizure, organic brain syndrome or
coma):
IV cyclophosphamide & IV methylprednisolone
➔ Methotrexate or azathioprine : steroid-sparing drugs
⁦ ⁩Drugs induced lupus: (which diagnosed by Anti-Histone AB)
● Hydrazine
● Isoniazid
● Chlorpromazine ● Procainamide

Meningococcal prophylaxis?
Antibiotics according to the causative bacteriab and age ; refambicin, ciprofluxacin or
ceftrixon
38 y male complain of constipation for 2 days painful perianal with positive crack on 6 &
12 clock
Anal fissure
Perianal abscess

COPD patient came to clinic with bilateral lower limb edema and pulmonary
hypertension. O2 sat 86% ,PO2 8.6, PCO2 7.5, pH above normal range Which of the
following the appropriate management to give for the patient now

A) Start oral furosemide


B) Start oxygen therapy
C) Prednisolone therapy

Pt with K/C CKD , ... at the end asking what is Most common cause of death in CKD?
A- CVD
B- hyperkalemia
C- Uremia
D- coagulation disorders

young male came to well baby clinic, upon examination his right testis was
palpable in the inguinal canal and small in size and easily moved to
scrotum, the left is normal, what is the cause?
-ectopic testis
-undescended testis
-testicular torsion
-Retractile testis

7 years child , systolic murmur heard in lower sternal area with no thrill, ( more
noticeable when supine)

ASD
VSD
AORTIC STENOSIS
STILL MURMUR

Pt's friend ask you about diagnosis of Pt's disease. You refused . What is ethical concept?
- privacy
- confidentiality
- dignity
- another option

pt of mesentric ishemia ..best investigation?


- CT
- Explatory Laprotomy
- US
-Laproscopy
* Male pt with features of SLE with active arthritis..ask about best management
- methotrexate
- HCQ
- Methotrexate, HCQ & steroid
-Another option

* pt came form southern east Asia with his of 2 weeks SOB cough
O/E Febrile + Rt upper lobe crepitation
Xray ..upper Rt lobe opacity and cavitation..
Best action :
- ceftrixone
- sputum for AAFB
- INH ,rifampcin , ethambutol , Pyrizinamide

Patient has SLE with neurological manifestation patient already on corticosteroids, what
to add now ?
A- cyclophosphamide
B- hydroxychloroquine
C-phenytoin
D- diazepam

Pt has rest tremor but when he move to pick something the tremor gone, but can’t do
finger nose test with his right hand where is the lesion ?

Right cerebellum cerebellum ‫بإ ختصار بما ن ا'ريض عنده تريمور يعني ا'شكلة في‬
Left cerebellum ‫ن حصل الكروس‬A ‫تجاه‬A‫صابة في نفس ا‬A‫وفي السربيلم تكون ا‬
Right hemisphere
Female patient presents With achalasia. what is the most appropriate management?

Non surgical Pnemoatic dialation


And botilinium toxin imj
Surgical treatment
Hellar myotomy
*CCB and Nitrate

Which is most indicative of malignant nodule rather than tuberculosis nodule:


-Hypercalcemia
-Nodule size >5cm

Child with foreign body aspiration?


Rigid endoscopy

128-Patient with HTN not responding for three anti hypertensive drugs on Ultrasound the
one kidney
is bigger what is diagnosis
A Adrenal hyperplasia
B Renal artery stenosis
C Pheochromocytoma

Drugs = Antidote

1⃣_ Acetaminophen (paracetamol)--➡️' N-


Acetyl Cystine (NAC)
2️⃣Beta Blockers➡️ Glucagon
3️⃣Opioids➡️ Naloxone
3️⃣Anticholinergic➡️ Physostigmine
4⃣ Benzodiazipines--➡️Flumazenil
5⃣ Warfarin-' ➡️FFP (early) or Vit K (late)
6⃣ Heparin ➡️Protamine sulfate
7️⃣ Insulin--➡️ Glucagon
8⃣ Iron---➡️ Desferoxamine
9⃣Digoxin ➡️Digoxine immune Fab
CCB ➡️Calcium chloride
1⃣1⃣Mg+2 Sulfate ➡️Calcium gluconate
1⃣2️⃣ASPRIN➡️sodium bicarbonate

Patient has complain about change of mood 3 days befor menstruation That affects her
work and life
What is the case :
A- PMDD. ,
B- B- PMS

* (pre menstrual syndrome)

RTA patient was transfused with 4 L of blood , which of the following would be the
complication?
A- Citrate toxicity
B- Hypocalcemia

Q: parents came with their 4 year old child who has not moved their right arm for 2 days
after falling on a stretched out arm. They claim on the first day they saw no bruise or
bleeding but thought its normal child not moving the arm after falling. On the second day
they got worried. On examination the child looked well and healthy with no other injuries
or signs of trauma. On x-ray both ulnar and radial fractures. In the patient file you saw
regular visits with no concerns about the family?
A- Contact child services
B- contact police
C- treat the injury only (this option was worded differently but it suggested no sign of
child abuse so focus on injury)
D- i dont remember
35-year-old pregnant women, classified as low risk pregnancy prior, came today with
vaginal bleeding and abdominal pain of which she was diagnosed as placenta abruption.
What is the next step?
A. continue same management.
B. classified her as high-risk patients and do U/S

Case after motor accident on examination of heart you hear muffled heart sound (there is
more but i forgot)
Labs:
80/40
X-ray: normal
Diagnosis:
A - cardiac tamponade
B - hypovolemic shock

* A triad of cardiac tamponade: hypotension. Muffled heart sounds, neck vein distended
(increased JVP)

Case of female child who lives in poor family with distended abdomen and pitting edema
there are lab results but I don’t remember them:
A - marasmus
B - Kwashiorkor

27 years old women come to the ER complain of mild vaginal bleeding, LMP was before
9 w, Ex revelead soft uterus felt above the symphysis pubic, what is the most appropriate
next step?
- pelvic US A this is threatend abortion to exclude missed abortion
- pelvic MRI
- CT abdomen
- CT chest abdomen pelvic

Child had type 1 diabetes and consulting the family regarding celiac screening , which of
the following is true ?
— A- Screen at diagnosis then every 5 years
— B- Screen at diagnosis then every 2 years
— C- Screen annually
— D- Screen at diagnosis then annually for 5 years
* #screening for celiac at diagnosis then every 2 years

#Screening for retinopathy and nephropathy at diagnosis then every 5 years


Lung node meassure 8mm
asymptomatic what to do
A) Follow up
B) CT scan
C) biopsy
D)review previous x-ray

case of PCOS asking about treatment.


Letrizole
tamoxifin
Progesterone
4th one was wrong
COCs ‫اذا مفكرة ما تحمل اول خيار حيكون‬

indications for therapeutic thoracocentesis

A. Loculated
B. Ph > 7.2
C. High glucose
Polyhydramnios cause ?
A/ Anencephaly
B/ Post term pregnancy
C/ Maternal ingested NSAIDs
D/ Posterior urethral valve

24 years old male, medically and surgically free , presented with manifestation of
intestinal obstruction, suspected to be small bowl obstruction
What is the most common cause
hernia
adhesion
meckles diverticulum
malignancy

*cuz surgically free

22 years female known case of severe depression ،suicidal attempts, she is controlled on
paroxetine Now, she is pregnant. What to do:
A. Stop paroxetine because of fetal malformation
B. Continue paroxetine and control her depression
-Not sure of other options
C. Switch to others drug i think
D. Stop paroxiten b/c of prematurity

25 years old primigravida 20 weeks GA history of mitral stenosis due to rheumatic fever
What physiological change makes her at high risk for heart failure?
- increase RBC mass
- increase stroke volume
- increase minute ventilation
- increase Renal plasma volume
Uterine Fibroid

Pregnant at 3rd trimester with asymptomatic bacteriuria:


-amoxicillin ‫جابة خطأ والسؤال ناقص لذلك راجعوا ا'انجمنت كامل‬A‫ا‬
-nitrofurantoin

Nitrofurantoin is contraindicated in 3rd trimester

Rheumatic fever is caused by a bacterium called group A Streptococcus

36 old male at ER C/O Right abdominal Pain , O/E : fever, anorexia , weight
loss , tenderness in RQ and Lower intercostal margines also patient is toxic Temp.
37.9 ( I think but it was elevated ) wbc high, bilirubin high US : cystic lesion
without septates CT : homogenous (not sure) and “THICK WALL with Peripheral
enhancement what’s most appropriate Mx :

A. Early laparoscopic cholecystectomy


B. Emergent stent chole
C. Cholecystectomy after 3 months
5 years old man Hx of laproscopic appendectomy 10 years ago, present to ER with
sudden abdominal pain and vomiting , mild generalized tender, vitally stable, abdomen x
ray shows multiple air fluid level and dilated loop in certian point, what is next step?

- diagnostic laproscopic
- laparotomy
- CT abdomen
- barium swallow

Patient presented with dysphagia to liquids more than solid


Which is the most appropriate initial investigation:
1/endoscopy
‫ اما‬B ‫جابة‬A‫بعد البحث في لكثر من مصدر كانت ا‬
2/barium swallow
3/US ‫ صحيحها كلها‬B ‫ و‬A ‫امبوس واب تو ديت قالوا ان‬
4/biopsy

Mobitz type II >> Pacemaker

1st degree:
no treatment
🟠2nd degree (M1):
if Symptomatic (Atropine or temporary pacing) Mobitz type II
🟠2nd degree (M2):
Atropine and pacing for unstable patients
Permanent pacemaker Stable. Unstable
🟠3rd degree:
Same as 2nd degree Pacemaker Atropine and pacing
Drugs in decreasing motality in HF?
ACE-I & BB
Patient with rectal bleeding 5 ; 7 o clock sclerotherapy done
What type of hemorrhoid treated:
1/internal
2/external
3/prolapse
4/thrombosed

Pt bring her 7 yrs old child worried about short stature


There is delayed bone growth
What is management:
1/reassure he will gain adult stature with puberty
2/ do basic investigations cbc.TFT.bone age.

CTG sinusoidal pattern


Management??
Cord prolapse. Cs

Serum K 6.1
Management? Ca gluconate
And what ECG change?
Peak T wave

Pt admited to ICU with inferior MI started ttt then brady cardia i think
What is the diagnosis:
1/first degree heart block
2/second degree heart block
Patient post cholecystectomy presented with abdominal pain .fever
Rt pleural effusion
Collection at gallblader fossa ..what is the apropriate management:
1/US aspiration
2/ ERCP
Patient with Rt upper quadrant pain...jaundice fever...O/E RUQ tenderness
High TWBCs
Amylase normal
Mildly dilated CBD 1.3
What is the apropriate diagnosis:
1/pancreatitis
2/cholangitis
3/appendicitis
4/acute cholecystitis

Patient with Rt upper quadrant pain...fever...jaundice..hx of stone


O/E generalised abdominal tenderness and sluggish bowel sounds
High TWBCs
High amylase
US showed mildly dilated CBD 1.1
What is the apropriate diagnosis:
1/pancreatitis
2/cholangitis
3/appendicitis

Appropriate management of papillary thyroid Ca ?


total thyroidectomy

Scenario of patient with nephrotic syndrome then developed generalized abdominal pain
and tenderness
Which of the following describe complication occurred:
1/peritonitis
2/pancreatitis
RTA patient came with deformed swell leg...there is pain and paresthesia and absent
pulse
Most appropriate management:
Fasciotomy
14 years old female with infrequent vomiting during her period_came this time with
vomiting associated with smal amount of blood then symptoms releived.no
vomiting..soft non tender abdomin
Most apropriate next step:
1/Admit for observation
2/Discharge home and tell to come if symptoms recur
3/Prepare for urgent endoscopy
...

Child point to red color..ride tricycle. Dress himself what is his age??
Tricycle at 3 years. Dress himself at 4 .. so 4 y.o. is correct

Rheumatoid arthritis patient on NSAID presented with sever epigastric pain and
tenderness This patient is on NSAID which conceded as risk
Most appropriate investigation: factor for gastritis and gastric ulcer.
1/Erect CXR
CXR will show air under the diaphragm that
2/US abdomen indicates a perforated viscus

Pic of Peripheral blood film show microcytic hypochromic cells with scenario ...HB low/
RBCs low
What suspect to be low:
1/MCV
2/Reticulocytes
3/platelets

Patient heavy smoker presented with SOB and wheeze


What immediate inhaler to give:
1/ipratropium bromide
2/ Ventolin
Senario of COPD

*Ventolin= Salbutamol
Mother 2 months post-delivery. asymptomatic. culture screening of urine more
than100000 E.coli sensitive to ciprofloxacin nitrofurantoin anf trimethoprim
sulphamethoxazole
Ask about treatment:
1/ciprofloxacin
2/ nitrofurantoin
3/trimethoprim sulph...
4/ no need

Younge patient complain of suprapubic pain and dysuria


No fever
Urine analysis given=
Urine colour yellow
Turbid
Leukocytes 10
Other normal
What is the next step:
1/ empirical antibiotics
2/ do culture and wait for result
3/ no need for treatment
...

female 15 days post c/s presented with suprapubic pain ,fever and vaginal discharge O/E
wound clean
Diagnosis:
1/ endometritis
....

Younge female presented with suprapubic pain and heavy vaginal discharge
O/E tender fornixes what diagnosis: I think this is pad recall this is tepical salpingitis
1/ cervicitis
2/ vaginitis
3/ endometritis

Scenario post deliver pph then ask about management of PPH:


1/ misoprostol
2/ propofol
3/ dexamethasone
No oxytocin in answers

Plasmodium falciparum
Asthmatic patient on ICS and use SABA inhaler at least once daily
What to add to his medications:
1/ leukotriene antagonist
2/ salmeterol
....

Child with sore throat..toxic with drooling..sitting upright with mouth open
What is the causative organism:
1/ hemophilus influenza type B
2/ parainfluenza
3/ Rhinovirus
4/adenovirus

Child with staccato cough_inspiration between cough and post tusive vomitiyn_ there is
history of conjectivitis and eosinophilia in investigations what is the diagnosis:
1/pertussis
2/ chlamydia pneumonia
3/ mycoplasma pneumonia
Pt on oral steroids presented with white layer in his toungue and buccal mucosa what is
the treatment:
1/ nystatin
2/ketoconazole
3/Amphotericine B

Pt known case of chronic hepatitis B and a symptomatic for follow up


US showed liver with mild coarse texure and nodule 3×3 what most apropriate
investigation:
1/ alfa feto protien
2/ biopsy
3/ US abdomin
4/ triphasic abdominal CT

4 yrs child contact with TB patient..PPD skin test 10mm:


1/Positive
2/negative
3/repeat test
4/no need for test

Pt 64 Male , HT , Diabetes, HF , history of stroke the CHADS2 is ?


A3
B4
C5

62 y with bph , BMI 41 , what is the risk of BPH in this case:


A age
B obesity

Pt did upper endoscopy and there is squamous carcinoma Which of the following is a risk
factor of esophageal cancer?
A GERD
B Barret’s esophagus
C Esophageal stricture
D.smoking

*GERD if adenocarcinoma

Long case: Most significant risk factor for MI?


-smoking
-age
-hypertension

* The most common risk factor among patients with initial MI was hypertension
(52.3%), followed by smoking (31.3%), dyslipidemia (28.0%), family history of CHD
(28.0%), and the least common traditional risk factor, diabetes (22.4%)

Mother brings her infant to family medicine clinic for regular check, the doctor advice
her to start iron supplement after 1 month from this visit, from this scenario her child in
which age now (in months)?
A.1
B.2
C.3
D.6
Baby Preterm diliver (didn't specify CS or SVD). 4 hours later complaining from SOB ,
Tachypnea, Tachycardia with grunting

1- Hyaline membrane disease


2- pulmonary "something"
3- Meconium Aspiration syndrome
4- Transient Tachypnea of the New born

* 4... For preterm


1... More in term delivery

Pregnant with recurrent UTI what to do?


A. X ray
B. US
C. ureteroscopy
D.cystoscopy and RFT’s

What is the most common cause of bleeding in postmenopausal women ?


- endometrial atrophy
-endometrial cancer

32 Female pt has Bloody nipple discharge what initial breast investigation?


‫ف على هذا السؤال لكن بناءا ً على اب تو ديت‬e‫يوجد اخت‬
Mammo
‫ماموقرام هي الصح وبناءا ً على امبوس التراساوند هي الصح‬

Pregnant case of epilepsy for 2 years and poor control despite medication , what will you
do?
- review the medication
Most causative organism in case of valvular replaced patient, vegetation seen?
strep viridians , strep mitis

What of the following will cause oligohydramnios?


A-Placental insufficiency
B- DM
C- Chriongioma
D-Dudenal atresia

*Polyhydramnios caused by DM & Duodenal atresia

Old male pt did total thyroidectomy then he developed swelling in the neck with
inspiratory stridor and sob How u will manage

1- thoracotomy
2- surgical exploration
3- drainage
4- Surgical evacuation

Child has fatigue and splenomegaly , Hb low,RBC low ,MCV low ,Retic normal ,Iron
normal , What to replace in this case ?
A Iron
B. B12
C. Folate
D. Erythrocytes
Pt. dignosied with lung cancer what will do before operation?
Chest x Ray
Ecg
Cbc
Lung function test
Echo

*all

55 years old male, no history of dm or any other disease


Bp: 159/75 What will give him ?
1/ lisinopril
2/ amlodipine
3/ hydrochlorothiazide
4 / lisinopril + amlodipine

Tracheomalacia highest diagnostic value?


Laryngoscopy.. Bronchoscopy and the flexible bronchoscopy

newborn, abdominal distention and palpable bilateral kidney.


Distended proximal urethra, ask about Dx?
bilateral hydronephrosis, cystic diseases, or due to malignant or benign infiltrative diseases.
Rarely, it can occur in bilateral renal vein thrombosis.

Last mensural cycle was (?/?/2019) , what’s the estimated date of confinement by
Naegele rule?
Add 9 month + 7 days
‫هذا السؤال بالذات بحث عنه طويل مالقيت ا'علومة اللي ابغاها بس بإختصار‬
‫ثة شهور والدراسات الحالية صارت‬e‫زم نوقفه قبل الحمل بث‬A ‫زمان كانوا يقولوا‬
‫تقول نوقفه قبلها بشهر ويبدولي بعد البحث والتحري ان الهيئة ما غيرت ا'علومة‬
Vesicourethral reflux
D ‫ هذه الخيارات هي‬u‫جابة الصحيحة من ب‬A‫القديمة لذلك ا‬

Patient takes retinoic acid asking when she can get pregnant
A- Tell her to stop retinoic acid now and try to get pregnant
B- She can get pregnant after she finish the course
C- get pregnant during the course no problem
D- She can get pregnant after 3 months of stopping retinoic acid

Female with bloatingand GI symptoms. Started gluten free diet by herself and now
she is better and visiting the GP. What to do?
A* Dietician referral
B* GI referral
C* No need to continue on gluten free diet
D* Stop gluten free diet and then do celiac serology

Pediatric pt have all UTI Sx and ask what’s the most diagnostic value?
A. Two mixed organisms from clean catch
B. Single organism from any colony from mid stream urine sample
C- Single organism (3x10^5) from suprapubic
Pregnant woman found to be non-immune to rubella, when to give vaccine?
3rd trimester
After delivery

1- child came with nasal congestion on examination and there is pale nasal polyp and
darkperiorbital swelling
A- allergic rhinitis
B- rhinitis medicamentosa
C- Acute bronchitis
D- foreign body

female 72 year
C/o knee pain,morning stiffness, wake her at night
Diagnosis ‫اتوقع فيه خطأ في السؤال لذلك راجعوا الفرق‬
Osteoarthritis ‫وستي والريماتويد‬A‫ ا‬u‫ب‬
Osteoporosis
Rheumatoid

6yrs with inability to bear weight on exam there was bilateral knee pain. Labs
showed positive ANA. Juvenile onset arthritis is suspected what type could he
have?
NB:Oligoarthritis
JIA

here was a case accidentally finding in health campaign a pediatric patient with
5cm thyroid nodule, TSH is normal, no compressive symptoms what’s the
appropriate next step:
A- radioactive iodine
B- FNA
C- nothing

Dysmenorrhea, how to best diagnose endometriosis:


A. laparoscopically
B. USC.
MRI

Patient came with vaginal discharge she have done CS with episiotomy 10 days
ago , the obstetrician diagnosed her with UTI and described Abx , but she did
not improve then she went to another obstetrician and he found infected vaginal
swab , What is the medical error done by the first obstetrician?
A. let the midwife assist him and depend on her
B. doctor failed to follow the surgical safety protocol in the OR
C. no communication between the second and first obstetrician
D. genuine differente assessment or diagnosis of patient case

Woman with hx of miscarriage, now in 7 weeks with vaginal spotting, no hx of


passing tissue , on examination os is closed and no bleeding
A. Threatened abortion
B. Normal delivery
C. Incomplete abortion
D. Inevitable abortion

A baby with noisy breathing and wheezing that improves when prone and
increases when supine. What is the diagnosis?
A- Laryngomalacia
B- Tracheomalacia
Pediatric patient diagnosed as croup in the ER, he was given the proper
management, but after 1 hour he did not improve. What is the appropriate
management?
A-exposure to cool humidified air
B-prednisilone (with dose)
C-dexamethasone (with dose)
D-racemic epinephrine

9 y/o pt his mom complaining about he is wetting his bed since one month. He
used to be dry since 7years The urine is foul smelling His lab showing high wbs
and turbid in colour and showed nitrate Diagnosis ?
A- UTI
B- B- normal for his age
C- C- nocturnal enuresis

Chloasma
‫ حنستمر‬u‫في سؤال بتجميعات الشهور ا'اضية بس هناك كانت تبغى تحمل وبالحالت‬
‫نه يعتبر سيف على الحوامل وا'رضعات واللي يبغوا يحملوا‬A ‫ج‬e‫على الع‬

pregnant patient on 20 week known case of SLE controlled on Plaquenil what to do?

1-Refer to rheumatologist to stop medication


2-Stop medication
3-Continue medication

Hydroxychloroquine is Plaquenil

They suspect hypovolemic AKI what you’ll see ?


A-BUN/Cr >20
B-FENA >2%

14- RT leg pale & absence of peripheral pulse + irregular irregular radial pulse, What is
the Source of thrombus?
A/ RT atrium
B/ LT ventricle
C/ Thoracic Aorta
D/ Abdominal Aorta

35y.o man presented with fever, rigors & weight loss for 10 days. He underwent Mitral
valve replacement by prosthetic valve 1 month ago, Echo showed a small vegetation.
Vitals: Blood pressure 90/70 mmHg, HR 100/min, Temp 38.7°C. Caustive organisim is ?

A. Coxiella burnetli
B. Staphylococcus aureus
C. Streptococcus viridans
D. Staphylococcus epidermidis

* After valve replacement divide the commonest organism depend on how many days
passed
Less than 60 days = epidermis
More than 60 days become like native valve = aureus
Pt post some cholangio intervention few hours later he devolps chills and rigor only no
fever mentioned
TWBs was 9 normal till 10
RR 18
HR 90
Bp 115/88
Temp :38 degree exactly
What is the most likely diagnosis :
A) sepsis
B) SIRS
C) bactermia
D) septic shock
IAm sure of the figures

15 month boy presented with refusal of feeding vomiting febrile was suspected to have
meningitis so CSF was done showed normal glucose high protein high cells
Mainly neutrophils
Culture showed gram positive diplococci
Which of the following is the most appropriate management :
A) ampicillin
B) ampicillin and gentamycin
C) Vancomycin
D) ceftriaxone and Vancomycin

Pediatric with seizure and other signs, labs showed metabolic


acidosis, urine analysis: aminoaciduria and glycosuria, and
cataract what’s the diagnosis?
A. Galactosemia
B. Homocystinuria
C. Methylmalonic Academia
D. Maple syrup urine syndrome

What is the structure having abnormality in cardiac tamponad :


A) pericardum
2) mayocardium
3) coronary artery
4) endocardium

Q20 - Patient underwent lap chole for multiple gallstones 7 days ago, presents with vague
abdominal pain. On US there is fluid around the gallbladder (something like that) and the
CBD is 9 mm.
What is the most likely diagnosis or cause?
A - CBD injury
B - Retained stone in CBD
C - Sub-hepatic collection

TB : Caseating granuloma / Treatment: AKT

Sarcoidosis : Non Caseating Granuloma/ Treatment: Steroids

* A K T 4 Kit Tablet belongs to the group of medicines called anti-tuberculosis drugs


primarily used to treat and prevent tuberculosis. A K T 4 Kit Tablet is a combination of four
anti-tuberculosis drugs, namely: Rifampicin, isoniazid, pyrazinamide, and ethambutol.

Case about baby ingest aspirin large dose!! Toxicity Q what abnormality in acid base
balance?
Fast breathing --- Respiratory alkalosis. ---- then metabolic acidosis .
Baby underwent forceps delivery and the physician noticed crushing the left
sternomastoid foramen what the baby expected to have ?
Left side cant close his eye
Lost of sensation in the anterior 2/3 of the tongue

Parents discover tonic clonic abnormal seizure of their child during sleep
What is the Dd?
Nocturnal sizure

47-year-old female presents with iron deficiency anemia. She has 3rd degree
hemorrhoids. Which of the following is the most appropriate thing to do?
A - Colonoscopy
B - Hemorrhoidectomy
C - CT scan

case of female auditory hallucination


And delusional
For 1 month after that she recover without treatment?

Brief psychotic episode. <1month


Schizophreniform >1monfh but less than 6month
Schizophrenia >6months

Pt known HTN and DM found out she's pregnant what is your management?
A) folic acid supplements.
B) ANC labs. ‫ بس باختصار كل ادوية‬UTD ‫م مناسب في‬e‫مالقيت ك‬
C) US. ‫السكر والضغط ما تناسب للحوامل‬
D) review her medications

Pt vith dysmenorrhea what is the important to ask?


A) Family hx
(B)MehstrualI HE
(C)Medical hx
(D)Surgical hx

patient presenting with severe bleeding in 9th week of pregnancy, Os is open, doctor saw
some tissue on the cervix. What is your management?
A-Expectant management
B-D&C
C-Oxytocin

Old patient hx of HF with dialated cardiomyopathy and A.fib . what is the most
appropriate management to control his heart rate?
A.propranolol
B. diltiazem/ verapamil
C. Digoxin

Depressed female. What is the most important risk factor of suicidal thoughts?
A- Age
B- Gender
C- Social isolation
D- Previous attempt.

) Highest risk factors for suicide?


A- Absence of medical illness
C- Social isolation
D- C- Age
E- Female
Prolonged deceleration

A postmenopausal female complaining of vaginal itching and irritation. There is vaginal


discharge
which she described as watery and odorless. On examination, there were multiple
scratches, and
it had a scaly appearance, and it bleeds with touch. What is your diagnosis? A)
Trichomoniasis
B) Atrophic vaginitis
C) Candida

Pt in ICU received 15 units of blood, now blood coming out from NGT, incision, and
cannula site
1. Transfusion reaction
2. Thrombocytopenia
3. Hypocalcemia
4. Von Willebrand

16 yo male CO 3 years altered bowel habit he has 3-4x a day diarrhea that is foul
smelling containing undigested food, no blood or mucus. He has post prandial abdominal
distention (no mention of the type of the food). No secondary sexual characteristics
(High LFT)
Most appropriate investigation?
A. Stool fat test
B. Barium follow though
C. Lactose Hydrogen breath test
D. IgA endomysia antibody

Pt of stab wound 2 cm penetrating injury with omentum passing out


Pt is stable
Ct report negative findings
Next step management
1 observation
2 close the wound
3 leave the wound open
4 laparotomy

d pt DM- HTn cardiac dis, long senario. And all treatment given and stable on
thrombolytic, What is the most common cause of death during his hospital stay:
A. bleeding
B. stroke
C. PE
D.MI

A 35 year old lady presents with a left nipple bloody discharge, by imaging it
was suggestive of Intraductal papilloma. What to do next?

A- Central Duct excision


B- Observation
C- Mastectomy
D- image guided biopsy

: What is the rational of antihypertensive medicatio in preeclampsia?


A.decrease UGR
B. Decrease mothers’ mortality B decrease maternal complication as stroke < as other recall
c- Decrease fetus mortality

• 56 YO female presented with a 2️ months history of jaundice associated with lethargy &
polyarthralgia. On examination, she is jaundiced & has clubbing. There are several spider
naevi on the front & back of the trunk. Her abdomen is soft & there is a smooth
hepatomegaly. Prior to her onset of symptoms, the patient has been fit & well. Her LFTS
reveal a bilirubin of 46 IU/L, AST 200, ALT 175, ALP 104. Viral serology is -ve & anti-
soluble liver antigen is detected. You decide to start this patient on management. What is
the most appropriate management?
A. Liver transplantation
B. Methotrexate
C. Prednisolone
D. CyclospoAntivir

*(autoimmune hepatitis)

Patient after 2 days post mi complains of chest pain radiate to back with st segment
elevation in leads II, III, and AVF present now with a new holosystolic murmur and
bilateral basal crackles. Diagnosis??
A- Right ventricular infarction
B- papillary muscle rupture
C-VSD
D-cardiac tamponade

Postnatal which should b treated first?


A-autism
B-dip in sacral area
C-tof
D-undescended testis

GA 39 weeks having prom of 24 hours..ctg normal and no contraction not in labor what
to do??
A-c/s
B-iol
C-observation
D-augmentation of labor

Pt with hepatitis C and cirrhosis came to the ER drowsy with abdomen distention, on
exam abdomen is tense with lower bilateral limb edema, what is the management?

1- perform paracentesis
2- start diuretic and spironolactone
3- TIPS

8 Y/O very obese lady,BMI 41 had plevic organ prolapse many time and then had
surgery, what do you expect she’s having now :
A- Entrocele
B- Rectcele
C- Uterine prolapse
D-Cytocele
E-Vesicocele

100-A pregnant lady complains of vaginal bleeding, shes a confirmed case of low lying
placenta, no
active bleeding now and no pain. Next step in management?

A- Ultrasound
B- Biophysical profile
C- CTG
D- Emergency delivery

Patient with hx of pancreatitis months ago and now referred to surgery clinic for mild
abdominal pain and , what initial inves?

A-U/S
B- CT
C- endoscope

Milestone:

Dress himself --5 years


Play with group-- 5 years
Tie shoe--6 years

eldery smoker k/c of poorly controlled DM comes with ulcers on tip of three of his toes,
diminished dorsalis pedis bilaterally, however, intact popliteal pulse, what’s the initial
management;

A- Amputation
B- Long term anticoagulation
C- Immediate surgical intervention
D- Diet modification and lifestyle changes
A 2-month baby came with SOB and centroperipheral cyanosis she was diagnosis after
birth but his parents forgot diagnosis what's likely diagnosis
A -TOF
B- ASD
C- VSD
D- coarctation of aorta

Increase cardiac out put , increase... , decrease ... resistance what type of shock
A- cardiogenic
B- Neurogenic
C- Septic
D- Hypovolemic

Patient with sudden severe epigastric pain for 8h associated with vomiting and nausea, on
examination epigastric tenderness,,,,labs only given elevated amylase.
What is most helpful to determine the disease severity or some thing like thing:?
Procalcitonin
ESR
CRP
Alt

A case of AAA in a hemodynamic unstable patient. They painted the scenario that the
man ate food and after a while he started having severe abdominal pain despising giving
analgesic. He became confused and unconscious later in the hospital. PE: Tender and
pulsatile mass, BP low. What is your most appropriate mgt ?
A. US
B. CT
C. Ex Lap

*Unstable
56 Y.0 male patient presented with the classical triad of polydipsia,
polyphagia and polyuria. A diagnosis of Diabetes Mellitus was
made. Which of the following causes impaired wound healing in
diabetic patients?
A. Decreased blood supply. ✅
B. Impaired phagocytosis.
C. Autonomic neuropathy.
D. Dehydration.

Patient known hypertensive on lisinopril and amlodipine, recently diagnosed as


TB
What you will do
1.stop lisinopril cough
2.stop amlodipine
3.increase lisibopril
4.stop rifampicin

Pt received 3 dose of HB vaccine came to clinic


‫ﻋ ﻤ ﻞ ﻓﺤ ﺺ ﻟﻠﻔﯿﺮ و س ﺷ ﻨ ﻮ ھ ﻮ اﻟﻤ ﺘﻮﻗﻊ‬
1.HBsAng
2.HBeAg
3.HBsAB +ve
4. HBcAB+HBsAB +ve

male patient htn DM c/o chest pain examination S4 was heard ECG showed t
wave inversion in lead V2-v5
Troponin high
what is the diagnosis
1. Stable angina
2. Unstable
3. NSTEMI
4. STEMI

48year old diabetic female with dysuria. Urine culture showed 10,000 E.coli .
High wbc and high creatinine. Which abx is contraindicated?

A. Nitro

B. Ceftriaxone
C. Trimethoprim-sulfamethoxazole

A case of PSGN what is the highest confirmatory test Serum albumin


Proteinuria Kidney US

Pt hear heart beat sound 2day no other symptoms examination clear lung
Bp 87/56
O2: 90% C ‫ و‬A "‫ عشان نفرق ب‬ECG ‫زم‬. ‫السؤال ناقص‬
PR:265(regular)
What Dx
1.atrial flutter
2.atrial fibril
3.SVT 4.complete heart block
Case of Asthma exacerbation , patient present with SOB , cyanosis , unable to
complete sentence.
Vitals = Not Sure ! ( Low O maybe) / RR : Not sure .
ABG ; respiratory acidosis.
What is the appropriate next step?
- IV mg sulfate
- Intubation
- Non invasive mechanical ventilation.
- ?/!

A 45-year-old smoker came to the clinic for his diabetes follow-up. During
discussion, he acknowledged that smoking is not good for his health. He plans
to quit this year. According to the Stages of Change Model, at what stage of
change is this patient?

A. Precontemplation
B. Contemplation
C. Preparation
D. Maintenance

Elderly was completely healthy except for elevated BP for the first time
A. Ambulatory BP measurement
B. Start anti HTN ‫السؤال ناقص بس شوفوا الصورة‬
C. Measure the BP two times later on in the clinic
D. Measure BP two time in hom

50-year-old man came with slight limitation in physical activity. Which class of
Heart Failure NYHA?
A-1
B-2
C-3
D-4


65 years old man what’s his systolic blood pressure goal?
A. 120
B.130
C.140
D.150
140/90

62 pt known diabetic
C/O. Cough. SOB. Fever
Chest exam crackle..small pleural effusion
Dx pneumonia
Which of the following is poor prognosis factor?
1.age
2.DM
3.crackles
4. Pleural effusion
‫وبي يحلفون انو بي‬.‫السؤال الذي حير العلماء اطباء ا‬
‫واطباء الريسبا يحلفون انو اي وانا احلف انو سي ههه‬
34 years female at 28 weeks with cough and difficulty in breathing , she
appeared restless and uncomfortable on auscultation of the lung shows bilateral
rhnochi, the nail bed and oral mucous membrane appear pale, she is allergic to
dust, pollen, mold and animal hair. No hx of smoking. She reports that the
symptoms began 2 days perior and started cough with clear color phlegm. All
vital normal.
What it's the most appropriate initial test?
- xry
- Hgh
-Spirometry
-ABG

60yrs Pt known diabetic and HTN


On CCB+lisinopril and indipamide C/O un control HTN which add?
Investigation :normal RFTe
1.thizide diuretic
2.aldosterone anatagonist
3. losartan
4. Metoprolol

Case of patient complaints of abdominal pain, exam tenderness and. The


pulsatile mass above Umbilicus What is the most appropriate investigation?
A.
US
B.
CTA
C. angiography

if stable-Ct, if not stable US

30-40 y.o Adult male medically free presented to ER C/O several episodes of
hematemisis for the first time, takes no medication, no family history of similar
attacks, normal abdominal examintion no guarding no tenderness.

Hgb : 9 ‫ او متعاطي كحول‬NSAID ‫ السؤال ناقص لو كان بي حيكون بالهيستوري‬T‫فع‬


ً
Plt: 250 ‫لو كان سي حيجي البيشنت بفومتينق‬
Alk: slightly decrease ‫لو كان دي حيجي البيشنت هيستوري اوف ليفر ديزيز‬
All other test were normal

What is the Diagnosis:

A-Erosive gastrisis
B-PUD
C-mallory weiss syndrome
D-Esophageal varices

Needs more info i guess


If there is hx of recurrent vomiting
It would be c
If there is hx of liver disease or signs of portal HTN D

16 years old boy come to the clinic by his family with jaundice and fatigue
and splenomegaly, and history of cholecystectomy before for stone, also
History of Required many blood transfusions before, Labs: microcytic
anemia (+/- hemolysis) Highest diagnostic test?

A- Peripheral blood smear


B- Hemoglobin electrophoresis ‫ن السيناريو ماشي مع‬. ‫اتوقع السؤال فيه خطأ‬
C- sickle cell genetic test ‫ن‬b ‫خير‬.‫ بس عطبها با‬hereditary spherocytosis
D- Bone marrow biopsy ‫ختبار‬.‫سيميا لذلك لو جاكم با‬T‫ وصار اقرب للث‬micro ‫قال‬
B ‫ اختاروا‬micro ‫ ولو جاكم‬A ‫ اختاروا‬normo
Whiff’s test positive. What is your diagnosis
A. Candidiasis
B. Trichomoniasis
C. Bacterial vaginosis

Patient on oxytocin, epidural and MgS04, preeclampsia. Her CTG: absence variability (or
non-reactive). What's the cause?
MgS04 toxicity > absence variability
A.
Do MgS04 toxicity Oxytocin > late deceleration
Epiduralanalgesia > prolonged deceleration + hypotensive
B. Oxytocin -

Telegram
$

C. Epidural analgesia
-
A

child have dry non purulent conjunctivitis, cracked red lips, erythema (I think trunk, sole,
hand)
what’s the diagnosis?
- Kawasaki disease
- Rubella
- Measles

Child came with anal itching mother noticed rice like, whats the organism responsible:
A-enterobius vermicularis

Younge female presented with suprapubic pain and heavy vaginal discharge
O/E tender fornices what diagnosis:
1/ cervicitis
2/ vaginitis
3/ endometritis Salpingitis
Senario post deliver pph then ask about management of PPH:
1/ misoprostol
2/ propafol
3/ dexamethasone
No oxytocin in answers

Asthmatic patient on ICS and use SABA inhaler at least once daily
What to add to his medications:
1/ leukotriene antagonist
2/ salmeterol

*LABA

Patient came with right-sided pleural effusion and a positive history of hemoptysis. There
was no shift of the trachea nor the cardiac apex. What is the diagnosis?
A. Rupture of the esophagus
B. Heart failure Answer : C

C. Tuberculosis Both C & D could be right, but I believe


D. Cancer obstructing the ipsilateral bronchus Ctracheal
is the right due to the absence of
shifting that might be seen w/
D d/t atelectasis

Cancer obstructing the bronchus will


…. cause ipsilateral lung collapse >
shifting to the ipsilateral side
-
Earth

18- World health organization (WHO) to determine the health of the population in KSA
What should KSA share to WHO:
A- Health determinant
B- Health indicator
C- Risk factors
D- Something variable

……

Child with recurrent Sinopulmonary infection ard Gi infection His brother dead 6 morth
due to sepsis What is the most likely diagrosis?
A- Iga deficiercy

?
B- X-linked chronic granulomatous disease 2

8
….
- Patient presented with neck swelling, she is completely asymptomatic, neck US was
done reveal a solid mass of 7 - mm with regular margins Which of the followirg is the
most appropriate rext step?
A- FNA
B- Thyroid scirtigraphy
DC-
• Check TSH

……

Bacterial vaginosis scenario with fishy smell no itching What cells will you see?
A- Granular epithelial cells
B- Atypical cells
C- Budding yeast

Which of the following nutrients fail to get absorbed in a patient with cystic fibrosis?

A. Vitamins A, D, E, and B complex


B. Vitamins A, B, E, and K
C. Vitamins B1, B2, B12
D. Vitamins A, D, E, and C
E. Vitamins A, D, E, and K

Female complaining of abnormal uterine bleeding she has an endometrial polyp, on US


endometrial lining was 19mm, what will you offer to this patient at this stage?

A) open hysterectomy
B) laparoscopic hysterectomy
C) hysteroscopy with polypectomy

82 year old male with very painful micturition what is the most approppriate management
1. Abx for uti
2. Foley catheter
3. Cysoscopy and turp

‫ ﻋﺎدٮ(ﺔ‬uti ‫ﻫﺬي‬
‫ﺣﺘ*ٮﺎر ﰷن‬5‫*ٮﺲ *ٮﺎﻟﺴ(ٮﻨﺎريﻮ *ٮﺎﻻ‬
Unable to urinate ‫وﰷن واﺿﺢ زي اﻻو*ٮﺴﺘﺮﻛﺸﻦ‬

foley catheter ‫اﻧﺴﺐ ﳾ ﻫﻮ‬


*
telegram
Q1_ The MOH is organizing campaigns, lectures, and health education in order to teach
the public about the dangers of obesity and it’s associated complications.What kind of
prevention is this?
A. Tertiary
B. Secondary
C. Primary✔
D. Primordial
━━━━━━━━━━━━━━━━

Q2_ The government has decided to set up campaigns to help increase the
awareness of hypertension to the public, as well as educate them about the risk
factors of hypertension and encourage low salt diets. What kind of prevention is
this?
a. Primordial
b. Primary✔
c. Secondary
d. Tertiary

〰〰〰〰〰〰〰〰〰〰〰

Q3_Secondary prevention of diabetes is :


A-Screening✔ Education= primary
B- chronic management of disease Screen= slecondry

C- education in health and disease


━━━━━━━━━━━━━━━━

Q4_.Pregnant woman has Hepatitis B which type of prevention we should preformed :


- primary
- secondary
- tertiary✔

〰〰〰〰〰〰〰〰〰〰〰
Those Q answered by a public health specialists (MPH).

━━━━━━━━━━━━━━━━

Her Note :

primordial
‫ باختصار اذا شوفتوا السؤال يقول‬:
‫ضرائب = ‪Taxes‬‬
‫او‬
‫ساسية صحية = ‪Policy‬‬
‫‪ primordial . ...‬هنا اختارو‬

‫‪ :‬اذا شوفتوا ‪ Primary‬بينما ال‬


‫‪Health Education‬‬
‫‪Health promotion‬‬
‫حمالت توعية =‪Awareness‬‬

‫‪ Scondary :‬ال‬
‫‪ .‬للمرض ‪ Screening‬مع ‪ S‬يكون من اسمها حرف‬

‫〰〰〰〰〰〰〰〰〰〰〰‬
‫‪Primordial‬‬
‫شي يتعلق بالحكومات والوزارات‪ .‬مثال قرار يطلع ممنوع التدخين في االماكن العامه‪ ،‬ممنوع استخدام الزيوت‬
‫المهد رجه‪ ،‬يجب وضع السعرات الحراريه لكل وجبه‪ ،‬زيادة ضرائب التدخين‪ ،‬ممنوع بيع العاب االطفال الي تكون ع‬
‫شكل دخان‪،‬‬

‫‪ Primary‬ولكن اذا الوزاره طلعت تسوي حمله هدفها توعي الناس تكون‬

Extra

on

Patient received 5mg warfarin his inr is 7 , known case of AF ?


A. Reduce to 2.5 mg
B. Same dose
C. Stop warfarin
D. hold and repeat inr
breast feeding mother found unilateral mass on the lower outer quadrant of right breast (i
think 3x3 cm) it was erythematous and hot on palpation. Mildly tender. Most appropriate
management?

A. chemo therapy
B. radiation therapy
C. needle core biopsy
D. Aspiration + Abx

Which of the following dermatological condition is associated with Sarcoidosis?

A. Erythema toxicum
B. Erythema nodosum
C. Erythema multiforme
D. Erythema larva migrans
E. Erythema marginatum

Which of the following findings does not indicate a poor prognosis for pancreatitis?

A. Metabolic acidosis
B. Hypocalcemia
C. Coagulopathy
D. Hypoglycemia Metabolic acidosis

E. Hyperglycemia

6yrs with inability to bear weight on exam there was bilateral knee pain. Labs
showed positive ANA. Juvenile onset arthritis is suspected what type could he
have?
Oligoarthritis
JIA

Which of the following is not a sign of severe immediately life-threatening asthma?


A. Reduced level of consciousness
B. Bradycardia
C. Inaudible breath sounds on lung auscultation
GO
D. Reduced PaCO2

?
Woman did some procedure (pelvic procedure but forgot what it was) and then came
complaining of urine from vagina during urination(micturition)
What is the dx?
A.Vesicovaginal fistula
B. Ureterovaginal fistula
C. Ureterovaginal fistula Uretherovaginal fisula
→ earth

D.Rectovaginal fistula Idk I couldn’t find anything


abt urerthovaginal fistula
on upt or Amboss

A child starts to develop an awareness for strangers and separation anxiety. How old is
the child in months?
a. 6 months
b. 7 months
c. 12 months
d. 24 months

Stranger anxiety at 6 months


Separation at age of 12

Pregnant with warts in vulva ttt?

A. Cryotherapy
B. electrophoresis
]

Patient 60 something k/c of dyslipidemia on statin report high blood pressure reading of
150/90
In the clinic
Labs all normal
Bp : 145/90
What’s your management?
A- Start amlodipine.
B- life style modification and weight reduction.

Pancreatitis 5 weeks ago. Now she has epigastric tenderness and cannot tolerate food
with
vomiting each time. By ultrasound you found large about 12X10 mass with thick wall
and fluid
inside. Labs: 346 amylase, Wbc 15k. What is the diagnosis? *
A. Pseudocyst A case of Pancreatitis 5 weeks ago. Now she has
B. Abscess epigastric tenderness and cannot tolerate food with
vomiting each time. By ultrasound, you found large about
C. Walled off necrosis 12X10 mass with thick wall and fluid inside which is
heterogenous and non-liquefied. Labs: 346 amylase,
I ' D
Wbc 15k. What is the diagnosis? f-

Istv an

A.Pancreatic Pseudocyst
B.Pancreatic Abscess C. telegram
• Walled off pancreatic necrosis

Patient RTA with head trauma, increased urine output, decrease in Urine osmolarity
increased blood osmolarity?

A. Central diabetes insipidus


B. Cushing •

C. Conn’s •

D. Nephrogenic diabetes insipidus

Septic shock case indicates adequate systemic perfusion?


To date, the most promising approach combines standard monitoring variables (blood
pressure and urine flow) with monitoring of both CVP and SCVO2 to guide goal-
directed therapy early in the course of shock.
A-Cardiac index Mixed venous oxygen saturation SMVO2 did not improve length of ICU stay or length
B- Mixed venous oxygen saturation (SMVO2) of vasopressor treatment and was associated with significantly higher cost.

C-Central venous oxygen saturation (SCVO2) Update believe that pulmonary artery catheters (PACs) should not be used in the
routine management of patients with sepsis or septic shock since they have not been
shown to improve outcome
PACs can measure the pulmonary artery occlusion pressure (PAOP) and mixed venous
oxyhemoglobin saturation (SvO2).
However, the PAOP has proven to be a poor predictor of fluid responsiveness in sepsis
and the SvO2 is similar to the ScvO2, which can be obtained from a CVC.
-

Dr. Ali Botharith


Pt in ICU received 15 units of blood, now blood coming out from NGT, incision, and
cannula site
1. Transfusion reaction
2. Thrombocytopenia
3. Hypocalcemia
4. Von Willebrand

Patient presented with sweating dyspnea, palpation and headache. She is a known case of
HTN and despite taking medication it’s not controlled. imaging reveals supra renal mass.
Which of the following meds u will give to control her HTN?

A. CCB
B. Alpha blocker
C. ACEI
D. BB

Normal vaginal delivery, Baby weight 4.2kg, Laceration which degree:

A-First
B-Second
C-Third
D-Forth
- wattle

Patient with TIA symptoms and Afib, what to give?

A. Aspirin
B. Warfarin INR goal 2-3
C. Warfarin INR goal 3-4
D. Enoxaparin

A Patient had lower parathyroidectomy, came to the clinic with bone pain, numbness
around the lips. Lab results showed high Ca + high PTH. What’s the most likely dx?

A. Missed adenoma
B. New adenoma
C. Parathyroid hyperplasia

Patient with raised JVP, Hypotension, unclear heart sound, clear lung sounds. How to
confirm the diagnosis?

A. ECHO
B. ECG
C. Chest X-ray
D. PFT

Patient with HTN on 25 mg once daily hydrochlorothiazide. BP not controlled.


Management?

A. Double thiazide dose


B. Add ACEI
C. Add beta blocker

A female get high energy accident (high velocity), with seat belt sign, on X ray have
chance fracture.
What will associated with this fracture?
A- Duodenal perforation
B- Gastric perforation
C- Jejunum perforation
D- Vena cava perforation

- Mont

15-year-old girl has sparse pubic hair not reaching the mons pubis, breast budding with
areolar enlargement, but with no clear distinction between breast and areola. No contour
of breast was seen. What Tanner stage is she?

a. Tanner I - B
b. Tanner II
c. Tanner III
d. Tanner IV

When does bedwetting becomes alarming?

A. 5 years
B. 6 years
C. 7 years
D. 8 years

Thyroid cancer with increased calcitonin:

A. Medullary thyroid cancer


B. Follicularthyroidcancer

G2p0 20w gestation with cervical incompetence (cervix length 30), Management??

?
A cervical cerclage
B progesterone supplementation Idk I believe it’s A cuz in
C strict bed rest Amboss it say the time is
less than 24 weeks
Note
Depends if 30 mm and cervix open not contracted =A
30 mm and cervix closed=B


waka

patient has midsystolic murmur grade 2/6 best heard at right lower sternal border, and has
S4 NO S3 best heard at the apex. What is the most likely diagnosis?

A. Aortic stenosis
B. Mitral regurgitation
C. Aortic regurgitation
D. Mitral stenosis
1YO P5 +3 presented to the clinic complaining of abnormal uterine bleeding her
Menstrual period is regular, associated with blood clots and pain that is not relieved by
analgesic she had previous myomectomy, she is a known case of PCOS&her BMI is 40?

A-Adenomyosis
B-Endometriosis
C-Uterine fibroid
D-Endometrial hyperplasia

What anti diabetic medication can reduce mortality in DM patients


A. Metformin
B. Gliflozin
C. Glipizide
D. Acarbose

4 years. old girl with decrease head growth, decrease social interaction, decrease in
language , what is the closest ddx?

— Autisim
— mental retardation
— Rett's disorder
— aSPERGER syndrome

A 54 yo female medically free comes for routine checkup ,Her cardiac exam revealed
grade 4 pansystolic murmur heard best at the apex and radiated to the axilla, she is
asymptomatic and the rest of her exam is normal, best next step to confirm the diagnosis?

A. TTE
B. TEE
C. ASO TITER
D. CHEST X ray
← Google

Hereditary spherocytosis picture, what will you order?


Osmotic fragility test (ans)

• Baby with a rash, both parents have eczema, where do you think the location of the rash
will be? On face and scalp.

• Apgar score q

Patient who’s diabetic, develops liver symptoms, dx?


A Nonfatty alcoholic jaundice
B viral jaundice

• Post splenectomy when to give vaccine?


2 weeks ,
2 months,
6 months ‫جاء‬ }

• Patient with asthma symptoms that are not controlled although he uses Bronchodilators,
corticosteroids, & LABA. His PFTs are 75% before the use of a bronchodilator and 95%
after using it. What to do?
Observe how he uses the inhaler

• When to check bacteriuria in pregnancy? 29/12

At 12th week ‫جاء‬ -screening for asymptomatic bacteriuria in Pregnancy


-12week
-26week

• Child came to clinic and the doctor advised his parents to start iron supplements next
month, how old is the child now?
3 months

• A child who plays in the farm or garden something like that, he has increased saliva,
tearing, diarrhea, sweating.. dx?
Organophosphate toxicity.

• Child with hip joint pain and refuses touching it, what’s the organism?
S.aureus ‫جاء‬

• A patient with a prosthetic valve who is about to do a dental procedure, which


prophylactic antibiotics?
Amoxicillin
Penicillin

Pt 10 years old no co-morbidities with history of rheumatic fever with no cardiac


involvement how long should her receive prophylactic antibiotics for?
5 years
10 years ‫جاء‬

Early sign of compensated shock?


What is the early sign of
Pale and cold skin. compensated shock
1- anuria
2-hypotension
3-confusion
4-pale and cold skin

What decreases mortality rate in esophageal varices?


IV ceftriaxone

Female has molar pregnecy bhcg at begining was (856 000)


On week one 560
Week 2. 360
Week 3 160
Week 4. 56
Week5 1.Yesitisone
*numbers are not exactly same but somehow near
How you will follow the patient
A. Measurement at weekly interval
B. Measure at one month interval
-
A
C. Discharge from clinin ( sure was like this)
D. Repeat ultrasound
Pregnant bp 140/90 or 150/90 what to give ? “Exactly like this”
A- nifidepine
B- hydralazine According to ACOG
Oral Antihypertensive Agents in Pregnancy for chronic HTN , gestational HTN, or
C- methyldopa preeclampsia without severe features
1- Labetalol (first line)
No labetalol in choices 2- Nifedipine
3- Methyldopa
Antihypertensive Agents Used for Urgent Blood Pressure Control in Pregnancy
1- Labetalol (first line)
2- Hydralazine
3- Nifedipine - watta

50 yr female c/o menorrhagia for one year , she has fibroid 7x7
medical tx failed, what’s the most appropriate for her?
- hysterectomy
- uterine artery embolization - Hormonal IUD

Female in 30s or 40s has excised fibroadenoma, histopathology showed (forgot type of
cells) with hyperplasia and atypia, which factor suggests malignancy?
A- Age
B- presence of Atypia
C- presence of hyperplasia
D- can’t recall ‫جاء‬

During the examination the doctor can feel the presenting part is soft?
- face
- Brow
⁃ Breach

Patient with SLE symptoms what's the most confirmatory test ?


A- ANA
B- Anti DsDNA
Case SLE + very high ferritin very low hbg high alkaline phosphatase? Most likely
anemia?
Anemia of chronic dx bc of the very high ferritin [In reply to FAIR ]
Hemolytic high ferretin = chronic
high retics = hemolytic
Pernicious high retics and igG = autoimmune
hemolytic
Aplastic case dependent because sle presents
with all

case of rheumatoid arthritis compliance on MTx, improve and stable for years without
attack, the LFT abnormal, what will you do?
A: stop MTx
B: adalimumab

At which age can we give peanuts and eggs to babies to prevent the risk of getting
allergic? Timing of first exposure — The timing of introduction of a food probably influences the development of allergy versus tolerance. Peanut allergy has more than doubled in young children in countries where
delayed introduction of peanut until at least three years of age was recommended [1,4]. In addition, the rate of peanut allergy is lower in countries where peanuts are introduced at a younger age [6,14,52].
However, dietary advice was generally poorly followed in countries that recommended delayed introduction, so the impact of these recommendations is unclear [4]. One study noted that the age at first

A. 10 months ‫جاء‬
exposure in children who developed peanut allergy and age at initial reaction (19 and 21 months) were lower during the period in which delayed introduction was recommended compared with the age of
exposure and allergy onset (22 and 24 months) in the period prior to those recommendations [53].

The Learning Early about Peanut Allergy (LEAP) trial was the first randomized trial to show benefit of early introduction of a major food allergen, with earlier introduction of peanut at 4 to 11 months of age

B. 14 months associated with a decreased risk of developing peanut allergy [43]. Introduction of highly allergenic foods is reviewed in greater detail separately. (See "Introducing highly allergenic foods to infants and
children", section on 'Suggested approach'.)

C. 18 months Other factors that may determine whether an individual becomes sensitized or tolerant to a food allergen are discussed in detail separately. (See "Pathogenesis of food allergy", section on 'Factors
influencing sensitization or tolerance' and "Pathogenesis of oral allergy syndrome (pollen-food allergy syndrome)" and "Introducing highly allergenic foods to infants and children", section on 'Introduction in
a high-risk population'.)

D. 24 months

27 y old fenale present with acute lower abdominal pain radaite to left shoulder whats is
the highest diagnostic investigation
A. Pelvic CT
B. Abdominal MRI
C. Pregnancy test

Scenario with symptoms of hypocalcemia( mouth twitching and spasm), whats Tx:
A. Oral calcium
B. IV calcium
- strawberry cervix and yellow green discharge?
Trichomonas vaginitis ‫جاء‬

- A baby with stridor noisy breathing and wheezing that improves when prone and
increases when supine. What is the diagnosis?
A.chest X-ray
B. Nasopharyngeal something
C. Respiratory culture
D.improve with his first birthday

Cryptorchidism typically resolves without


treatment via spontaneous descent of testicles
Persistent cases require surgery, which should
5 y/o boy one testicles undescended. Mx? be performed as soon as possible after 6
months of age.

Orchidectomy Orchidopexy
Open/laparoscopic orchidopexy is used when
testes are not palpable in the scrotum.
Orchidopexy ‫جاء‬ Exposure and fastening of the testicle to the
scrotum
Orchiectomy: in cases of nonviable testicular
Wait till puberty remnants or late discovery of undescended
testicle (> 2 years)
Close urological monitoring and early treatment
are necessary in individuals with an increased
risk of testicular cancer and infertility.

Pt Elderly tired, thirsty on ex abdomen tender rigid and for investigation What is next in
management:
Hydration
Endoscopy
Antibiotics

Trauma patient. Vitally was stable. FAST done was positive next step?
Peritoneal lavage
Ct abdomen ‫جاء‬
Laparotomy
Diagnostic laparoscopy
child with perforated dum and discharge come out what is the diagnosis
1) otitis media
2) otitis externa child with perforated dum and
discharge come out what is the
diagnosis
1) acute OM
2) otitis externa
3)Chronic OM

treatment of brucellosis I am sure no neurological symptom


6 weeks
3 weeks
3 months
6 months

12 years old found to have klebsiella colony 100000 from mid urine no sign or symptom
1) no need to treat
&
2) treat as acute UTI
3) give empirical antibiotic ◦ A

Type of vitamin D given in nutritional rickets?


type of Vit d in child with rekits D1
D2
1) D1 D3
D4
2)D2
3) D3

e.
12 years old boy diagnosed with DM type 1 he has been complain with hypoglycemia
since he take his medication 2 month ago and not reach control
1)brittile phenomena
249.12 years old boy diagnosed as DM1, he
2) down phenomena is compliant with insulin and diet for 2
3) smoggi phenomena months, now he is suffering from
4) honeymoon phenomena hypoglycemia. What is this called
A- Brittle diabetes
B- Smogey effect
C- Honeymoon phase
D- Dawn phenomenon

pediatric known case of HF and HTN ,lower limb edema and dyspnea on exertion what to
give
1) frusemide
2) reassure
3) BB

child come with fever cough and diffuse wheez what is the mangement
1) admit for O2 and antibiotics (not sure) child with fever and cough was diagnosed as
2) admit for O2 and fluid
µ bronchiolitis case. on exam there is diffuse wheezing
and sun costal retraction.
3) discharge patient with antibiotic what is the approprite management?

- admit patient for oxygen and antibiotics


•- admit for oxygen and fluid
- admit for steroids and bronchodilators

38 weeks dilivary complain from rachypnea and granting


what is the diagnosis
1) transient tachpnic attack transient tachypnea attack
2)RSD
3) muconium aspiration ☆ Note :
Either way choose A as wafa
said fetal distress >> TTN

child with forign body aspiration where is the exact location


1) rt bronchi
2) left bronchi

child with inhaled seeds for several month and his mother bring him many time to
hospital and discharge him
now came with wheez and pnemonia in right lower lung how to treat
1) regid bronchoscope
2) flexable bronchoscope

9 years old child came with his mother with anuresis what is the muscle affected
1) detruser
patient came with sign and symptom of epiglotitis drolling saliva and severe ill ask about
treatment
1) discharge with antibiotic
2) intubate the patient and admit to ICU

1m year old came with eczema on the face trunk elbow what is the diagnosis
1) infintile eczema
2) edupathic urticaria

Pregnant with suspicious of cervix lesion what next


a. Cone biopsy
b. Cur Rita get
c. Colposcopy
d. Pap

* in general (UTD)
• Cervical biopsies ---> relatively contraindicated
Specifications (Kaplan)
• Pap smear -> indicated as 1st trimester test (routine)
• Colposcopy -> can be done
• Ectocervix biopsy -> can be done
• Endocervical curettage (ECC) -> CONTRAINDICATED in pregnancy
. 31-year-old female was following up for the last 5 year with women health, last year
negative and this year, negative Pap smear and negative HPV Test, next time to do Pap
smear:
a) 6 months
b) 1year
c) 2 years
d) 3years

single umbilical artery what you suspect - What is associated with a single umbilical
artery in a newborn?
1) diabetic mother A. Congenital anomalies 80% B. Maternal
Diabetes Mellitus

An adnexal mass was felt in healthy female with normal pelvic examination, no
pregnancy, had her menstruation 2 weeks? - Young female with adnexal mass healthy, with normal
pelvic exam, -ve pregnancy, last period was 2 week ago ,
a) follicular cysts (not sure) what is the Dx? :
A-Follicular cysts
b) luteal cysts B-Corpus Luteal cyst

c) pco
*Follicular cyst >* usually not hemorrhagic no blood +
asymptomatic (incidentally diagnosed, examination normal)

*Corpus leuteal cyst* > usually hemorrhagic, bloody,


4 month milestone: painful, and might rupture causing hemoperitoneum

A. Sit without support


B. Head control
C. Start trying to crawl
D. Follow objects ects

case of patient complaints of abdominal pain, exam tenderness and. The pulsatile mass
above Umbilicus What is the most appropriate investigation?
A. US
B CTA
C. angiography

* if it’s CT only then choose it

Extra Note:
Cystocele only = anterior colpoperineorrhaphy
- Rectocele only= posterior colpoperineorrhaphy
-cystorectocele= Ant&post colpoperineorrhaphy
- both of them + uterine prolapse=fothergill's operation ( manchester)

Girl Every month there is exacerbation sob relieved by nebulized in emergency what to
add
A. add inhaler budesonide
B. chromylyn
C. slow theophylline
D. oral steroids Note
Depends if pt still in ER=D
If for long term management to reduce ER visit= A

A girl who’s on low mood, increased appetite, sad, irritated that usually happens before
her menstruation & she has abdominal pain associated with her menstruation that affects
her social life, what treatment to put her on?
A. OCP
B. selective serotonin uptake inh
young male came to well baby clinic, upon examination his right testis was
palpable in the inguinal canal and small in size and easily moved to
scrotum, the left is normal, what is the cause?
-ectopic testis
-undescended testis
-testicular torsion
-Retractile testis

Drugs = Antidote

1⃣_ Acetaminophen (paracetamol)--➡️' N-


Acetyl Cystine (NAC)

2️⃣Beta Blockers➡️ Glucagon

3️⃣Opioids➡️ Naloxone

3️⃣Anticholinergic➡️ Physostigmine
4⃣ Benzodiazipines--➡️Flumazenil

5⃣ Warfarin--' ➡️FFP (early) or Vit K (late)

6⃣ Heparin ➡️Protamine sulfate

7️⃣ Insulin--➡️ Glucagon

8⃣ Iron---➡️ Desferoxamine

9⃣Digoxin ➡️Digoxine immune Fab

CCB ➡️Calcium chloride

1⃣1⃣Mg+2 Sulfate ➡️Calcium gluconate

1⃣2️⃣ASPRIN➡️sodium bicarbonate
female pregnant with hypertension and proteinuria, she has right upper quadrant pain
what is the reason ?
A- Distended Hepatic Capsule According to ACOG: Pain is thought to be due to periportal and
B- Hepatic Rupture focal parenchymal necrosis, hepatic cell edema, or Glisson’s capsule
distension, or a combination.
C- Gallbladder Stone Wafa -

57 yo woman has 3rd degree hemorrhoids (with NO bleeding) which of the following is
the appropriate management?
A fiber supplementation
B rubber band ligation **%
C surgical hemorrhoidectomy

Pt did upper endoscopy and there is squamous carcinoma Which of the following is a risk
factor of esophageal cancer?
A GERD
B Barret’s esophagus
C Esophageal stricture
D.smoking *
*D because its squamous
if it says Adeno carcinoma then its B

What is the most common cause of bleeding in postmenopausal women ?


- endometrial atrophy
-endometrial cancer

Patient on oxytocin, epidural and MgS04, preeclampsia. Her CTG: absence variability (or
non-reactive). What's the cause?
MgS04 toxicity > absence variability
Oxytocin > late deceleration
A. MgS04 toxicity Epiduralanalgesia > prolonged deceleration + hypotensive
-

Telegram
$

B. Oxytocin

:
C. Epidural analgesia

eldery smoker k/c of poorly controlled DM comes with ulcers on tip of three of his toes,
diminished dorsalis pedis bilaterally, however, intact popliteal pulse, what’s the initial
management;

A- Amputation
B- Long term anticoagulation
C- Immediate surgical intervention
D- Diet modification and lifestyle changes

Blood loss of 25%, what to expect to be decreased?


- RR
- Pulse pressure
- GCS scale
- Urine output

Important DDx of Vaginal Conditions 🛑

-Dyspareunia +/- dysuria + frequency in > 51 YO ( +/- vaginal itching /dryness) ➡️


suspect Atrophic vaginitis ( Topical estrogen cream).
- Itchy , tender white plaque of the vulva ( become itchier at night) ➡️ Lichen
Sclerosis ( Topical steroids | Follow up) .

- White Thick discharge ➡️ Candida (Vaginal Thrush ) ( Topical clotrimazole).

- Yellow-greenish offensive discharge + Vaginal itching +/- Strawberry Cervix +/- pH


> 4.5 ➡️ Trichomonas Vaginalis ( Trichomoniasis) ( Metronidazole )

- Offensive discharge WITHOUT itching +/- fishy smell +/- pH > 4.5 ➡️ Bacterial
Vaginosis ( Gardnerella Vaginalis ) ( Metronidazole)

‫الصور‬

Falciparum malaria

Rooting reflux
ABSENT RED REFLEX

ENDOMETRIOSE
BASAL CELL CA

ANEMBREONIC SAC
Man wants to travel and seeking travelers diarrhea prophylactic treatment , his labs show
abnormal KFT. What treatment to give ?

— A- Ciprofloxacin
— B- Bismuth
— C- No need
?
Most ppl answered no
— D- Probiotics need but idk I think it’s
bismuth

Whiff’s test positive. What is your diagnosis


A. Candidiasis
B. Trichomoniasis
C. Bacterial vaginosis

Child c/o fever, bloody stool and tenesmus, abdominal examination showed abdominal
distention, Dx? A- m e biases

A. Ascaris
B. Amebiasis
C. Giardiasis
D. Rotavirus
Pediatric case h/F sore throat & fever then 3 days he develops body rash on the thighs
and buttocks.
Lab finding suggest renal insult with anemia and normal Plat count) plus ll edema +black
tea urine.

A. -post streptococcal GN
B -HUS
C -HSP (Henoch-Schonlein Purpura)

according to celiac disease what is the appropriate to confirm diagnosis ?


- Biopsy
- ani tans and ani endo
- Gluten free diet and relieve of symptoms
other Male with perianal swelling
and discharge then ruptured:
a→ ‫ڡﺎل ﻋﻨﺪه ﺣراره‬L‫ى و‬J‫ﻫﺬا اﻟﺴؤال *ﺣﺎﻧ‬
‫ * ﰷن واﺿﺢ‬Fever

• Analfissure
• hemorrhoid
• anal abscess
Bo
• Analfistula

dult with Hypertension and Dm has 2 months history of Small peranal paniful Swelling other Q :

Bp: 160 / 100 ? * note from telegram


"Pain truly caused by hemorrhoids usually
arises only with acute thrombus formation.
Preanal abscess This pain peaks at 48-72 hours and begins to
decline by the fourth day as the thrombus
Preanal fistula organizes. However, pain from distention of
the overlying skin by the blood clot may last
Thromboed pile 5-12 days. New-onset anal pain in the
absence of a thrombosed hemorrhoid should
prompt investigation for an alternate cause,
such as an intersphincteric abscess or anal
fissure."

Elderly female with Hx of COPD came to you complaining of SOB DYSNPEA She is
using LABA, LAMA, SABA, ICS and i think other medications , she did chest
physiotherapy Vitals are stable ABG normal O2%90-93% PH normal Co2 normal As
much as i remember everything was normal What is the next step? A- repeat
physiotherapy
B- add oral steroids
C-long term O2 therapy
D- Mechanical ventilation

• A -
Alhavbi
c-s surgeon couldn’t control bleeding and discuss it with the assistant consultant and they
decide to do hysterectomy to save patient life
A procede with hysterectomy without consent
B Be limited on what was mentioned in the consent
C take husband consent
D wait for concent or something?
Uterine fibroid
Drug given for long therapy of esophageal varices :
BB

Abruption placenta 32wks and bleeding stopped your action??


_admission and follow up in hospital
_discharge and normal follow-up
_high risk and follow-up with serial u/s
_

37 Y/O Female pt with family Hx of mother had brest cancer & sister had ovarion cancer
which screening test is appropriate for her?
A) Annual breast mammogram starting from now
B) Annual breast mammogram starting at age of 40
C) annual screening….
D) ….
Health organisation “good health for all” the year 2️03️0 will be the end of preventable
death
for children under 5 y/o by ?
A. School health
B. Health education
C. Immunizationion

5- 14 y/o boy ingested a bottle of acetaminophen tablets brought by his parents


20 hour later, asking about which stage of toxicity?
A1
B2
C3
D4

An 8 years old girl ingested 2 pack of paracetamol after a fight with her mother come
after a 24 of ingestion with RUQ pain in which stage of toxicity she is
Stage 1
Stage 2
Stage 3
Stage 4

Which of the following medications, when used alone as maintenance Therapy in


persistent asthma, is associated with an increased risk of asthma-related mortality?
A- Inhaled fluticasone
B- Inhaled salmeterol
C- Oral zafirlukast
D- Oral prednisone

Timing of cleavage for monochorionic monoamniotic twins: 8-12 days

Otitis media most common organism viral


-rhinovirus
-rotavirus
- coronavirus

*First RSV , then rhinovirus

12 year old received a nonspecific blunt trauma on his abdomen and later presented with
generalized abdominal pain. Imaging of the spleen showed
a 7 mm hematoma and 4 cm tear (grade 3). Your management:
A. splenectomy
B. Spleen preserving surgery
C. Conservative

A patient is admitted to ICU and intubated because of a shock . Which of the following is
indicating systemic perfusion ?
- cardiac index
- central venous pressure
- Mixed and central venous oxygen saturation
Alharbi
if the Q says )systemic pressure ) then choose central venous pressure
-

A pregnant in 8 weeks GA , US showing a fetus without viability . What is the


appropriate management ?
- misoprostol
$ !

Note to remember (ACOG+UTD)


Inventible, incomplete, and missed abortions are managed by:

- Expectant management (if ≤13¬ weeks GA)

Up tp 8 weeks, if the pregnancy has not passed in a reasonable time or if bleeding,


infection, or other complication develop -> Managed by medical or surgical.

- mifepristol - Medical (up to 20 weeks GA)

- surgical By two different medications mifepristone followed by misoprostol


- conservative management **considered in women without infection, hemorrhage, severe anemia, or bleeding
disorders

- D&C (up to 20 weeks GA)

**considered in women who present with hemorrhage, hemodynamic instability, or signs


of infection should be treated urgently with surgical uterine evacuation.

Drug to be safely givin to ckd 3rd stage:


Metformin
Nitrophoratin Warfarin is the only safe among those 4 drugs
Warfarin
Lithium

What is the absolute side effects of amitriptyline?


— A- weight gain
— B- Constipation
— C- Insomnia

Which one of the following drugs is a selective beta-1 agonist?


A- Amrinone
B- Digoxin
C- Dobutamine
D- Dopamine
other
Preeclampsia what
Q .

expected to be significantly
decrease ?
1- Urea
2- Creatinine
Pregnant with sever preeclampsia which of the following is significantly decrease 3- Plasma volume
4- Platelets
(written like this not increase I read it several times to make sure ) :Let’s Exclude!!
- Uric acid -> Serum uric acid increases with
A) Serum urea A •
preeclampsia
- Creatinine -> it may increase with preeclampsia
B) plasma volume not decrease
- Plasma volume?? -> it increases in pregnancy in
C) Serum creatinine general
- Thrombocytopenia -> may occur and may reach
D) Serum uric acid severe levels as part of HELLP

NO platelets
‫ڡﺎء‬U‫كﻼم دﻛﺘورة و‬
Scenario of pt alcholic presented with jaundice fatigability only no fever other wise
asymptomatic and on examination you found soft smooth enlarged live span 13 cm
No splemnomegly
There was a list of labs investigation:
AlT.,AsT mildly elvated
high ferritin = C
Alp mildly elevated
Bilirubin high
Iron was high
Ferretin was very high 450 compared to Normal in exam
HCV negative
HBsAg negative
HBsAb apossitive
What is the most likely diagnosis?

A) chronic Hepatitis B
B) alcoholic Hepatitis
C) heamochronatosis
D)

hild had pneumonia (labs show leukocytosis) and in oral Abx for one week mother till his
symptom improved., what next :

— A- No need more investigation.


— B- Culture and sensitivity

A diabetic patient presented with exudates from a wound in his leg with poor healing and
no
sign of inflammation. What is the cause of the poor wound healing in this diabetes
meilitus patient?
A. Decreased phagocytosis process.
B. Stimulated bacterial growth.
C. Decreased immunity.
D. Increased blood supply to the wound

atient with multiple joint ache, Malar rash, positive Anti Smith. C3 and C4 positive,
creatinine high, liver functions high too. Treatment?

A. Chloroquine n methotrexate
B. Chloroquine and MMF
C. Methotrexate and ibuprofen
Long case of COPD patient with oxygen saturation 88% and PHTN is 50 What improve
survival?
A) Long term oxgen
B)oral steroid
C) over night pulse oximeter

Trauma patient presented with a wound on his thigh, subcutaneous fat is lost but
vasculature is intact. What provides the best management?

A Debridement with primary closure


B Primary repair
‫ابو عطيه‬
C Debridement with secondary closure
D Debridement with vacuum-assisted closure

Vascular intact
SO closure with graft
‫ لو قال‬necrotic skin ‫ نختار‬debridment with vaccum
‫ لو قال‬scalp laceration ‫نختار‬debridment with primary closure

30y male came with dyspnea and chest pain PND orthopea
O/E pansytolic mumur gallop s3 with displaced apex no any significant medical hstiory

?
most appropritate next step?
A/chest xray
B/EcG I think B
C/Echo

ediatric with seizure and other signs, labs showed metabolic


acidosis, urine analysis: aminoaciduria and glycosuria, and
cataract what’s the diagnosis?
A. Galactosemia
B. Homocystinuria
C. Methylmalonic Academia
D. Maple syrup urine syndrome
Parent’s brought their child to the hospital for inability to move his arm xray showed
fracture
at both distal ulna and radial bone. The child looks healthy and dressed well, and good
hygine they recall that 2 days ago the mom droped a plate at the child arm and noticed he
couldn’t move his arm and after 2️nd day the child couldn’t move his arm they didn’t
notice
any brusing so they thought it’s just a strain. Which of the following is indicative of child
abuse.
1)Type of fracture Why not A?Diaphyseal
fractures common in abuse,
2) Delayed presentation here distal fractures

3) Consistent story of the parents


4) the appearance of the baby

9- A child with pica, hepatosplenomegally and failure to thrive. Coming from a low
socioeconomic economic status family.
HGB low
Lead - 2 high
PT normal
PTT normal
INR normal
Iron normal
What’s the most appropriate treatment?
A. Vitamin K
B. D-pinicillamine
C. Iron supplements
????

?
if lead above than 2 , then choose C

Supracondylar fracture case with no pulsation (not mentioned cold or warm)


A- k wire -12 year old boy injured with supracondylar
fracture and distal radial pulse absent , which of
B- surgical exploration the following is the appropriate next step in
management?
A-K wire
B-intramedullary nail
C-surgical exploration
D-Closed reduction

9 month came to clinic with fever irritability disturbance of sleep on exam performed
drum
with pus in external canal
A. rotavirus
B. rhinovirus
C. adenovirus
D. coronavirus

A patient underwent sigmoidoscopy found sigmoid lesion, histology report: Sigmoid


Adenocarcinoma with free margin with no invasion to mucosa. What is the next step?

A- Abdominal CT scan
B- Sigmoidectomy
C- Pelvic MRI
D- PET scan

A for staging
B if the pt came with obstruction

Note

BIRAD IV - core biopsy

Phylloid - mastectomy
(IF large size)
If small wide local excision

Screening for 37 female her mother had brest Ca, sister ovarian Ca
- breast US
- mammogram
- don't remember the rest
(BRCA)
High vascular resistance low Cardiac output
Shock type
-hypovolemic
- cardiogenic

Asymptomatic bacteriuria screening in pregnancy -- 12 w

29/12
-screening for asymptomatic bacteriuria in Pregnancy
-12week
-26week
Introduce peanuts to child - 10m

Neuropathy pain TX - amitriptyline

Sogren, hyokalemia RTA type - 1

Symatrical uterus enlarge - adenomyosis


Tnf

◦ Aspergillosis tz - voriconazol

Adult with meningitis



A- ceftrixone + Azethromycine +steroid ◦
B- Azethromycine + ceftrixone
C-Vancomycin
Answer:ceftrixone +vancomycin +Dexa

Pregnant with HTN on hydrothiazide + ARBs >


- stop all medication
- continue all medication
- stop ARBs give methyldopa

Prsthatic valve prophylaxis


Vancomycin + gentamycine + rifampicine

582-29 y/o male with few days history of Seizure and confusion, and Hx of unprotected sexual intercourse for 6

Seizure have multiple unprotected sex


years.
WBC 1.4
What is the next step?

- HIV serology
A- HIV serology
B- CSF analysis for syphilis
C - CSF analysis for toxoplasma

- syphilus D- Herpes pcr

Answer:A

Old man came complaining that (he can’t remember things), he sometimes forgets his
friends’ names or celebrities in his community and phone numbers. His wife is worried
he has Alzheimer’s.
Labs normal. He has tender knees (not something amazing just cause he’s old). No
mention how it affects his daily life.
- Alzheimer’s
- Benign forgetfulness

A baby diagnosed with Cystic fibrosis. He has positive sweat chloride test his brother
is normal. To confirm diagnosis of cystic fibrosis?
A. CFTR gene in parent
B. CFTR in sibling
C. Chloride test in parent
D. Chloride test in sibling

Patient diagnosed with thalassemia major and you decide to start regular blood
transfusion. Which of the
following is the most important vaccine?

A. Hepatitis A,
B. Hepatitis B,
C. DTP,
D. MMR,
E. Influenza.

child came with pallor and jaundice. His father and grandfather are both known to have
chronic hemolysis and
underwent gallbladder removal. What is the best test for diagnosis?

A. Hb electrophoresis,
B. Sickle cell test,
C. Coombs test,
D. Osmotic fragility test.

40s old female, with heavy bleeding came to ER, what to do?
A- IUD
B- D/C
C- Hysterectomy
D- Mefenamic acid

*missing more info

A patient with liver cirrhosis presents with worsening deterioration in consciousness for
some of days, examination revealed Tense ascites.
Albumin 30
Bilirubin 30
INR 1.9
What class of the Child Pugh?

A. Class A
B. Class B
C. Class C
D. No classification
← telegram note ?

Young female (not pregnant) with hx of PE 1 year ago , presented with swollen leg, Us
showed proximal DVT, most appropriate management?
- long term anticoagulant therapy
-anticoagulant for 2 weeks
-thrombolysis
- IVC

Patient with epilepsy, most common shoulder dislocation:


- inferior
- supracromine posterior
- posterior

Pt did let neck surgery then develop numbness (or loss of sensation) in the lower part of
pinna + left triangle of mandible. Which nerve is injured?
A. Third occipital
B. Great auricular
C. Great occipital
D. Lesser occipital

what could be cured after splenectomy?


A-ITP
B-TTP
C-Thalassemia .. Splenectomy ‫ *ٮﻌﺪ ﻋﻤﻠ(ٮﺔ‬CURED ‫ى‬J‫ڡول اٮ(ﺶ اﻟﴚ ال‬L(‫وﻟﻤﺎ ٮ‬
- Idiopathic thrombocytopenic purpura.
- Hereditary spherocytosis.

. ‫ڡﻀﻞ‬U‫ أ‬ITP ‫ وﻟﻮ *ﺣﺎب اﻻﺛﻨ(ٮﻦ‬.. ‫ ﺻﺤ(ٮﺤﺔ‬Thalassemia ‫ وﻻ‬Sickle ‫ﻻ‬

^
‫ڡﺮى‬L‫ڡروب ام اﻟ‬L
Infant Mortality Rate has decreased to 4.28 in 2020 in Saudi Arabia.
Which of the following figures involved in This:

A) Number of women in childbirths age


B) Number of deaths in saudi arabia in 2020
Telegram answers C
C) children who died in 2020 and not completed their first year

8 YO female presents to the clinic after finding of hilar lymphadenopathy on


CXR. She has on and off cough, but denies any fever, headache, n/v, weight loss
all negative. Labs insignificant except for hypercalcemia X-ray shows confirmed
bilateral hilar lymphadenopathy CT guided biopsy shows noncaseating granuloma
What is the best next step in management
A-Observe A •

B-Start prednisolone
C-Start azithromycin
D-Start anti TB medication

2.5 y/o boy keeps yelling NO (and some other like this) what will you advise the parents?
A- Ignorance
B- Counseling
D- positive reinforcement

Appendix perforation antibiotic coverage


Positive aerobes
Positive anerobes
Negative aerobes
Negative anerobes and aerobes

⁃ Hepatitis A vaccine after blood transfusion


As sechduale • A
After 3 m
- child K/C of SCD just got blood transfusion after a
crisis 2 weeks ago when should take vaccine for
Hepatitis A ?
A. As it scheduled
B. After 6 months
C. After 6 weeks
Infant Mortality Rate has decreased to 4.28 in 2020 in Saudi Arabia.
Which of the following figures involved in This:

A) Number of women in childbirths age


B) Number of deaths in saudi arabia in 2020
C) children who died in 2020 and not completed their first year

8 YO female presents to the clinic after finding of hilar lymphadenopathy on


CXR. She has on and off cough, but denies any fever, headache, n/v, weight loss
all negative. Labs insignificant except for hypercalcemia X-ray shows confirmed
bilateral hilar lymphadenopathy CT guided biopsy shows noncaseating granuloma
What is the best next step in management
A-Observe
B-Start prednisolone
C-Start azithromycin
D-Start anti TB medication

2.5 y/o boy keeps yelling NO (and some other like this) what will you advise the parents?
A- Ignorance
B- Counseling
D- positive reinforcement

Appendix perforation antibiotic coverage


Positive aerobes
Positive anerobes
Negative aerobes
Negative anerobes and aerobes

⁃ Hepatitis A vaccine after blood transfusion


As sechduale
After 3 m
• A
Infant Mortality Rate has decreased to 4.28 in 2020 in Saudi Arabia.
Which of the following figures involved in This:

A) Number of women in childbirths age


B) Number of deaths in saudi arabia in 2020
C) children who died in 2020 and not completed their first year

8 YO female presents to the clinic after finding of hilar lymphadenopathy on


CXR. She has on and off cough, but denies any fever, headache, n/v, weight loss
all negative. Labs insignificant except for hypercalcemia X-ray shows confirmed
bilateral hilar lymphadenopathy CT guided biopsy shows noncaseating granuloma
What is the best next step in management
A-Observe
B-Start prednisolone
C-Start azithromycin
D-Start anti TB medication

B ‫ى‬will
2.5 y/o boy keeps yelling NO (and some other like this) what ‫ام القر‬you advise the parents?
A- Ignorance
B- Counseling
D- positive reinforcement

Appendix perforation antibiotic coverage


Positive aerobes
Positive anerobes B ‫ام القرى‬
Negative aerobes
Negative anerobes and aerobes

⁃ Hepatitis A vaccine after blood transfusion


As sechduale
After 3 m
Infant Mortality Rate has decreased to 4.28 in 2020 in Saudi Arabia.
Which of the following figures involved in This:

A) Number of women in childbirths age


B) Number of deaths in saudi arabia in 2020
C) children who died in 2020 and not completed their first year

8 YO female presents to the clinic after finding of hilar lymphadenopathy on


CXR. She has on and off cough, but denies any fever, headache, n/v, weight loss
all negative. Labs insignificant except for hypercalcemia X-ray shows confirmed
bilateral hilar lymphadenopathy CT guided biopsy shows noncaseating granuloma
What is the best next step in management
A-Observe
B-Start prednisolone
C-Start azithromycin
D-Start anti TB medication

2.5 y/o boy keeps yelling NO (and some other like this) what will you advise the parents?
A- Ignorance
B- Counseling
D- positive reinforcement

Appendix perforation antibiotic coverage


Positive aerobes
Positive anerobes
Negative aerobes
Negative anerobes and aerobes

⁃ Hepatitis A vaccine after blood transfusion


As sechduale
After 3 m

36- which of the following indicate severe asthma.


Infant Mortality Rate Baseline
has decreased
PEFR 400ml
to 4.28 in 2020 in Saudi Arabia.
Which of the following figures involved in This:
A-PEFR less than 300
B- RR>25 Earth -

C-HR more than 100

A) Number of women in childbirths age


B) Number of deaths in saudi arabia in 2020
C) children who died in 2020 and not completed their first year

8 YO female presents to the clinic after finding of hilar lymphadenopathy on


CXR. She has on and off cough, but denies any fever, headache, n/v, weight loss
all negative. Labs insignificant except for hypercalcemia X-ray shows confirmed
bilateral hilar lymphadenopathy CT guided biopsy shows noncaseating granuloma
What is the best next step in management
A-Observe
B-Start prednisolone
C-Start azithromycin
D-Start anti TB medication

2.5 y/o boy keeps yelling NO (and some other like this) what will you advise the parents?
A- Ignorance
B- Counseling
D- positive reinforcement

Appendix perforation antibiotic coverage


Positive aerobes
Positive anerobes
Negative aerobes
Negative anerobes and aerobes

⁃ Hepatitis A vaccine after blood transfusion


As sechduale
After 3 m
Infant Mortality Rate has decreased to 4.28 in 2020 in Saudi Arabia.
Which of the following figures involved in This:

A) Number of women in childbirths age


B) Number of deaths in saudi arabia in 2020
C) children who died in 2020 and not completed their first year

8 YO female presents to the clinic after finding of hilar lymphadenopathy on


CXR. She has on and off cough, but denies any fever, headache, n/v, weight loss
all negative. Labs insignificant except for hypercalcemia X-ray shows confirmed
bilateral hilar lymphadenopathy CT guided biopsy shows noncaseating granuloma
What is the best next step in management
A-Observe
B-Start prednisolone
C-Start azithromycin
D-Start anti TB medication

2.5 y/o boy keeps yelling NO (and some other like this) what will you advise the parents?
A- Ignorance
B- Counseling
D- positive reinforcement

Appendix perforation antibiotic coverage


Positive aerobes
Positive anerobes
Negative aerobes
Negative anerobes and aerobes

⁃ Hepatitis A vaccine after blood transfusion


As sechduale
After 3 m
A new screening test for diabetes has a sensitivity of 90% and specificity of 80%

Which of the following is the best interpretation?


A. 80% of non-diabetics had positive result
B. 80% of diabetics had positive result
C. The test was positive for 90% of diabetic patients
D. For each 100 positive test results, 90% were diabetics

Perinatal mortality:

a) Includes all stillbirth after the 20th week of pregnancy


According to NCHS and
b) Includes all neonatal deaths in the 1st 8 week of life WHO definition , UTD
c) Includes all stillbirths & 1stweek neonatal deaths
d) Specifically neonatal Deaths.
e) Is usually death per 10,000 live births

Rickets supplement?
Vit D1 orally D3 or D2 according to
Vit D2 orally UTD

Vit D3 orally Cytic fibrosis, AMBOSS

Vit D4 orally

Which of the following is not associated with cystic fibrosis?


A. Sinusitis
B. Lung cancer
C. Pancreatic insufficiency
D. Small bowel obstruction

Which of the following tests is used to establish a diagnosis of acromegaly ?

A. Liver function tests (LFTs)


B. Complete blood count (CBC)
C. Oral glucose tolerance test (OGTT)
D. Hemoglobin A1C (HbA1c)
E. Thyroid function tests (TFTs)
153) child with bilateral knee swelling, (they gave the diagnosis as juvenile idiopathic
arthritis) asking about which type?

A. Systemic
B. Poly
C. Mono

154) 17 yr old male presented with severe asthma exacerbation on albuterol and long
acting glucocorticoid, hx of admission 2 yrs ago

On exam: resp distress and diffuse expiratory wheezing

He Received systemic glucocorticoid and b2 agonist inhaler but no


improvemnt.. Cxr : hyper inflation

Ph 7.3

What next;

A-, admission to icu

B-.discharge and reassurance

C-disscharge and follow up next day

D- Admit to medical word

155) 50 yr old , with IHD and dm

Admitted to icu with severe pneumonia and was treated with abx ..

After 3 days of admission, he developed hypotension and treated with hydration and
inotrope

,On admission lab was normal

After 3 days, LFT was abnormal

Total bil 20(increased)

Very high ast and alt( 1000)

Mild
increase
in LDH Us
:
unremark
able What
is dx:

A- ischemic hepatitis
36 old male at ER C/O Right abdominal Pain , O/E : fever, anorexia ,
weight loss , tenderness in RQ and Lower intercostal margines also
patient is toxic Temp. 37.9 ( I think but it was elevated ) wbc high,
bilirubin high US : cystic lesion without septates CT : homogenous (not sure)
and “THICK WALL with Peripheral enhancement what’s most appropriate
Mx :

A. Early laparoscopic cholecystectomy


B. Emergent stent chole
C. Cholecystectomy after 3 months

24 years old male, medically and surgically free , presented with manifestation
of
intestinal obstruction, suspected to be small bowl obstruction
What is the most common cause
‫د‪ .‬عبدهللا الغانم‬
‫د‪ .‬محمد المقداد القحطاني‬

‫‪April 2022‬‬

‫تجميعات شهر ابريل‬


‫الخطأ وارد‬

‫اللهم إني أستودعك ما قرأت وما حفظت وما تعلمت‪ ،‬فرده عند حاجتي إليه‪ ،‬إنك على كل شيء قدير‬
AMBOSS

Ventricular tachycardia

Pt has asthma attacks after playing sports. He got relieved after taking
montelukast and albuterol.

What is best Maintenance therapy for his asthma:

A) Oral steroid and saba as needed.

B) Laba twice daily

C) ICS twice daily and saba as needed.

D) Montelukas and Albuterol as needed.

Male with family history of HTN. Creatinine high glucose normal. What to give?
ACEI
BB
Furosemide
(No CCB)

Patient has lesion ascciiated with diarrhea and wl with stricutre in iluem and
jujunm this condion is associated with
A erthema margenatum in chron
B peranal disease
Pt known he asthmatic he is on SABA only came to ER with sob and now dx by
(persistent moderate asthma) what you will add on his medication

A. LABA
B. ICS
C. ICS and LABA

Case of COPD , absent gag reflex (nothing about consciousness ) on sever


respiratory distress Ph:7.25 High pCO2 and low O, PO sat: 83, What is your next
best step:

A. Intubation and ventilation


B. Non invasive
C. High flow nasal cannula

patient immune compromised what to avoid?

A. MMR or vercilla
B.Pertussis
C. PV
D. Influenza
2. 7 years female with eyelid erythema and thickening of skin over
metacarpophalangeal joints + proximal muscle weakness??

A- Juvenile Dermatomyositis
B- Scleroderma
C- SLE
Upper eyelids (heliotrope rash)
Mechanic's hands: thickened and cracked skin on the sides of the fingers and palms
First-line test: anti-Mi-2 antibodies
Muscle biopsy: Gold standard for diagnosis of IIM
• First-line: glucocorticoids
o Prednisone
o In severe disease or with multisystem involvement, consider an initial
short course of pulsed IV methylprednisolone.
• AND (usually) a steroid-sparing immunosuppressive agent, e.g.: [16]
o Methotrexate
o Azathioprine

Most common organism of bronchiolitis


A) RSV : Most common: respiratory syncytial virus (RSV)
B) parainfluenza virus
C) staph areus
Dx clilncal Nasopharyngeal aspirate test for RSV
Chest x-ray Bronchodilators, epinephrine, and corticosteroids

Patient with 2 stage chronic renal disease presented with fatigue and pallor His
lab was: Hgb Low MCV Low What is the most appropriate test you will order for
him?

A. Iron study
B. Erythropoietin level
C. Hgb electrophoresis
. OTHER Q BANK

18- World health organization (WHO) to determine the health of the population in
KSA What should KSA share to WHO:
A- Health determinant
B- Health indicator
C- Risk factors
D- Something variable

chancroid , tb , ground glads apperancr, ctg with variable PICTURES

Risk factor of suicidal?

) Depressed female. What is the most important risk factor of suicidal thoughts?
A- Age
B- Gender
C- Social isolation
D- Previous attempt.
Answer: D then C.
Answer is: B then C

9) Highest risk factors for suicide?


A- Absence of medical illness
B- Social isolation C- Age
D- Femal

Pictur of IDA

Faciprium malaria
Rooting reflux
SVT
ABSENT RED REFLEX
ENDOMETRIOSE
BASAL CELL CA
ANEMBREONIC SAC
Inferior mi
Skin tag
Malaria
Chloasma

‫اﻟﺼ ﻮر‬
Osteoarthritis.
Chancroid.
Failure ro thrive(diagnosis and treatment of celiac).
HSP
Vesicouretric reflex
Rectal prolapse
P falciparum pic

%Preschool vaccination
%riding tricycle.. milestone
%Rubella picture in a child
%A case of infective endocarditis.. causative organism
%A case of scarlet fever .. causative organism
%risk factor for BPH
%A case that develop UTI after intercourse (A.treat if symptomatic B. give 2
weeks Ab course C. Post coital Abs D. 6 month Ab course
%A case of Heparin induced thrombocytopenia what to do?
%gout on microscope(needle crystals with -ve birefringent
%physician who change in research results (falsification)

&Drugs = Antidote '

1⃣_ Acetaminophen (paracetamol)--➡' N-


Acetyl Cystine (NAC)

2⃣Beta Blockers➡ Glucagon

3⃣Opioids➡ Naloxone

3⃣Anticholinergic➡ Physostigmine

4⃣ Benzodiazipines--➡Flumazenil

5⃣ Warfarin--' ➡FFP (early) or Vit


K (late)

6⃣ Heparin ➡Protamine sulfate

7⃣ Insulin--➡ Glucagon

8⃣ Iron---➡ Desferoxamine

9⃣Digoxin ➡Digoxine immune Fab

)CCB ➡Calcium chloride

1⃣1⃣Mg+2 Sulfate ➡Calcium gluconate

1⃣2⃣ASPRIN➡sodium bicarbonate

Skin tag
Hemothorax
Heart block
Basal cell carcinoma

Pt's friend ask you about diagnosis of Pt's disease. You refused . What is
ethical concept?
Case of patient after ct with contract develop bleeding from nose mouth wound
site I think because it manifested as
A dic “bleeding” and you know DIC is basically
B anaphylactic contrast reaction bleeding, and clots randomly.
If it was anaphylactic shock, it’s wouldn’t
manifest as bleeding anywhere.

Comatose pt 5day wayof feeding???NG tube for short term


2gastertomy for long more than 2 or 3 weeks AMBOSS

Pt under went emergency dialysis??


Arteriovenous fistula1...
Arterialcola UTD
For emergency the answer is central venous catheter

Pt in ICU received 15 units of blood, now blood coming out from NGT, incision,
and cannula site UTD
1. Transfusion reaction
2. Thrombocytopenia
3. Hypocalcemia
4. Forgot

Question about DM incidents and prevalence after insulin

Decrease prevalence
Increase prevalence
Decrease incidence
Increase incidence

Role of acyclovir in pregnancy


A decrease viral sheeding and infection duration
B decrease placental transmission
C decrease infection severity

man has HTN and his family father and 2 sister have HTN, medically free before,
O/E and invesX all with normal limits, his bmi 27 and his blood pressure is
160/100, high creatinine
And high blood urea nitrogen level.
what is the appropriate management?
AMBOSS

- ACEI
- alpha blocker
-BB
-CCB

man had recurrent episodes of arthritis in his big toe, he diagnosed with gout
and he is k/c of psoriasis. He is compliant to gout medications but with no
improvement and high uric acid level. Now he presented bilateral ankle arthritis,
the diagnosis is?
AMBOSS
A. Pseudogout arthritis
B. Gout arthritis
C. Psoriasisarthritis

Hx of psychological disorders
e past Suicidal attempts
Social isolation
Use of alcohol or drugs

Patient diagnosed with malignancy What is the point of a multidisciplinary team?

a. Compassion
b. Patient treatment optimization.
c. Judgement-free discussions

which GFR metformin is harmful for diabetic pt ?

A. 30-40 AMBOSS
B. <30
C. <15

Patient K/C of SLE, which of the flowing type of anemia associated with SLE?
Hemoglobin electrophoresis: normal, Ferritin: High, Reticulocyte: High, Lactate
dehydrogenase: high, Haptoglobin: low?
UTD
A. Anemia of chronic disease
AMBOSS

B. Hemolytic anemia
C. Iron deficiency
D. Thalassemia

Which of these indicate severe asthma?

A. PEF <250
B. RR >20
C. HR 100/min
D. O2sat 95%

patient has midsystolic murmur grade 2/6 best heard at right lower sternal
border, and has S4 NO S3 best heard at the apex. What is the most likely
diagnosis? AMBOSS

A. Aortic stenosis
B. Mitral regurgitation
C. Aortic regurgitation
D. Mitral stenosis

30s female complaining of SOB arthralgia dyspnea fever she had dental
procedure 2 weeks ago and another procedure 2 months ago on PE there is
heart murmur radiation to axilla.UA showed hematuria and proteinuria what is
the diagnosis?

A. Infective endocarditis AMBOSS


B. Post streptococcus Glomerulonephritis
C. SLE

SLE Patient on hydroxychloroquine and Mycophenolate mofetil, what is the best


non-pharmacological management?AMBOSS

A. Smoking cessation
B. Weight reduction

Elderly pt moved to care center before 4 months, now he c/o loss of wt &
appetite, low mood and loss of interest for 4 w, also complain of loss of short
memory with attacks of low mood, what is the Dx?

A. Alzehaimer
B. Depression
C. Parkinsonism

Patient with PAD has 100-meter claudication's, DM, heavy smoker, not getting
better , how to improve his walking distance?
A. Supervised exercise program
B. Strict glycemic control
C. Smoking cessation

65-year-old Female Patient with AF, hx of Dm, HTN, TIA, calculate her
CHA2DS2-VASc score?

A. 3
B. 4
C. 5
D. 6

Elderly Patient with uncontrolled HTN, he is on ARBs, CCB, thiazide?

A. Add ACEI
B. Add BB Second-line medication (AMBOSS)
C. Add spironolactone
AMBOSS
APRIL 2022

2 y/o pt on chemo for ALL, contacted a chicken pox pt, what to do?

A. Acyclovir 7days
B. Give vaccine now
C. Stop chemo
D. Give varicella zoster Immunoglobulin Ref:UTD

Man with 3 months hx of epigastric pain, With loss of weight and appetite , most
investigations?
A. Endoscopy
B. CT
Ref:UTD

Pregnant with warts in vulva ttt?


A. Cryotherapy
B. electrophoresis

Patient with epilepsy, most common shoulder dislocation:


A- inferior
B- supraacromine posterior

pt on antipsychotic medication, the greatest complication to happen:

A. Akathisia
B. Sedation ??
C. d
9 y male with history of headache for 5 days and neck stiffness for two days With low
grade fever for one month CSF analysis Turbid Glucose low WBCs neut: 27 Lymph: 87
Protein high Diagnosis:

A. bacterial meningitis
B. viral meningoencephalitis
C. TB meningitis
D. fungal meningitis

9 y male with history of headache for 5 days and neck stiffness for two days With low
grade fever for one month CSF analysis Turbid Glucose low WBCs neut: 27 Lymph: 87
Protein high Diagnosis:

A. bacterial meningitis
B. viral meningoencephalitis
C. TB meningitis
D. fungal meningitis

patient had polyps excised from his sigmoid colon and on histopathological report there
was cancerous cells with free tissue margins, Next step:

A. Follow up
B. Sigmoidoscopy
C. Sigmoidectomy

14 y.o female with epigastric abdominal pain since a month Patient was complaining that
it is affecting her life, On exam patient had multiple bruises in abdomen when asked
about itShe said it appeared after she fell over her cycle 18 days back, next inv?

A. CT abdominal
B. Abdominal US
D. laparotomy
Patient post RTA with massive bleeding in the nose and mouth with leg fracture,
decrease bp, Pt can take his breath but he is afraid from what happened. What is the next
step?

A. Intubate with stretching cervix


B. Packing nasal bleeding
C. IV fluid
??

Patient present to ER after RTA, with SOB, in examination the tracheal shifted to the left
side, in chest X-ray, the lungs are expanded and winded mediastinum. What is the
diagnosis.
A. Tension pneumothorax
B. Cardiac tamponade
C. Plural effusion
D. Raptured esophagea

Which of the following indicate the severity of asthma?

A. PEF less than 300 If less Than 250


B. RR more than 55
C. PR 110

A 60-year-old man is admitted to the Coronary Care Unit with an acute myocardial
Infraction. His hemodynamic parameters 2 hours later are:
Blood pressure :80/50 mmHg
Heart rate 40 /min
Oxygen saturation °C 95% on room air Which of the following would be the appropriate
management?
A. IV0.6 mg atropine sulphate
B. Normal saline infusion
C. IV isoproterenol
D. IV dobutamine

??

Harbi
Patient with intermenstrual bleeding. What is the most appropriate investigation
A) CBC
B) TFT RefDrWaffa
C) B-HCG
D)US

which vaccine can be given in immune deficient?


A. Dtap
B. MMR
C. Polio

35y.o man presented with fever, rigors & weight loss for 10 days. He underwent Mitral
valve replacement by prosthetic valve 1 month ago, Echo showed a small vegetation.
Vitals: Blood pressure 90/70 mmHg, HR 100/min, Temp 38.7°C. Caustive organisim is ?

A. Coxiella burnetli
B. Staphylococcus aureus ? Med scape
C. Streptococcus viridans
D. Staphylococcus epidermidis

Patient presented with sweating dyspnea, palpation and headache. She is a known case of
HTN and despite taking medication it’s not controlled. imaging reveals supra renal mass.
Which of the following meds u will give to control her HTN?

A. CCB
B. Alpha blocker
C. ACEI Ref: ‫علي‬
D. BB ‫بالحارث‬
Patient RTA with head trauma, increased urine output, decrease in Urine osmolarity
increased blood osmolarity?

A. Central diabetes insipidus Ref: ‫بالحارث‬


B. Cushing
C. Conn’s
D. Nephrogenic diabetes insipidus

Septic shock case indicates adequate systemic perfusion?


A-Cardiac index Ref: UTD
B- Mixed venous oxygen saturation (SMVO2)
X
C-Central venous oxygen saturation (SCVO2) or lactate

Patient 56 y.o came for routine follow up, known case of CHF asymptotic not on
medication his EF 25% what you want to give him?

A. Beta Blocker
B. ACEI Ref: amboss ‫بالحارث‬
C. Spironolactone
D. Diuretics

DM, heart failure and admitted for hysterectomy and received Normal Saline cause poor
oral intake, 4 d nurse noticed decreases in Oxygen Sat. & SOB, o/e: crackles, how to
prevent this complication?
Overload is prevented by daily
A. cardiopulmonary consult Ref: ‫بالحارث‬ fluids assessment
B. daily fluid assessment
C. Order daily lasix ‫الحربي‬

Patient k/c of COPD alert, has moderate respiratory distress.SO2 93%, ABG showed
hypoxia, hypercapnia and acidosis. What is next step?

A. noninvasive ventilation
B. Decrease oxygen Ref: Amboss and utd
C. Increase oxygen
D. Mechanical ventilation
Typical case of hyperthyroidism and palpation. What will you do for her palpation?
A. PTU
B. Methimazole Easy Q
C. Propranolol

Patient with a fib and dilated cardiomyopathy, what to give to control the rate now?
A. Digoxin
B. BB
C. Diltiazem

Patient with TIA symptoms and Afib, what to give?


A. Aspirin
B. Warfarin INR goal 2-3 ?
C. Warfarin INR goal 3-4
D. Enoxaparin

A Patient had lower parathyroidectomy, came to the clinic with bone pain, numbness
around the lips. Lab results showed high Ca + high PTH. What’s the most likely dx?
A. Missed adenoma
B. New adenoma
C. Parathyroid hyperplasia

Patient with raised JVP, Hypotension, unclear heart sound, clear lung sounds. How to
confirm the Diagnosis?
A. ECHO
B. ECG Easy Q, Beck’s triad
C. Chest X-ray
D. PFT

43-year-old male with hypertension on -sartan & amlodipine, his blood pressure is not
controlled. What to add?

A. Lisinopril
B. Atenolol
C. Indapamide
D. Zosin (alpha blocker)

Patient with HTN on 25 mg once daily hydrochlorothiazide. BP not controlled.


Management?
A. Double thiazide dose
B. Add ACEI ? Why not A
C. Add beta blocker

How to know fetal weight intrapartum in 37w?


— Femur length
— head circumflex
— Biparietal diameter
— Abdominal circumference

elderly Patient with uncontrolled HTN, he is on ARBs, CCB, thiazide?


A. Add ACEI
B. Add BB ?
C. Add spironolactone

When does bedwetting becomes alarming?


A. 5 years
B. 6 years UTD
C. 7 years
D. 8 years

Thyroid cancer with increased calcitonin:

A. Medullary thyroid cancer


Easy q
B. Follicularthyroidcancer
Female with previous 2 preterm labour, now she is in 20 weeks of gestation and her
cervix opened 30 mm, what you will do?

A- Immediate cervical cerclage


B- Give tocolytic & wait
C- Strict bed rest
D- progesterone supplement

Normal vaginal delivery, Baby weight 4.2kg, Laceration which degree:

A-First
B-Second
C-Third
D-Forth

41YO P5 +3 presented to the clinic complaining of abnormal uterine bleeding her


Menstrual period is regular, associated with blood clots and pain that is not relieved by
analgesic she had previous myomectomy, she is a known case of PCOS&her BMI is 40?

A-Adenomyosis
B-Endometriosis
C-Uterine fibroid
D-Endometrial hyperplasia

A 39-week pregnant patient, history of caesarean section, due to breech presentations,


now she is in labor, with regular contractions 4 every 5 min, cervix fully dilated, full
effacement, station +3, What is your management?

A. Ventouse delivery
B. caesarian section
C. examine her after 2 hours

Patient with chronic limb ischemia, presented with sudden leg pain, diminished popliteal
and distal pulses in right leg, and diminished distal pulse with intact popliteal in the left,
what's the appropriate next action?

A. Heparin
B. CT angio
C. US
D. conventional angio

Elderly Patient with small lung nodule (<8 mm), he is asymptomatic, not smoker, no
family hx of cancer, no constitutional, your next step?

A. f/u with CT scan


B. Biopsy
C. Chemotherapy

65-year-old Female Patient with AF, hx of Dm, HTN, TIA, calculate her CHA2DS2-
VASc score?

A. 3
B. 4
C. 5
D. 6

* 65=1
Female=1
DM=1
HTN=1
TIA=2
Score =6

Patient with PAD has 100-meter claudication's, DM, heavy smoker, not getting better.
How to improve his walking distance?
A. Supervised exercise program
B. Strict glycemic control
C. Smoking cessation

*might be C

Elderly pt moved to care center before 4 months, now he c/o loss of wt & appetite, low
mood and loss of interest for 4 w, also complain of loss of short memory with attacks of
low mood, what is the Dx?
A. Alzehaimer Ref: psyh-smle
B. Depression
C. Parkinsonism

SLE Patient on hydroxychloroquine and Mycophenolate mofetil, what is the best non-
pharmacological management?
A. Smoking cessation
B. Weight reduction

30s female complaining of SOB arthralgia dyspnea fever she had dental procedure 2
weeks ago and another procedure 2 months ago on PE there is heart murmur radiation to
axilla.UA showed hematuria and proteinuria what is the diagnosis?
A. Infective endocarditis
B. Post streptococcus Glomerulonephritis ?
C. SLE

patient has midsystolic murmur grade 2/6 best heard at right lower sternal border, and has
S4 NO S3 best heard at the apex. What is the most likely diagnosis?
A. Aortic stenosis
B. Mitral regurgitation
C. Aortic regurgitation Easy q
D. Mitral stenosis

which of these indicate severe asthma?


A. PEF <250
B. RR >20
C. HR 100/min Already answered
D. O2sat 95%

* >30 breaths/min; heart rate >120 beats/min;


Patient K/C of SLE, which of the flowing type of anemia associated with SLE?
Hemoglobin electrophoresis: normal, Ferritin: High, Reticulocyte: High, Lactate
dehydrogenase: high, Haptoglobin: low?

A. Anemia of chronic disease


B. Hemolytic anemia
C. Iron deficiency
D. Thalassemia

* Reticulocyte high so hemolytic anemia

which GFR metformin is harmful for diabetic pt ?


A. 30-40
B. <30
C. <15 UTD

At what age baby talk 6-10 word and recognize two body part
A-17 month
B-19month
C-24 month Safadr says 18 months
D-12month ?

Child 4 years old with asymptomatic umbilical hernia what's the treatment?
A. Reassurance
B. wait until 5 years
C. Surgical repair
r Amboss and UTD

*not sure , B or C.. I would go for c

A female brought her baby to the well baby clinic ,he is still on formula /breast fed the
doctor advised her to give the baby iron supplement after the appointment
How old is the infant now?

A- 1 month
B- 2 months
C- 3 months Ref: cdc, UTD
D- 6 months

4 years. old girl with decrease head growth, decrease social interaction, decrease in
language , what is the closest ddx?

— Autisim
— mental retardation
— Rett's disorder
— aSPERGER syndrome

403- Patient “45 years old, came with history reducible hernia in inguinal area it pop out
every two
days”interval” with mild pain and and he Manuel reducible it by his finger, then he
suddenly
developed severe “constant ” pain in early morning in that area and came to hospital, on
examination
of the inguinal area and scrotum examination and he is free from both them and pain is
disappear
Now just with persistent nausea and vomiting next step ?
A- Pelvic and Abdomen CT
B-hernia repair today after 2 hour i think ?
C reasurance and discharger

Hx of psychological disorders
past Suicidal attempts
Social isolation Bad recall
Use of alcohol or drugs

emale with Pelvic pain increases with menses. On examination: uterus normal And there
is uterosacral nodularity and tenderness dx?

A. Endometriosis Amboss
B. Adenomyosis
C. PID
D. Fibroids
56 y/o woman presents to the clinic with a non-healing ulcer over her right lateral
malleolus, she is hypertensive. pulse is normal and her local exam shows dark
discoloration of the skin around the ulcer & a viable ulcer bed. best next step is ?

A. CT angio
B. Venous duplex US ?
C. Arterial doppler US
D. Conventional angio

man had recurrent episodes of arthritis in his big toe, he diagnosed with gout and he is k/c
of psoriasis. He is compliant to gout medications but with no improvement and high uric
acid level. Now he presented bilateral ankle arthritis, the diagnosis is?
A. Pseudogout arthritis
B. Gout arthritis
C. Psoriasisarthritis
r
Which of the following is the highest risk factor for cerebral palsy ?
• A-Hypoxia
• B-Preterm
• C-Gestational diabetes Amboss
• D-Neonatal sepsis

Case acute RUQ pain,jaundice,US thickened GB and bladder stones what to do


-Laparoscopic cholecystectomy
-ERCP
-I don’t remember may be IV Fluid Bad recall ?? Can’t judge

43 years female complaining of rectal bleeding no past medical or surgical history


diagnosed as grad 3 hemorrhoids what is your next plan in management?
A/ colonoscopy
B/ hemorrhoidectomy other sources
C/ sclerotherapy and ligation
Anboss
37 years old male complaining of sever left loin pain radiated to groin and red urine he is
irritable and try to found suitable positions to reduce the pain what is diagnosis?
A/ ureteric stone
B/ pyelonephritis
C/ glomerulonephritis

12 gestation with fundal hight 19 wks and bhcg270000 ( very high) most likely dx:
‫ جا سؤال مشابه له بس بسيناريو كامل و فيه‬bhcg ‫ مره عالي والـ‬fundal high ‫و يسأل نوع الحمل‬
‫ عالي‬fundal height

Ectopic pregnancy
Molar pregnancy

40 years old female, P4 with a history of tubal ligation 4 years ago after her last CS
delivery. Her last Menstrual period was 6weeks ago.
Presented to ER with vaginal spotting. NO abdominal pain
O/E: OS is closed. Tenderness and fullness in her abdomen in RLQ. What is the best
Initial investigation?
A- Pregnancy test
B- US
C-CT Abdomen

Patient at 29 weeks, didn't feel fetal movement for 1 day, CTG was reactive, Biophysical
profile was 8. What to do next?
A. Steroid and repeat Biophysical profile after 24 hours
B. Repeat Biophysical profile at 1 week
C. IOL
D. Urgent CS

Pregnant, twins one cephalic and another is breech presentation, how to deliver?
A- Cesarean section
B- Normal delivery
Woman in delivery bleeding not stop, she wants to conceive in the future, which structure
you should ligate?
A. Uterosacral ligament
B. Externaliliacartery
C. Internaliliacartery
D. Uterinevein

Which is most contraindication in pregnancy ?


- thyroidectomy
- radioactive iodine
- beta blocker

ileal resection cause any vitamin deficiency


*bile
B12
Iron

Pregnant came to the ER today with moderate bleeding. What's the inext step? (Not
mentioned if there’s Hx of placenta previa)

A- transfer to Us
B- insert 2 large IV cannula Bad recall
C- Emergency delivery
C- pelvic examination

*B then A

New born after ventose delivery swelling limited to suture

A. Cephalohematoma
B. Carbuncle
1 yrs old woman presents with postcoital bleeding her cycles are becoming more heavy
and irrigulae than usual, she has type 2 diabetes and BMI 38 us showed endometrial
thickness 18mm
What is the most appropriate next step in her management?
A) pelvic ct
B) hystroscopy ?
C) clinical endometrial sample

Question about DM incidents and prevalence after insulin

Decrease prevalence
Increase prevalence ? Less people die from the disease
thus increased prevelance !
Decrease incidence
Increase incidence

Role of acyclovir in pregnancy


A decrease viral sheeding and infection duration
B decrease placental transmission
C decrease infection severity

man has HTN and his family father and 2 sister have HTN, medically free before, O/E
and invesX all with normal limits, his bmi 27 and his blood pressure is 160/100, high
creatinine
And high blood urea nitrogen level.
Ref:UTD however consider the possibility of
what is the appropriate management? pheochromocytoma!
- ACEI
- alpha blocker
-BB
-CCB

Urine culture in pregnancy ?


12-16 wk ✅
12 weeks for uti

Man came with scalp open wound, after 6h assault, what wound management? A-
secondary closure
B-Debridment with 1ry closure Utd
C-Debridment with granulation
D-Leave it for granulation!

Ectopic pregnancy
Bhcg
1st 1000
2nd 1200
3rd 800
What to do next ?
Decrease > 15% >>> should continue observation till 0

Child throws ball and draws straight lines what age?


• 9 months
• 15 months
• 18 months
Its 2 years
Child able to run, dress and undress himself, tell stories, tie his shoes:
A- 5 yrs
B- 4 yrs 6 is more accurate

*if there is 6 years I will choose it

Kid with cough coryza conjunctivitis macuopapular Rash that started on face then to
hand and feet?

A Gonoccocal bactremia
Ref: Amboss
B Rubella if No measles
C Toxic shock syndrome
D Rotair syndrome’s

*Coryza it’s for Measles


Pt in ICU received 15 units of blood, now blood coming out from NGT, incision, and
cannula site
1. Transfusion reaction If fever
2. Thrombocytopenia
3. Hypocalcemia
4. Forgot
Indication of surgery

Patient known case of peptic ulcer disease in medications but failed to respond, upper
endoscopy done and revealed multiple
ulcers in antrum. What is the best treatment?
A. Pyloroplasty and vagotomy
Antrectomy is the procedure of choice
however C is the closest one

B. Total gastrectomy
C. partial gastrectomy

Pt in ICU received 15 units of blood, now blood coming out from NGT, incision, and
cannula site
1. Transfusion reaction
E
?
2. Thrombocytopenia
3. Hypocalcemia
4. Forgot

117. The uterus is inverse after delivery of the placenta what is the site of insertion to be
like this shape?
A. Fundal
B. Anterior
C. Posterior
D. Lateral

Pedia come with her parent after seizure Hx of cough .nasal congestion flu like symptoms
temp38.5
Next step ??
Phyniton
Ampiciln
Paracetmol

Pt under went emergency dialysis??


Arteriovenous fistula1... central venous catheter (CVC) for emergency blood
Arterialcola access.
Avf needs to mature before use.

Comatose pt 5day way of feeding???


NG tube
2gastertomy

3day infant develop rash erythromatus on red base all inv..normal ???
1refer to dermatology
2.reasure and this is not a serious condition can resolve
3.skin biopsy

Lactiting women small around mass in areolar with small inverted of nipple and reteurn
normallyDx????
Fibro adenoma
Breast cyst
Duct Ectasia

(Duct ectasia, also known as mammary duct ectasia, is a benign (non-cancerous) breast
condition that occurs when a milk duct in the breast widens and its walls thicken. This can
cause the duct to become blocked and lead to fluid build-up. It's more common in women who
are getting close to menopause.

40 Male co abd nominal pain associated with fatty meal ...relive in 2 hours Dx??
Gastritis
Biliary colic
Pancerititis

35 years Male complain of epigastric pain increase when supine position and not good
taste ??
Peptic ulcer
Esophagitis
Gastritis

Child RtA weak periphral puls .BP 70/40


Next step??
OROPHARYNX TUBE
ENDOTRACHEAL
NASO GASTRIC

Long air travel Rt side diminished air entry???


Rt site tension pneumothorax
RT SITE HEAMOTHORAX

Child with joint pain and rash


Urine ..red cell Dx .. Bad recall
Sle
RA
Lupus nephritis

If this is the
Victim of rta ..Hr.50 correct q > 2
BB.60 /45 how tncrease BP???
1.normal saline
2.bloood negative o

Lady inferty amonnorhea for 1year .do curettage because up normal utrine bleeding what
cause of infert???
1.Asherman
2.shehan
3.pco

Case of patient after ct with contract develop bleeding from nose mouth wound site
A dic
B anaphylactic contrast reaction

Female 43 years want to do screening for cancer what best at her age
A Colonoscopy
B mammogram
Ref: USPSTF
C clonoscopy and mammogram

After d and c uterine perforation occur risk of placenta accreta


A increase
B decrease
C no change
D unknown

Role of acyclovir in pregnancy


A decrease viral sheeding and infection duration
B decrease placental transmission
C decrease infection sevirty

ediatrics patient with abnormal movement, tea coloured urine, odeama around eyelid,
HTN, low C3, mother mentioned that child has preceding sling infection

1-PSGN
2-HUS Ref: amboss
3-IgA

( 1 because presence of HTN and low C3)


post streptococcal glomerulonephritis

Mother concerned about her child how have abnormal face and mouth movements during
sleep
mid temporal spikes in ecg? Ref: UTD
Ronaldic seizure

Pelvic organ prolapses at the introitus


A-cystocele
B-enterocele
C-rectocele Amboss
D-paravaginal

Pt6 hrs post femoral artery repair, pt developed swelling and neurological signs
What to do next
Cta
Ct
Duplex ultrasound
Compartment pressure release
Pateint with immunodeficiency disease has peranal region mass with fluctuations and
erthema creptius management
A injection of antibiotics
B aspiration
C topical cream i cant remember its name

Patient fall from 4 floor with open wound in leg and necrotic dirty tissue and stable first
to do
A antibiotic
B surgical debridement
C immediate reduction Ortho consultant ?

Patient has lesion ascciiated with diarrhea and wl with stricutre in iluem and jujunm this
condion is associated with
A erthema margenatum Amboss
B peranal disease

What anti htn contraindication in preg??


Hydralazine
Labetalol
Captopril

6years loud s2 bluish skin.sob.???


1.aortic stenosis Ref: teachmepediatrics
2.tof
3.vsd

ventricular septal defect (VSD), pulmonary stenosis, a misplaced aorta and a thickened
right ventricular wall (right ventricular hypertrophy)
Child 6 month complain of drooling of saliva ... surgery done hiatal repair...
Most complication of this opration:::
1..GASTRIOSPHAGEAL REFLUX
2:PHERNIC NERVE INJURY
3;

Child with strep throat, has a brother 2 years old without symptoms. What to do?
Strep antigen
Observe
Antibiotics

1- Pregnant woman about to deliver, past history significant for Asthma and E.Coli. What
to give during delivery?
Betamethasone
?
Ampecilline

7 months old with fever, inspiratory crackles and respiratory crepitus. What does he
have?
Asthma Amboss
Bronchiolitis
Pneumonia

Male with family history of HTN. Creatinine high glucose normal. What to give?
ACEI
BB
Furosemide Previously answered
(No CCB)
Ventricular tachycardia

3 7 yo with Breast lump is hard, next? Ref: UTD


1-US
2- Mammogram

All breast masses should get mammogram exception is pregnancy and breastfeeding

16 years old boy come to the clinic by his family with jaundice and fatigue and
splenomegaly, and history of cholecystectomy before for stone, also History of
Required many blood transfusions before, Labs: microcytic anemia (+/- hemolysis)
Highest diagnostic test?
Amboss
A- Peripheral blood smear
B- Hemoglobin electrophoresis
C- sickle cell genetic test
D- Bone marrow biopsy
g scenarios about a child 2 years old
with SOB and Expiratory wheeze and other
respiratory symptoms he had a viral illness
2 days ago Amboss
A. Respiratory monitoring for hypoxia
B. Inhaled epinephrine and steroids
C. Intubation and antibiotics
D. SABA
Pt with hepatitis C and cirrhos

35 years old female her dad has colon cancer when he was 55 and her mom had breast
cancer when she was 43 asking about screening:
A- She should do mammogram annually ‫العبيدي‬

B- Start mammogram at 40
C- Start mammogram at 40 and colonoscopy at 55
D- Colonoscopy

30-40 y.o Adult male medically free presented to ER C/O several episodes of
hematemesis for the first time, takes no medication, no family history of similar attacks,
normal abdominal examination no guarding no tenderness.

Hgb : 9
Plt: 250
Alk: slightly decrease
All other test were normal

What is the Diagnosis: Q is missing something


A-Erosive gastritis
B-PUD
C-Mallory Weiss syndrome
D-Esophageal varices

(No Hx of cough moreover C will not cause severe drop in hgb like this case)
Case of patient complaints of abdominal pain, exam tenderness and. The pulsatile mass
above Umbilicus What is the most appropriate investigation?
A. US
B. CTA symptomatic
C. angiography

* just noticed B is CTA not CT scan So i would go for U/S

60yrs Pt known diabetic and HTN


On CCB+lisinopril and indipamide C/O un control HTN which add?
Investigation :normal RFTe
1.thizide diuretic
2.aldosterone anatagonist Need more details like K level
3. losartan
4. Metoprolol

4 month milestone:
A. Sit without support
B. Head control
C. Start trying to crawl
D. Follow objects ects
34 years female at 28 weeks with cough and difficulty in breathing , she appeared restless
and uncomfortable on auscultation of the lung shows bilateral rhnochi, the nail bed and
oral mucous membrane appear pale, she is allergic to dust, pollen, mold and animal hair.
No hx of smoking. She reports that the symptoms began 2 days perior and started cough
with clear color phlegm. All vital normal.
What it's the most appropriate initial test?
- xry Dr. alharbi and website Q bank ?
- Hgh
-Spirometry
-ABG

* She is pregnant I’d go with spirometry. If not pregnant Xray

$pt with HCV ,, developed abd pain, CT done and show HCC which is 6cm =
A. Radiotherapy
B. chemotherapy
C. excision

postmenopausal woman complaining of 1 year hx of recurrent vulvar itching associated


sometime with blood streak secretions , recently develop pea size mass in the labia?
a) bartholin cyst
b )cystic adenosis
c) bartholin gland cancer Utd
d) squamous cell cancer of vulva

Female 27 years old, she is asymptomatic, her last pap smear was 3 years ago and it
showed unconcerned squamous cells. What is the most appropriate thing to do?
A-Repeat pap with cytology
B-No need and reassure
Ref: Wafa
C-Colposcopy
D-Cervical swab
female patient came to the clinic complaining of a mass on a vagina she has a history of
repeated unprotected intercourse with multiple partners, upon examination. she has a wart
in
the vagina, the causative agent is:
A- Herpes simples
B- Neisseria Gonorrhoea Amboss
C- Treponemma pallidum
D- Molluscum contagiosum

Mother brought her 9yrs child for short stature Physical exam was normal GH
normal FSH normal TSH NORMAL T3-4normal Insulin like growth factor decreased
X ray: bone age of 6yrs Dx?

Familial short stature


GH deficincy
Constitutional growth delay

****Bone age more than 2 years , so it is GH deficiency

Pediatric age pt presented with his mom to ED she said that her child ingested large
amount of paracetamol (i think) the pt was healthy no signs of any toxicity on
presentation, they observed the pt for 4 hours with no any change in health no signs and
symptoms of toxicity, the mom then mentioned that she found some of the drug was
spilled on his clothes and in the floor, what is next?
A. Reassure
B. Watch for another 4 hours
C. Give antidote ? But No sx

Pt come with violence which imbalance responsiable for violence ?


Low serotoinin or testosteron ?
Ref: psych smle residents
Low serotoinin
62 pt known diabetic
C/O. Cough. SOB. Fever
Chest exam crackle..small pleural effusion
Dx pneumonia
Which of the following is poor prognosis factor?
1.age
2.DM Ref: slideshare, UTD
3.crackles
4. Pleural effusion

Elderly was completely healthy except for elevated BP for the first time
A. Ambulatory BP measurement
B. Start anti HTN
C. Measure the BP two times later on in the clinic
D. Measure BP two time in home

A 45-year-old smoker came to the clinic for his diabetes follow-up. During discussion, he
acknowledged that smoking is not good for his health. He plans to quit this year.
According to the Stages of Change Model, at what stage of change is this patient?

A. Precontemplation
B. Contemplation
C. Preparation
D. Maintenance

psych sale answer


However I Don't agree
Case of Asthma exacerbation , patient present with SOB , cyanosis , unable to complete
sentence.
Vitals = Not Sure ! ( Low O maybe) / RR : Not sure .
ABG ; respiratory acidosis.
What is the appropriate next step?
??
- IV mg sulfate
- Intubation
- Non invasive mechanical ventilation.
- ?/!
Introduction of soild food to children?
6 months

Pubmed

A& B
But A cuz of pregnancy

Child with unilateral wheezing ???


orgein body aspiration

3 month old coming with staccato cough (written in question), history of conjunctivitis a
few days ago. No history of diarrhea. Has basal crackles on auscultation?
- Chlamydia pneumonia
- Adenovirus pneumonia
Ref: AMA
- Pertussis
- Mycoplasma pneumonia
40 or 50 female k/c of dm Came with retrosternal pain at rest for 3 hours ECG done T
inversion in (V2 v3 not sure about leads)
Labs :
high troponin 10

What is your diagnosis

Stable angina
Unstable angina
St elevation infraction
Non St elevation infraction

** Pain and relieve at rest> SA


Pain even at rest no cardiac enzymes> UA
Pain even at rest high cardiac enzymes> NSTEMI
High cardiac enzymes + ST elevation> STEMI

Pt Dx grand mall seizure from child hood on NA valoprate develop. Pt c/o generalized
tonic colonic seizure which drug IV use in management???
1.phenytion
2.diazepam
Amboss
3.phenobarbitone
4. Not remember

Pt hear heart beat sound 2day no other symptoms examination clear lung
Bp 87/56
O2: 90%
PR:265(regular)
What Dx
1.atrial flutter
2.atrial fibril
3.SVT Ok but how to differentiate it from 1 ??
4.complete heart block

A case of PSGN what is the highest confirmatory test


Serum albumin
Proteinuria Amboss
Kidney US

which valvular heart disease has the worst outcome in pregnancy?


A)Mitral regurgitation
B)Mitral stenosis
C)Aortic stenosis Mitral stenosis (MS), the most common manifestation of
rheumatic heart disease, remains the most common acquired
D)Aortic Regurgitation valvular lesion in pregnant women and the most common
cause of maternal death from cardiac causes worldwide.
8year old diabetic female with dysuria. Urine culture showed 10,000 E.coli . High wbc
and high creatinine. Which abx is contraindicated?
A. Nitro
B. Ceftriaxone
C. Trimethoprim-sulfamethoxazole

Pt pain less defication


Blood in th stool. Biles reduced spontanius in 3 areas 3.7....Hb very low
Dx hemmorriid was given which of the following is Definitive management ?
1 conservative manag
2.fiber diet
Medscape
3.rubbber band ligation
4.hemorroidectomy

*** grade 2

male patient htn DM c/o chest pain examination S4 was heard ECG showed t wave
inversion in lead V2-v5
Troponin high
what is the diagnosis
1. Stable angina Repeated

2. Unstable
3. NSTEMI
4. STEMI

Pt received 3 dose of HB vaccine came to clinic


‫عمل فحص للفيروس شنو هو المتوقع‬
1.HBsAng
2.HBeAg Easy Q
3.HBsAB +ve
4. HBcAB+HBsAB +ve
Patient known hypertensive on lisinopril and amlodipine, recently diagnosed as TB
What you will do
1.stop lisinopril We should discuss this
2.stop amlodipine
3.increase lisibopril
4.stop rifampicin

Pregnant with candida?


Oral anti fungal
Topical anti fungal
Oral abx

56 Y.0 male patient presented with the classical triad of polydipsia, • polyphagia and
polyuria. A diagnosis of Diabetes Mellitus was made. Which of the following causes
impaired wound healing in diabetic patients?
• A. Decreased blood supply.
• B. Impaired phagocytosis.
• C. Autonomic neuropathy.
• D. Dehydration.

71-Patient diabetic with pneumonia came with parapneumonic effusion


what will made poor prognosis:

A- DM Again same question, if age is mentioned and its


B- high WBCs 65 or above > i will chose it
C- pleural effusion
Dm is a bad prognostic factor
D- Age
Patient hear aliens
‫يسمع أصوات كائنات فضائيه تتكلم معه‬
Dx.
1.anxeity disorder
2. Severe deprresion Bad recall, its 4 until
scenario is complete
3. bearevmenbt disorder
4. severe bipolar +not remember

US
CBD
‫سيناريو‬
Patient c/o RUQ Pain with
N/V tender examination
Investigation
. High liver enzymes
. High Bilirubin Repeated
High TWBc
US.. Show GB 1.5cm stone
CBD dialation 12 mm
Most appropriate management
1.lap cholecystectomy
2.CT scan
3.ERCP ????

Pregnant lady 30 wk with letharg and fatiuqe tremor i think c/o neck swelling
Investigation
TST. 0.1 ONLY
US diffuse multi nodular swelling
Most appropriate management
A. FNA
B. RA iodine Surgery is indicated only if
C. Thyroid surgry before gestational week 24

D. Anti thyroid drug

** Both methimazole (MMI) and propylthiouracil (PTU) may be used during pregnancy
45 y/o female complete mammogram screening when to do it again ?
-after 1 year
-after 2 years
-no need
-forgot but i think after 6 months

*after 55 of age then every 2 years


So

Aggressive behaviour, what is the cause ?


A. low serotonin
B. high serotonin
Repeated
C. low endorphin
D. high endorphin

patient in 40 came to with fibro adenoma of breast and you did excision and in the lab
they
till you there its benign and inside it there is cancerus
O/E no lymph node palpable in axilla and every thing is normal
what will you do for her
A) mastictomy
B) follow up

Female pt with 2 months jaundice fatigue and pruiritis


Obese Medically free
High AST ALT ALP
Viral serology negative
+ANA
CT Liver
Plasma cell
Management :
Vit E and weigh loss
Methotrexate
Steroid
168 772

2
Vascular disease (history of myocardial infarction, peripheral vascular
disease, or aortic atherosclerosis)
1
Age 65–74 years old
1
Sex (female, 0 points for male)
1

4 yrs child contact with TB patient..PPD skin test 10mm:


1/Positive
2/negative
3/repeat test
4/no need for test

Pt known case of chronic hepatitis B and a symptomatic for follow up


US showed liver with mild coarse texure and nodule 3×3 what most apropriate
investigation:
1/ alfa feto protien
Gold is biopsy however we
2/ biopsy should revisit this Q
3/ US abdomin
4/ triphasic abdominal CT

Pt on oral steroids presented with white layer in his toungue and buccal mucosa
what is the treatment:
1/ nystatin
2/ketoconazole
3/Amphotericine B

Asthmatic patient on ICS and use SABA inhaler at least once daily
What to add to his medications:
1/ leukotriene antagonist
If following the stepwise
2/ salmeterol approach > Laba is next
...
Patient heavy smoker presented with SOB and wheeze
What immediate inhaler to give:
1/ipratrobium promide
2/ ventolin ? Amboss
Senario of COPD
Pic of Periphral blood film show microcytic hypochromic cells with scenario
...HB low/ RBCs low
What suspect to be low:
1/MCV Toronto notes
2/Reticulocytes
3/paltelets

Rheumatoid arthritis patient on NSAID presented with sever epigastric pain and
tenderness
Most apropriate investigation:
1/Erect CXR Amboss
2/US abdomin
...

Senario of patient with nephrotic syndrome then developed generalized


abdominal pain and tenderness
Which of the following describe complication ocurred:
1/peritonitis
2/pancreatitis
...

Pt admited to ICU with inferior MI started ttt then brady cardia i think
What is the diagnosis:
1/first degree heart block Utd
2/second degree heart block
..

Serum K 6.1
Management? Ca gluconate
Amboss
And what ECG change?
Peak T wave

Drugs in decreasing motality in HF?


Indications for therapeutic thoracocentesis

A. Loculated
B. Ph > 7.2
C. High glucose

Lung node measure 8mm


asymptomatic what to do
A) Follow up
B) CT scan
Previously seen
C) biopsy
D)review previous x-ray

Case after motor accident on examination of heart you hear muffled heart sound
(there is more but i forgot)
Labs:
80/40
X-ray: normal
Diagnosis: Previously answered

A - cardiac tamponade
B - hypovolumic shock
A triad of cardiac temponade : hypotension.. Muffled heart sounds , neck vein
distended ( increased JVP )

RTA patient was transfused with 4 L of blood , which of the following would be
the complication?
A- Citrate toxicity Both are correct !

B- Hypocalcemia

128-Patient with HTN not responding for three anti hypertensive drugs on
Ultrasound the one kidney
is bigger what is diagnosis
One kidney should be smaller than the
A Adrenal hyperplasia other > B
B Renal artery stenosis
C Pheochromocytoma

Which is most indicative of malignant nodule rather than tuberculosis nodule:


-Hypercalcemia
The larger the nodule, the more likely it is to be
-Nodule size >5cm malignant > NIH
-
Calcification occur in 14% of lung cancer

Pt has rest tremor but when he move to pick something the tremor gone, but
can’t do finger nose test with his right hand where is the lesion ?

Right cerebellum
Left cerebellum
Right hemisphere

Patient has SLE with neurological manifestation patient already on


corticosteroids, what to add now ?
A- cyclophosphamide
B- hydroxychloroquine
C-phenytoin
D- diazepam

* pt came form southern east Asia with his of 2 weeks SOB cough
O/E Febrile + Rt upper lobe crepitation
Xray ..upper Rt lobe opacity and cavitation..
Best action :
- ceftrixone Need to establish the dx first ✅
- sputum for AAFB
- INH ,rifampcin , ethambutol , Pyrizinamide

* Male pt with features of SLE with active arthritis..ask about best management
- methotrexate
- HCQ
- Methotrexate, HCQ & steroid
-Another option

76 year old male retrosternal chest pain with exertion relieved by rest on
examination S4 was heard ECG showed t wave inversion in lead V2-v5 what is
the diagnosis
1. Stable angina Previously answered however no cardiac enzymes are
mentioned here making angina more likely
2. Unstable
3. NSTEMI
4. STEMI

60 y old new hypertension 160/ 80 which medication start ? ccb

Less than 55 or has diabetic, ACEI


More than 55 or african ,CCB

Cirrhotic patient complicated came with portal hypertension , came with upper
GI bleeding
due to varices diagnosed by Upper Endoscopy, what to give after resuscitation
?
A. Somatostatin
B. Terlipressin
C. Octreotide same l Amboss: octreotide
Utd: Terlipressin
D. None?
Reference: UTD

Hernia no surgical hx
adhesion
meckles diverticulum
malignancy No surgical hx, in this case (hernia) is the most common cause.

Female patient presents With achalasia. what is the most appropriate


management?
Non surgical Pnemoatic dialation Pubmed
https://www.uptodate.com/contents/overview-of-the-treatment-of-achalasia

And botilinium toxin imj


Surgical treatment visit the link for more
Hellar myotomy explanation !
UTD

*pt of mesentric ishemia ..best investigation?


- CT angio
- Explatory Laprotomy Reference: UTD
- US
-Laproscopy

young male came to well baby clinic, upon examination his right testis was
palpable in the inguinal canal and small in size and easily moved to
scrotum, the left is normal, what is the cause?
-ectopic testis Reference: UTD
-undescended testis
-testicular torsion
-Retractile testis

Child with foreign body aspiration? RIGID ENDOSCOP


Reference: UTD

Surgery
*Senarois about abdominal & chest truma Q about most app next step??
* pancreatitis PancreaticPseudocyst
* cholecystites
* breast abcess
*Liver hemangioma risk factors

Pt had polyp 1.5 Tubular adenomas and remove when to follow up


3-6 months
UTD: "
https://www.uptodate.com/contents/

same Q
image?imageKey=GAST/127126
UTD

o
Every 3 years
Every 10 years
Forgot

38 y male complain of constipation for 2 days painful perianal whth positive


crack on 6 & 12 clock
Anal fissuer
Perianal abcess

+- type of polyp with highest risk of cancer


1 villous
2-Tubuler
3- tubulvellous UTD

A 65-year-old male comes to the clinic with a mild intermittent urinary flow
reduction Rectal examination, urinalysis and prostate specific antigen
studies are normal (see report).Ultrasound prostate: Enlarged median lobe.
Which of the following is the best way to investigate?

A. Annual renal function monitoring


B. Periodic PSA measurement UTD
C. Beta-blocker therapy
D. Cystoscopy It’s the most appropriate answer!! Screening for prostate cancer
because the patient’s age is a risk factor. UTD
Spina bifida /what is the best diagnostic method mri

Commonest cause of esophageal carcinoma


Scc: Tobacco and alcohol UTD
Adeno ca: gerd, barretts

Man came with scalp open wound, after 6h assault, what wound
management? A-secondary closure
B-Debridment with 1ry closure UTD
C-Debridment with granulation
D-Leave it for granulation! I think wrote like this

8- Stab wound to the neck and injury to Zone 3. There was active bleeding.
They did not mention if stable or not. There was no Vitals. What is the most
proper initial management?
A. Open repair
UTD
B. Endovascular repair
C. Artery ligation
(Im sure about these choices)

12 year old boy injured with supracondylar fracture and distal radial pulse UTD!!!
absent , which of the following is the appropriate next step in management?
A-K wire
B-intramedullary nail
C-surgical exploration
D-Closed reductio -

Adult soldier with bilateral inguinal swelling with - positive cough impulse what is
the most appropriate management
A - Laparoscopic repair UTD:
https://www.uptodate.com/contents/
B - herniotomy
image?imageKey=SURG/107556
C - Open hernia repair and mesh
D – observation

... patient in 40 came to with fibro adenoma of breast and you did excision and
in the lab they Pubmed
till you there its benign and inside it there is cancerus
O/E no lymph node palpable in axilla and every thing is normal T
what will you do for her High risk for breast
A) mastictomy cancer
B) follow up !!!

Pt pain less defication


Blood in th stool. Biles reduced spontanius in 3 areas 3.7....Hb very low
Dx hemmorriid was given which of the following is Definitive management ?
1 conservative manag UTD
2.fiber diet Most common used!
3.rubbber band ligation
4.hemorroidectomy 2

35 years old female her dad has colon cancer when he was 55 and her mom AMBOSS
had breast cancer when she was 43 asking about screening:
A- She should do mammogram annually bex family hx
AMBOSS

B- Start mammogram at 40
C- Start mammogram at 40 and colonoscopy at 55
D- Colonscopy

37 yo with Breast lump hard , next ? mammogram


AMBOSS
An old patient underwent rectal surgery, developed DVT, what is the
management?
A-Enoxaparin
B-Heparin
C-Warfarin
D-IVC

pancreatic pseudocyst, one is 18 cm x 24 cm or 17 cm , how to manage ? AMBOSS


A- per cutaneous drainage
B- endoscopic drainage
C- surgical drainage

Q3 - 27-year-old obese woman presents with right iliac fossa pain associated
with anorexia, nausea, and vomiting. On examination, there is moderate right
lilac fossa tenderness. Labs: leukocytosis UTD
What is the most appropriate management?
A - Open surgery
B - CT
C - US
D - Diagnostic laparoscopy

90-Year-old man with multiple comorbidities presented with many cancer in


stomach, lung and colon but the original site of the cancer is unknown. What’s
your most appropriate action?
A. resection of tumors
B. Palliative chemotherapy
C. Comfort care = Palliative care

Patient fall from 4 floor with open woud in leg and necrtic dirty tissue and stable
first to do UTD
A antibiotic
B surgical debridemnt
C immediate reductio

Pateint with immunodeficiency disease has peranal region mass with


fluctuations and erthema creptius management AMBOSS
A injection of antibiotics
B aspiration
C topical cream i cant remembe its name
UTD
Perianal abscess.
Incision and drainage Culture should be obtained and Abx.
Pubmed
Victim of rta ..Hr.50
BB.60 /45 how tncrease BP???
1.normal saline
2.bloood negative o

Long air travel Rt side diminished air entry???


Rt site tension pneumothorax
UTD
RT SITE HEAMOTHORAX

Child RtA weak periphral puls .BP 70/40


Next step??
OROPHARYNX TUBE
AMBOSS
The logic answer!
The priority for IV NS
if present in the
choices..

G ENDOTRACHEAL

P
NASO GASTRIC

35 years Male complain of epigastric pain increase when supine position and
not good taste ?? AMBOSS
Peptic ulcer
Esophagitis GERD

p
Gastritis

40 Male co abd nominal pain associated with faty meal ...relive in 2 hour Dx??
Gastritis UTD
Biliary colic
Pancerititis

Doctor to save pt life ..hysterectomy???


Do without consent ...

Lactiting women small around mass in aerolar with small inverted of nipple and
reteurn normallyDx???? UTD
Fibro adenoma
Breast cyst
Ductactesia

Pt6 hrs post femoral artery repair, pt developed swelling and neurological signs
What to do next
AMBOSS
Cta UTD
Ct 2
Duplex ultrasound
Compartment pressure release

Pelvic organ prolapse at the introitus


A-cyctocele
B-enterocele
C-rectocele
D-paravaginal

Patient known case of peptic ulcer disease in medications but failed to respond,
uppe endoscopy done and revealed multiple AMBOSS
ulcers in antrum. What is the best treatment? A. Pyloroplasty and vagotomyB.
Total gastrectomy C. partial gastrectomy

Q24 - An old male patient was admitted as a case of large intestinal obstruction.
He underwent rigid sigmoidoscopy that showed a mass in the sigmoid region. A
biopsy was taken and came back as adenocarcinoma.
What is the best next step? UTD
A - Colonoscopy
B - CT abdomen
C - MRI pelvis
D - Sigmoidectomy

Man came with scalp open wound, after 6h assault, what wound
µ management? A-secondary closure
B-Debridment with 1ry closure
C-Debridment with granulation
D-Leave it for granulation!

AMBOSS
ileal resection cause any vitamin deficiency
*bile
B12
Iron
...

Case acute RUQ pain,jaundice,US thickened GB and bladder stones what to do


-Laparoscopic cholecystectomy UTD, AMBOSS
-ERCP The pt jaundiced, which raise suspicious of CBD, in such
-I dont remember may be IV Fluid case ERCB.
00000000000000000 Surgery is not optional because it is acute case.

A 45 year old with history of pancreatitis , then recurrent vomiting, + upper GI


bleeding
After resuscitation what should be done Splenomegaly, normal portal , thrombo-
splenic . He
became stable after injection of sclerotherapy , which of the following best way
to
UTD
management ?
• A. Splenectomy AMBOSS
• B. Splenorenal shunt
• C. Venocaval shunt or portocaval shunt
• D. None

56 y/o woman presents to the clinic with a non-healing ulcer over her right
lateral malleolus, she is hypertensive. pulse is normal and her local exam shows
dark discoloration of the skin around the ulcer & a viable ulcer bed. best next
step is ? UTD

A. CT angio
B. Venous duplex US
C. Arterial doppler US
D. Conventional angio

403- Patient “45 years old, came with history reducible hernia in inguinal area it
pop out every two
days”interval” with mild pain and and he Manuel reducible it by his finger, then
he suddenly
developed severe “constant ” pain in early morning in that area and came to
hospital, on examination
of the inguinal area and scrotum examination and he is free from both them and
pain is disappear
Now just with persistent nausea and vomiting next step ? AMBOSS
A- Pelvic and Abdomen CT
B-hernia repair today after 2 hour i think
C reasurance and discharger
D hernia repair after 2 days

Elderly Patient with small lung nodule (<8 mm), he is asymptomatic, not smoker,
no family hx of cancer, no constitutional, your next step?

A. f/u with CT scan


B. Biopsy
C. Chemotherapy

Patient with chronic limb ischemia, presented with sudden leg pain, diminished
popliteal and distal pulses in right leg, and diminished distal pulse with intact
popliteal in the left, what's the appropriate next action?

AMBOSS
AMBOSS

E A.B. Heparin
CT angio
then ultrasound

C. US
D. conventional angio

...

Thyroid cancer with increased calcitonin:


r
A. Medullary thyroid cancer
B. Follicularthyroidcancer

Patient present to ER after RTA, with SOB, in examination the tracheal shifted to
the left side, in chest X-ray, the lungs are expanded and winded mediastinum.
What is the diagnosis. 7
A. Tension pneumothorax
B. Cardiac tamponade
C. Plural effusion
D. Raptured esophagea

Patient post RTA with massive bleeding in the nose and mouth with leg fracture,
decrease bp, Pt can take his breath but he is afraid from what happened. What
is the next step?
AMBOSS

A. Intubate with stretching cervix


B. Packing nasal bleeding
C. IV fluid

14 y.o female with epigastric abdominal pain since a month Patient was
complaining that it is affecting her life, On exam patient had multiple bruises in
abdomen when asked about itShe said it appeared after she fell over her cycle UTD
18 days back, next inv?

A. CT abdominal
B. Abdominal US AMBOSS
D. laparotomy
UTD

patient had polyps excised from his sigmoid colon and on histopathological

G
report there was cancerous cells with free tissue margins, Next step:

A. Follow up
B. Sigmoidoscopy
C. Sigmoidectomy AMBOSS

S
Patient with epilepsy, most common shoulder dislocation:
AMBOSS
A- inferior
B- supraacromine posterior

5 y/o boy one testicles undescended. Mx? AMBOSS

A. Orchidectomy
B. Orchidopexy
C. Wait till puberty

breast feeding mother found unilateral mass on the lower outer quadrant of right
breast (i think 3x3 cm) it was erythematous and hot on palpation. Mildly tender. mastitis
Most appropriate management?

A. chemo therapy
B. radiation therapy
C. needle core biopsy
D. Aspiration + Abx

A child has scrotal pain ,on exploration the cord was edematous and inflamed
with red right hemiscrotum
SMLE-B

A- Testicular torsion
B- Inguinal hernia
C- Testicular appendages torsion
D- epididymoorchitis

GCS (confused, open to sound, localized pain)


UTD

A- minimal
B- mild
C- moderate
D- severe

Interpretation
• GCS 3 (minimum score): Deeply comatose or imminent brain death
• GCS ≤ 8: severe TBI
• GCS 9–12: moderate TBI
• GCS ≥ 13: mild TBI
• GCS 15 (maximum score): Full consciousness

A case of cholangitis: what is the best imaging?


UTD
A. Serial X-ray
B. U/S
C. MRCP

‫ ﻟﺸﮭﺮ اﺑﺮﯾﻞ‬fair , smle b , asma ‫ﺑﺤﻤﺪﷲ ﺗﻢ ﺟﻤﻊ ﺟﻤﯿﻊ اﻻﺳﯨﺔ اﻟﻤﻮﺟﻮدة ﻓﯿﮫ ﻗﺮوب اﻟﺘﻠﺠﺮام‬
‫ﻟﺴ ﺖ ﻣ ﺘ ﺎﻛ ﺪ ﻣ ﻦ دﻗﺔ ا ﻹ ﺟ ﺎ ﺑ ﺎ ت‬
Dr. GROUP

This file has been collected and double checked by Dr. Lama, Dr. Azzah, Dr. Muluk, Dr.
Amal, Dr. Hussam, Dr. Rk, Dr. Wejdan, Dr. Raghad, Dr. Group members, with highlights
and notes with the the aid of the available resources and mentors. We hope that it would
be including most of January 2021’s repeated questions solved mostly correctly as more
than two thirds or by whole. We continue to double check most of the questions and
update them so stay tuned. We wish you all prosperity and success in your multiple
SMLE trials! Follow us on Dr. Groups!
‫ال تنسوا كل من ساهم في هذا العمل من دعائكم‬

1-most recommended tool for checking prognosis after asthma attack?

A- Chest x-ray

B- ABG BMJ

C- Pulse oximetry

D- Peak flow rate

2- Female came with white breast discharge and high prolactin, what radiology you will do?
UTD

A- Pelvic MRI

B- Chest MRI

C- Sella turcica MRI


3- 60 year old male diabetic BMI: 41, diagnosed with BPH. What is the most important risk factor?

A- Age UTD

B- Diabetes

C- Obesity

D- Hypergonadisim (something like that)

4-Miscarriage in an old lady (~45yrs) she asked if her age had anything to do with her miscarriage:

A- risk of miscarriage is 3% at this age


UTD
B- from 10 to 50%

C- 80%

D- no risk

Hacker and Moore

5-An 18 or 25 yo F. Worried about cervical cancer. She took her first dose of HPV vaccine 3 months ago.
What the best thing to do at this visit today?
UTD
A. Schedule app after 3 months

B. No need to do anything at this visit

C. Give 2nd dose at this visit

D. Repeat 1st dose

>15 of age 0,1 to 2, and 6 months

6-A pediatric patient came in RTA. Has splenic rupture and thus splenectomy was done.

What’s an appropriate vaccine later on?


UTD
A. Pneumococcal

A. Meningococcal
C. MMR

7- week GA pregnent lady with UTI witch of the following drug is contra-indicated:

A- nitrofurantoin UTD
B- ciprofloxacin

C- amoxicillin

D- Ceftriaxon

8-Screening for pregnant for asymptotic Bacteriuria according to U.S. Preventive Services Task Force
(USPSTF)
UTD
A- 12 weeks

B- 20 weeks

C- 26 weeks

D- 32 weeks

9-camping in the community to educate people about health determents considerd as?

A-health education Fair + SMLE-B + Earth !

B-active surveillance

10-pt with fever and chest pain and they mentioned an ECG finding they ask about which part will be
affected

A- Pericardium

B- Myocardium
11- a 42 y.o male, with rectal bleeding, a biopsy from sigmoid showed “adenoma”, at what interval shall
you do colonoscopy screening?

A- 3 and 6 months

B- 3 years

B- 10 years

C- No need
UTD

12-Case of stroke and complain of weakness in the arm leg and face, where’s the damage?

A. Middle cerebral artery

B. Anterior cerebral artery

C. Mid basilar artery

13-old patient came with subdural hematoma with signs of lateralization imaging revealed 13 mm
shifting. his GCS 7/15 then was intubated and resuscitated what to do next

A- iv mannitol BMJ
B- admit ICU and observe

C- craniotomy

D- insert intracranial monitor

14-Old patient hx of HF with dialated cardiomyopathy and A.fib .. what is the most appropriate
management to control his heart rate? BMJ
UTD
A- Digoxin

B- amlore

15- Pregnant lady everything was normal but she complain of itching and had elevated liver enzyme,
what is the dx?
Intrahepatic choleestasis of pregnancy
A- Viral hepatitis
UTD
B- Pregnancy cholestasis

16- Victim of RTA came with pelvic fracture and there’s blood in the meatus, next step?

A- Folley catheter
UTD
B- Ureteroscope urethrogram

C- Pelvic CT

17- RTA pt with maltipule mandpular fracuter with sever bleeding, uncontious, no vitales menstioned

How would you mange his Airway? UTD

A. Laryngial mask

B. Orotracheal

C. nasotracheal

A- D.Cricothyrotomy

18-What of the following decrease the risk of preeclampsia? UTD


A- Antibiotic

B- Aspirin

19-The time for introducing solid food for baby;

A- 14 months
UTD
B- 18 months

C- 22 months

D- 6 months
UTD
20-Immune competence with aspergillosis infx

Voriconazole initial treatment

Add echinocandin in immunosuppressed

21-Pt was on malaria medication doctor didint now about it and started him on digoxin

Anti-malarials increase serum digoxin le

vels (reaction)

23- Cystic fibrosis complications

• Nasal polyps
• Chronic and persistent sinusitis with complications such as mucopyocele formation
• Bronchiectasis one of the correct choices if there was no pancreatitis
• Atelectasis
• Pneumothorax
• Hemoptysis
• Hypertrophic pulmonary osteoarthropathy
• Allergic bronchopulmonary aspergillosis (ABPA)
• Gastroesophageal reflux
• Pulmonary hypertension
• Cor pulmonale
• End-stage lung disease
• Pancreatitis most common uptodate
• Cystic fibrosis–related diabetes mellitus
• Meconium ileus
• Distal intestinal obstruction syndrome
• Rectal prolapse
• Vitamin deficiency (especially fat-soluble vitamins)
• Fatty liver
• Focal biliary cirrhosis
• Portal hypertension
UTD
• Liver failure
• Cholecystitis and cholelithiasis
• Rickets
• Osteoporosis
24- Flight nausea tx

A-Ondasetron

B-Omeprazole

Ondasetron has been effective in motion sickness, diphenyhdramine is for prevention

Not clear q!! 25- Iud with brown discharge

COC, NSAIDS

If there was signs of infection start abx

It is probably due to expulsions /malposition uptodate

Fair + SMLE-B
2

26- Blood diarrhea +joint pain tx

5-ASA

27- Anofistula intenal openning ? Earth

Left Lateral

28- Preschool most important vaccination

MMR Dtap varicella OPV


UTD

29- Pt was on malaria medication doctor didint now about it and started him on digoxin
SMLE-B:
What should done to avoid this complication: An interaction between quinidine and digoxin is
known to occur with toxic concentrations of digoxin
A- Pt should bring his medication in the serum resulting when the two drugs are
administered together. The effects can be very
B- Nurse should know what pt is on dangerous, especially in those patients with advanced
and severe cardiac disease. doctor is faulty because
C- Computerized system medication he did not take medication history and patient also
D- There was no choices about taking details history from pt or faulty because he did not inform the medical staff
about his medications. computerized healthcare
system of course will prevent many mistakes to
happen but in places where no electronic facility,
health education for the patients to be careful in
30- Cystic fibrosis complications ; giving full information to prevent such events and for
the health practitioners also to be very careful in
taking history, apologizing after harming the patient
A-Bronchiectsasis will not fix the damages or losses.

B-Pulmonary emphysema

SMLE-B

32- Iud with brown discharge

A-Utrine perforation

B-Pid
32- Pt found to have anofistula on 7 o’clock

M.c Anofistula intenal openning ?


Earth + Fair
A-Lt lateral

B-Medial posterior

C-Medial anterior

: 33- Surgery safety checklist:

A-Before anesthesia, before skin incision, before leave OR.

B-before admission, before anesthesia, before discharge

C- before OR, during operation, at recovery room

34- Pt sustained injury to the left chest now there is sucking wound between 3rd and 4th ICS how will
you manage : Dx: Open pneumothorax

A- Chest tube

B- three sides dressing

C- analgesics

35- Pt had previous ectopic pregnancy.

The percentage of recurrence?


UTD
A- 5%

B- 10%

C- 30%✅

D- 50%

Hacker and Moore says 30% although there was a debate between 10% /30%

36- clinical pic of opioid toxicity, asking about the antidote?


UTD
Naloxone✅
37-5- 14 y/o boy ingested bottle of acetaminophen tablets brought by his parents 20 hour later, asking
about which stage of toxicity?
UTD
A- 1

B- 2

C- 3

D- 4

UTD
38- which type of RTA comes with Hyperkalemia?

A- RTA type 1

B- RTA type 2

C- RTA type 3 hyperchloremia

D- RTA type 4 ✅

(Renal tubular necrosis)

39- elderly with dysuria, and dificulty in urination, PSA high, ALP high, what is the diagnosis? UTD

A- Urinary bladder ca

B- Prostatic ca✅

C- BPH

40- Risk factor for esophageal ca? UTD


A- esophageal stricture

B- Barrett's esophagus✅

C- GERD

41- A mother brought her 4 years old child cause he frequently wakes up in the middle of the night
setting on the bed , eyes opened but unresponsive for 2-3 min then he go back to sleep , in the morning
he doesn’t remember what happened?
UTD
Night walking & seizure!!

A- nightmares

B- night terror

C- night walking✅

D - epilepsy 2
42- Full term baby will be discharge from hospital what will you give him?

A- BCG, HBV

B- HBV, MMR

43- patient with renal failure presents with low calcium, high alkaline phosphatase, and high PTH. What
is the best form of vitamin D to give this patient?
BMJ
A- Calcitriol✅

B- Vitamin D2

C- Vitamin D3

2
44- Pt came 3 days after roux-Y surgery complaining of fever chill and left shoulder pain, best diagnostic
investigation?
UTD
A- CT with contrast ✅

B- Endoscopy

C- Laparoscopy

D- exploratory laparotomy

45-- 35 year old lady presents with left nipple bloody discharge, diagnosed as Intraductal papilloma, how
will you treat her?

A- duct exsion✅
UTD
B- opservation

C- mastectomy

46- child with Spinal Muscular Atrophy. The pediatrician deemed he needs intubation. Both parents
don’t want intubation as they had a son with the same illness who died. What do you do?

A- call child protection


B- Search the internet for solutions of similar cases

C- Discuss with hospital ethics committee

D- Ignore both parents and intubate the child as this is an emergency✅

46- What treatment will you give for moderate asthma?


AMBOSS
A- SABA

B- LABA

C- LABA + Inhelar corticosteroids ✅

D- LABA + Anticholinergic

47- pt came with his shoulder flat, his arm addcted and internally rotated, what is the diagnosis?
AMBOSS
A- Anterior dislocation

B- Inferior dislocation

C- Posterior dislocation✅

48-Patient post thyroidectomy had arm spasm during blood pressure measurement, what is your next
step?
AMBOSS

A-Give analgesia

B-Take blood pressure again

C-Check calcium level✅??

Hypocalcemic symptoms, treat Hypocalcemic then recheck

49-Child had Type1 diabetes and was consulting the family regarding celiac screening, which of the
following is true:

A-Screen at diagnosis then every 5 years UTD


B-Screen at diagnosis then every 2 years✅

C-Screen annually
50-Milestones of a child that can ride a tricycle but can’t draw a square?

3✅

51-Cervical cancer screening for a 35 year old woman: UTD

Not indicated

Every 5 years pap and hpv✅

Annually

52-Menopausal woman complaining of insomnia and flushing:

A-Ocp oral and ssri UTD

B-Local hormonal replacement

C-Systemic hormonal replacement

53-Pneumonia case asking about the appropriate abx regimen:


AMBOSS

A-Ceftriaxone + azithro✅

B-Vanco + azithro

C- Meropenem + azithro

54-Vaccines for a 9 month old:


UTD

Measles - MCV
Brain natriuretic peptide
55-Which of the following can cause an increase in BNP:

UTD
A-COPD✅

B- Furosemide

C- Obesity

56-Which antipsychotic is associated with weight gain:


AMBOSS

A-Olanzapine and colanzpine ✅

Choose clozapine if there were both

57-Long case of a medullary thyroid cancer (diagnosis given) what is the appropriate management?

AMBOSS
A-Sub total thyroidectomy

B-Total thyroidectomy ✅

C-Hemithyroidectomy

58-Case of a patient who had itchy skin in scalp and multiple areas (eczema), with recurrent infections,
and thrombocytopenia. AMBOSS

Wiskot Aldrich Syndrome✅

aah
59-Case of a pregnant woman in 39 week of gestation who had severe headache, with hypertension and
proteinuria.

AMBOSS
A-Plasma exchange

B-Delivery✅
60-6 year old child who’s toilet trained and complaint of bedwetting recently

2 UTI✅

61-most recommended tool for checking prognosis after asthma attack?

A- Chest x-ray

B- ABG

C- Pulse oximetry

D- Peak flow rate✅

62-Female came with white breast discharge and high prolactin, what radiology you will do?

A- Pelvic MRI

B- Chest MRI

C- Sella turcica MRI✅

63- 60 male diabetic BMI: 41, diagnosed with BPH. What is the most important risk factor?

A- Age✅ AMBOSS
UTD
B- Diabetes

C- Obesity

D- Hypergonadisim (something like that)

64 - -**year-old boy, diagnosed with TB. On rifampin. What is the side effect?

A- discoloration in body fluid


UTD
B- changing behavior
C- decrease immunity

D-Renal low plt and hg

65- Miscarriage in an old lady (~45yrs) she asked if her age had anything to do with her miscarriage:

A- risk of miscarriage is 3% at this age

B- from 10 to 50% ✅

C- 80%

D- no risk

years and rose to a high of 57 percent for women age ≥45 years uptodate

Up to 50 % hacker

66-An 18 or 25 yo F. Worried about cervical cancer. She took her first dose of HPV vaccine 3 months
ago. What the best thing to do at this visit today?

A. Schedule app after 3 months

B. No need to do anything at this visit

C. Give 2nd dose at this visit✅

D. Repeat 1st dose

67- A pediatric patient came in RTA. Has splenic rupture and thus splenectomy was done.

What’s an appropriate vaccine later on?

A. Pneumococcal✅

A. Meningococcal

C. MMR

67-10 - week GA pregnent lady with UTI witch of the following drug is contra-indicated:

A- nitrofurantoin
B- ciprofloxacin

C- amoxicillin

D- Ceftriaxon

68- Screening for pregnant for asymptotic Bacteriuria according to U.S. Preventive Services Task Force
(USPSTF) :

A- 12 weeks✅

B- 20 weeks

C- 26 weeks

D- 32 weeks

69-camping in the community to educate people about health determents considerd as?

A-health education✅

B-active surveillance

70-pt with fever and chest pain and they mentioned an ECG finding they ask about which part will be
affected

2 A- Pericardium✅
B- Myocardium

71- a 42 y.o male, with rectal bleeding, a biopsy from sigmoid showed “adenoma”, at what interval
shall you do colonoscopy screening?

A- 3 and 6 months
B- 3 years ✅

B- 10 years

C- No need

72-Case of stroke and complain of weakness in the arm leg and face, where’s the damage?

A. Middle cerebral artery✅

B. Anterior cerebral artery

C. Mid basilar artery

73-old patient came with subdural hematoma with signs of lateralization imaging revealed 13 mm
shifting. his GCS 7/15 then was intubated and resuscitated what to do next

A- iv mannitol

B- admit ICU and observe

C- craniotomy

D- insert intracranial monitor

The best next step after resuscitation is craniotomy for evacuation ASAP

74-Old patient hx of HF with dialated cardiomyopathy and A.fib .. what is the most appropriate
management to control his heart rate?

A- Digoxin

B- amlore

75-Pregnant lady everything was normal but she complain of itching and had elevated liver enzyme,
what is the dx?

A- Viral hepatitis

B- Pregnancy cholestasis✅
76-Victim of RTA came with pelvic fracture and there’s blood in the meatus, next step?

A- Folley catheter

B- Ureteroscope urethrogram✅

C- Pelvic CT

77-RTA pt with maltipule mandpular fracuter with sever bleeding, uncontious, no vitales menstioned

How would you mange his Airway?

A. Laryngial mask

B. Orotracheal

C. nasotracheal

D.Cricothyrotomy✅

78-What of the following decrease the risk of preeclampsia?

A- Antibiotic

B- Aspirin✅

79-The time for introducing solid food for baby;

A- 14 months

B- 18 months

C- 22 months

D- 6 months✅

80-Most common type of fibroid:


AMBOSS
A-Submucosal

B-Subdural

C-Intramural ✅

81-38 wk Pregnant with preeclampsia ( headache, high BP, ) management

Delivery ✅

82-Pregnant with bleeding, placenta abruption


Wafa
A- Rapid response team ✅

B- ICU admission under OBs care

83-Case of cirrhosis with ascites L I UTD

Presenting with abdominal pain

Management:

A- Antibiotic

B- digonostic laparoscopy

C- exploratory laparoscopy

84-Women with Breast benign phylloid tumor, management


UTD
A- localize excision

B- mastectomy

C- localized excision + radiotherapy

85-Patient with rest tumer, mask face, shuffling steps:


UTD
Parkinson’s disease

86-Alternative treatment for sever depression:


A-TCA UTD

B-Psychotherapy

C- Electrical

‫ثوابه‬
87- Post appendectomy with RLQ pain UTD

X
_ The best step in management would be to looking for any post-operative collections by CT that would
m
indicate percutaneous drainage and piperacillin tazobactam until a culture is taken for abx selection

_The question has missing information

88-Patient post RTA present to ER with urethral trauma

Next step in management


UTD
Suprapubic catheter

89-Ministry recommend doing campaign to educate people about decreasing high carbs and fatty diet to
decrease the risk of HTN
UTD
Type of prevention

A- primary ✅

B- secondary

C- tertiary

D- primordial

90-Case of cirrhosis with ascites

Presenting with abdominal pain

Management:

A- Antibiotic

B- digonostic laparoscopy

C- exploratory laparoscopy
91-Patient post RTA present to ER with urethral trauma UTD

Next step in management

Suprapubic catheter

92-Same with urethral trauma:

A- urethral catheter

B- pelvic CT
UTD
C-urethroscopic urethrogram

93-Asthmatic on Short acting B blocker

With increase symptoms with exertion

What to add:

A- Inhaler corticosteroid

B- inhaler corticosteroid + long acting B blocker

94-Patient with cough and inspiratory strider, But croup ruled out

What is best management

A- warm humidifier

B- repeat epinephrine dose

Approved by doctors group

95-most common cause of abortion in first trimester

A- chromosomal abnormalities ✅
UTD
B- uterine abnormalities

C- infection UTD

96-Women present with vaginal bleeding, gynecologist ordered Hysteroscopy,

What is the complication of doing hysteroscopy ?


UTD

a- infection

b- hemorrhage

If there was perforation then choose it

97- Sca pt pregnant, antenatal complication associated with her condition?


UTD
A) Low birth weight

B) Chest infection

C) Uti

Answer:

98- Pt had fibroid myomectomy, risk of placenta accreta? UTD

A) Decrease

B) Increased

C) Unknown

D) No risk

Answer: approved by uptodate and obgyne consultant

99-with hepatic benign tumor dx is hemangioma (mentioned), decided to be managed conservatively,


but we should advise the pt to?
BMJ
A)Avoid exercise!/trauma

B) Loss weight

C) Smoking cessation

100- Pregnant lady GA (not sure but full term) 24 h ROM. Mother vitally stable

A-CTG reassurance UTD

B- Consecative

C- Induce labor

D-Emergent CS.

We suggest prompt delivery for women with term PROM. Labor is induced as soon as possible, unless
there are contraindications to labor or vaginal delivery, in which case cesarean delivery is performed as
soon as possible.

Vaginal delivery if no indications for Cs UTD

101- Hypothyroidism in thyroxin, need to elective surgery

A) Stop thyroxin than do surgery


BMJ

B) Throxin preced surgery

C) Delay until Euthyroid

102-35 year old female, smokes 12-14/day. Came to antenatal clinic .. what is the greatest risk factor for
9 sub fertility? I
A) smoking

B) Maternal age

103- Newborn, full term, diabetic mother with Respiratory distress and desating, every thing normal
except respiratory distress. No vitals mentioned ?
AMBOSS
A) Intubation and MV

B) ICU, monitor blood glucose and observation

Answer

104- scenario of baby 1 months that wake from sleeping ever some hours and crying for 2 to 3 hours?
This is normal in?

t
A) First 3 months

B) First 6 months UTD

What is the TX ?
AMBOSS É
105-long scenario and the end they said in the blood film there is spherocytes , Hb was 9

A) hydroxyurea

B) transfusion

C) FFP

106-40s female with heavy bleeding came to ER , what to do ?


BMJ
A) D&C

im
B) mefenamic acid

C) IUD

D) hysterectomy

107-30 s yo male with Lung nodule, no symptoms no wt loss nothing, what makes you think it’s
malignant ?

a
A) high serum calcium
UTD
Another
recall for Q
108

B) size more than 2.5 cm

C) Pt age

108- male in his 40s on the maximum dose of PPI (written like this), still has symptoms, did scope which
was normal, what is next step?

A) 24 ph monitoring

B) manometry

I
There is a similar question with Esophagitis in which there is mild improvement with PPI and the answer
would be nissen.
UTD
109-Toxoplasmosis in Hiv patient TX?

A) sulfadiazine and pyrimethamine

110- invasive aspergillosis TX ? SMLE-B


UTD
A) Voriconazole

111- most common genotype of hepatitis?

A) 4

112- Post hiatal hernia repair, when can she get pregnant?

A) 3 months Pubmed
B) 6 months

C) 12 months

113-2 or 3 yo with swelling behind the ear, and FNA showed was clear lymphatic fluid. What to do:

A) Surgery ‫العبيدي‬

B) Sclerotherapy
Gets
C) Radiotherapy

D) chemotherapy UTD

t
114-Female had previous ectopic pregnancy want to conceive:

A) early detection of pregnancy

B) folic acid therapy

115- 42 yo hx of molar pregnancy 2 years ago, she wants to conceive. What to do regarding her history:

A) early follow up in pregnancy

B) contraception she shouldn’t get pregnant


UTD

116-female postpartum 2 weeks , she is breastfeeding, what contraception method ?

A) POP

B) medroxyprogesterone acetate

C) COC

D) IUD

117-Pt intrapartum, after delivery of the baby she has 800 ml vaginal bleeding, what type of bleeding ?AMBOSS

A) 1ry hemorrhage UTD

B) 2ry hemorrhage

118- female GA 31 presented with cervical dilatation, no contractions, what to give ?

A) dexa & tocolytic UTD

B) dexa & ampicillin & erythromycin

Drs. GROUP : A

Obgyne consultant :B, and I personally agree with B.

119- Pt 30s with irregular period for 6 month, she uses pills to get the menstruation , he has hx of
infertility for 2 years , underwent many investigations: Anovulation (ovarian factor)
First line of medication (anti estrogen) clomiphen

gym
TSH normal

Prolactin normal

Progesterone and estorgen were low

FSH, LH are high

What to give for fertility:

A) GHRN

B) ovulation induction with clomiphene

C) ovulation induction with Gonadotropins

120- A child with pica, hepatosplenomegaly and failure to thrive. Coming from a low socioeconomic
status family. HGB 8, Lead 2, PT high, PTT, INR, What’s the most appropriate treatment?

A) Vitamin K UTD

B) D-penicillamine

C) Iron supplements

121- When to do urine test for pregnant


2
A) 10

B) 15

C) 20

USPTF: 12 weeks

122- 45 yo female came for colonoscopy screening:


UTD
A) Till her that screening start at 50 of age

B) Low risk 50 - High risk 10 years younger

133- 35 year old woman known case of DM and knee osteoarthritis ,She has been using NSAID to relieve
her pain. She developed HTN, Na normal, K below borderline. What do think the cause of HTN ?

A) Essential hypertension

B) pheochromocytoma

C) NSAID induced HTN

D) Primary hyperaldosteronism

Labs are consistent with PHA

Nsaids cause hyperkalemia

134-16 years presented to ER after multiple generalized tonic clonic seizure what is the management
“no fever”? Status epilepticus
AMBOSS
A) diazepam

B) phenytoin

135-Pt with knife injury in the hand reached a tendon and nerve

_y
A) primary repair for structure

B) Debridement with primary closure

C) Debridement with secondary closure

136- Sigmoid resection what is the highest rate of the histopathology to be malignant:

.A Villous

B. Tubulovillous

137- pt have HTN, heavy bleeding and anemia what contraceptive she can use?

A) tube ligation UTD


B) IUD mirena
UTD

C) condom

D) OCP
2
138- Child 9 months introduced to solid food late, still breast feed. What will you give:

A) Iron

B) Vit D

139-neck injury in zone 3, CTA show’s vessel injury what’s your next step

A) ligation

B) primary suture

C) embolization

140-Family physician have approved FDA drug for something and Share holders To community

A) conflict of interest

B) neglected something

141-foul smelling watery diarrhea, green in color, what is the causative organism?

A) shigella UTD

B) salmonella

C) Rota

142- Woman takes OCP with strong androgenic progestin found out she is pregnant what do you expect
to find in female fetus

A) External genitalia masculine ( virilization) UTD

B) Feminine

C) Adrenache -
D) No effect
It
child hypoglycemia & seizure & metabolic acidosis & characteristics smell & positive ketone Diagnosis?

A) Phenylketonuria AMBOSS

B) maple syrup urine disease

Pediatric pt suddenly developed sob after Insertion of coiled orogastric tube which is shown on the xray,
what will be the complication that will most likely occur?

Aspiration
AMBOSS
Gerd

Mechanism of acyclovir in chickenpox

A) Reduced duration /severity


B) Rapid Resolution of illness

Pt with neurogenic shock (warm pink extremities) hypotension and bradycardia (Hr 45). He received 1 L
of Iv crystalloid and did not improve, what is the most appropriate mx?

Administration of another liter of Iv crystalloid

Pregnancy with IBD which medication to stop:

A) hydroxy

B) metferin

C) Angiotent reception inhibitor

Anti-monoclonal antibodies :

3 years old with 10 cm mass lateral neck mass on aspiration ear


‫سؤال مختلف عليه‬
lymphatic fluid what to do? r
a
Chemo✅

Radiation

Surgery

Sclerotherapy

Thyroid mass no mention of thyroid profile US showed large cystic mass


SMLE-b
with small solid mass

FNA from cystic

- FNA from solid ✅

US

Thyroid scan

:long scenario and the end they said in the blood film there is spherocytes , Hb was 9
What is the TX ?

A) hydroxyurea

B) transfusion

C) FFP

40s female with heavy bleeding came to ER , what to do ?

A) D&C

B) mefenamic acid

C) IUD

D) hysterectomy

: 30 s yo male with Lung nodule, no symptoms no wt loss nothing, what makes you think it’s malignant ?

A) high serum calcium

B) size more than 2.5

C) Pt age

: male in his 40s on the maximum dose of PPI (written like this), still has symptoms, did scope which was
normal, what is next step?

A) 24 ph monitoring

B) manometry

: toxoplasmosis in HIV Pt TX ?

A) sulfadiazine and pyrimethamine

: invasive aspergillosis TX ?

A) Voriconazole

: . most common genotype of hepatitis?

A) 4

Pt intrapartum, after delivery of the baby she has 800 ml vaginal bleeding, what type of bleeding ?

A) 1ry hemorrhage

B) 2ry hemorrhage

: A child with pica, hepatosplenomegaly and failure to thrive. Coming from a low socioeconomic status
family. HGB 8, Lead 2, PT high, PTT, INR, What’s the most appropriate treatment?
A) Vitamin K

B) D-penicillamine

C) Iron supplements

When to do urine test for pregnant

A) 10

B) 15

C) 20

: 45 yo female came for colonoscopy screening:

Till her that screening start at 50 of age

- Low risk 50 - High risk 10 years younger

: year old woman known case of DM and knee osteoarthritis ,She has been using NSAID to relieve her
pain. She developed HTN, Na normal, K below borderline. What do think the cause of HTN ?

A) Essential hypertension

B) pheochromocytoma

C) NSAID induced HTN

D) Primary hyperaldosteronism

: 16 years presented to ER after multiple generalized tonic clonic seizure what is the management “no
fever”?

A) diazepam

B) phenytoin

: Pt with knife injury in the hand reached a tendon and nerve

A) primary repair for structure

B) Debridement with primary closure

C) Debridement with secondary closure

Sigmoid resection what is the highest rate of the histopathology to be malignant:

A. Villous

B. Tubulovillous

: pt have HTN, heavy bleeding and anemia what contraceptive she can use?

A) tube ligation
B- Intravascular hemolysis
C- ICU related jaundice
D- aclculus cholycystitis

156) what is the Symptom ssuggest severity of dementia

A- Increase irritability
B- Difficulty finding word
C- Forget appointment
D- Difficulty in reading

My answer is C

157) Typic Copd case ((SYMPTOMES was not severe ))

The mMRC (Modified Medical Research Council) was 2

A- Combined LABA + ICS


B- Phosphodiesterase-4 (PDE inhibitors + LABA + ICS
C- SABA as needed + LABA+ pulmonary rehabitilization
D- SABA as needed + ICS

158) Pregnant women with 2 hx of preterm labour, presented with vaginal spotting
What to give her:
A-Estrogen

B-Progesterone

C-indomethacin

D-mg sulphate

159) 33 year old pregnant women presented with lower abdominal pain and
moderate vaginal bleeding,
K/c of BA, epilepsy and smoker

What is the highst risk factor for her condition

A- Age

B-smooking

C-BA

D-Epilepsy
160) Pt with intermenstraul bleeding

What is the most appropriate investigation

A
-
C
B
C
B
-
T
F
T

C-.B-HCG

D-US

161 )most common cause of dic

A- Abruptio placenta

B-placenta previa

C-uterine rupture

D-congenital blood disorder

162) 90 yr old female, htn and dm ,

Admitted to the hospital as a case of HF and was treated with medication,

There was incidintal finding of uterine procedentia, when taking hx from pt , she said it
has been for 10 year and never bothered her ,

Seh used to push her uterus back when urinate and pass stool

What is your management

A-vaginal pessary

B-vaginal hysterectomy

C-Radical hysterectomy

D-lap hysterectomy with sacro-spinous suspension

163 )Pt post op(explaratoy laparotomy) develop septic shock syndrome ,


What of the following is the most coomon physiologic responce

A-hypokalemia

B-resp acidosis

C-cellular anaerobic respiration

D-increase GFR

164) Lactating women present with right breast pain for 6 day

On examination , hot tender swelling lateral to the right aerola

Pt started to take floxacillin

Us/

Cystic lesion, thickened content , ddx could be galactocele, abcess, complicated


cyst for correlation What next

A-I&D

B-repeated aspiration

C-Exisional biopsy

D- I fogot...could be reassurance or antibiotic

165) 6 m boy presented with abnormal movement and yellowish discoloration for 1
week .

He is exclusively breast feeding

2 week ago his mother started to interduce fruit and fruit juice, since that , pt started
vomiting, irritable,

On examination:

Hepatospleenomegaly

Distended abdomin

Generalised jaundice

* Urine: positive reducing substances( normal is


negative) Dx?

A-Tyrosinemia

B-galactosemia

C-Alpha 1 anti trypsin def


D-conginital fructose intolerance

166) Pregnant women 32 weeks with vaginal bleeding:

A) Early postpartum bleeding


B) Late postpartum bleeding
C) Antepartum bleeding

167 )SE of lidocaine:

A) drowsiness
B) Nystagmus

168) autoimmune thyroiditis with thyroid lesion, what type of cancer is this lesion? A)
papillary B) follicular

C) aplastic
D) medullary

169) Reublla antibody negative during pregnancy ?

A- Answer is postpartum vaccine


B- No avoid exposure in choises
Follow up appointment and postpartum vaccine or vaccinate one month before
pregnancy

170) renal tubular acidosis with hypo k and hyper cl, what type?

A) 1
B) 2
C) 3
D) 4

172) Child with runy nose , nasal congestion , periorbital selling , watery eye,
turbinate swelling A-Allergic rhinitis
B-atopic dermatitis

C-Rhinitis medicamentosa

173) most recommended tool for checking prognosis after asthma attack?
A- Chest x-ray
B- ABG
C-Peak flow meter
174) Milestones of a child that can ride a tricycle but can t draw a
square? A-3

B-4

C- 5

175) Cervical cancer screening for a 35 year old woman:


Every 5 years Annually.

176) restlessness , disturbed sleep What's the diagnosis


A-Rhinitis

177) Female came with white breast discharge and high prolactin, what radiology you
will do? A- Pelvic MRI

B- Chest MRI
C- Sella turcica MRI

178) s male diabetic BMI: 41, diagnosed with BPH. What is the most important
risk factor? A- Age

B- Diabetes
C- Obesity
D- Hypergonadisim (something like that)

179) 10-year-old boy, diagnosed with TB. On rifampin. What is the side effect?

A- discoloration in body fluid


B- changing behavior C- decrease immunity

180 )Miscarriage in an old lady (~45yrs) she asked if her age had anything to do with her
miscarriage:

A- risk of miscarriage is 3% at this age


B- from 10 to 50%
C- 80%
D- no risk
Hacker and Moore
181) An 18 or 25 yo F. Worried about cervical cancer. She took her first dose of HPV
vaccine 3 months ago. What the best thing to do at this visit today?

A. Schedule app after 3 months


B. No need to do anything at this visit
C. Give 2nd dose at this visit
D. Repeat 1st dose

182) A pediatric patient came in RTA. Has splenic rupture and thus splenectomy was
done. What s an appropriate vaccine later on?

A- Pneumococcal
B- Meningococcal
C. MMR

183) 10 week GA pregnent lady with UTI witch of the following drug is contra-
indicated:

A- nitrofurantoin

B- ciprofloxacin
C- amoxicillin
D- Ceftriaxon

184- Screening for pregnant for asymptotic Bacteriuria according to U.S. Preventive
Services Task Force (USPSTF) :

A- 12 weeks
B- 20 weeks
C- 26 weeks
D- 32 weeks

185) camping in the community to educate people about health determents considerd
as? A-health education
B-active surveillance

186) pt with fever and chest pain and they mentioned an ECG finding they ask about
which part will be affected A- Pericardium
B- Myocardium

187) a 42 y.o male, with rectal bleeding, a biopsy from sigmoid showed adenoma, at
what interval shall you do colonoscopy screening?

A- 3 and 6 months
B- 3 years
B- 10 years C- No need

188) Case of stroke and complain of weakness in the arm leg and face, where s the
damage?
A. Middle cerebral artery

B. Anterior cerebral artery


C. Mid basilar artery

189) old patient came with subdural hematoma signs of lateralization imaging revealed
13 mm

shifting. his GCS 7/15 then was intubated and resuscitated what to
do next A- iv mannitol

B- admit ICU and observe


C- craniotomy
D- insert intracranial monitor

190) Old patient hx of HF with dialated cardiomyopathy and A.fib .. what is the most
appropriate management to control his heart rate? A- Digoxin

B- amlore

191) Pregnant lady everything was normal but she complain of itching and had
elevated liver enzyme, what is the dx? A- Viral hepatitis

B- Pregnancy cholestasis

192) Victim of RTA came with pelvic fracture and there s blood in the meatus, next step?
A- Folley catheter

B- Ureteroscope urethrogram
C- Pelvic CT

193)RTA pt with maltipule mandible fracuture with sever bleeding, unconscious, no


vitales mentioned

How would you mange his Airway?

A. Laryngial mask
B. Orotracheal
C. nasotracheal D.Cricothyrotomy

194) What of the following decrease the risk of preeclampsia?

A- Antibiotic
B- Aspirin
195) The time for introducing solid food for baby;

A- 14 months
B- 18 months
C- 22 months
D- 6 months

196) 18y girl married ask the Doctor for screening for PAP smear

A. At 21 y
B. When sexual active

197) This picture with a scenario of open wound racture, who is stable and already
received antibiotics, what do you want to do next? A- Closed reduction with above
knee cast

B- Closed reduction with below knee cast


C- Open reduction with intramedullary nail Answer 3

198) Pregnant known of a sickle cell anemia, you want to give her folic acid, what is the
right prescription for her?

A- 5 mg folic acid till 12 week


B- 5 mg folic acid till she gave birth
C- 5 μg folic acid till 12 weekg folic acid till 12 week
D- 5 μg folic acid till 12 weekg folic acid till she gave birth

199) Medical student came to you in the clinic wants you to give him sick leave to his
final exam tomorrow because he was with his sick mother in hospital last night and
couldn t study for the test, what is your action?

A- Refuse and tell him he was supposed to study earlier


B- Refer him to another colleague who will accept this
C- Give him a sick leave and save your future colleague
D- Call the hospital and make sure he s saying the truth and his mother is sleeping there.
200) Swimming ear case:

A- topical neomycin (cortisone otic (neomycin-polymyxin-hydrocortisone))

201) Percentage of developing GERD after gastric sleeve?

A- 5%
B- 10%
C- 20%
D- 30%
202) Pt with bilateral hydronephrosis with dilated bladder and proximal
urethra? A. Posterior urethral valve

B. Ureteropelvic occlusion

203) Patient claudication long case, then after full treatment with thrombolysis, patient
is risk for what while staying in hospital?

A. Stroke
B. PE

204) male came with multiple episodes of forceful vomiting, He has abdominal pain,
dyspnea, and Subcutaneous emphysema in the chest, what is the diagnosis?

A. Esophagitis
B. Perforated deudonal ulcer
C. Boerhaave syndrome

205) pt with MI discharged from the hospital then come back with facial and tongue
swelling, which drug is responsible

A) Acei

B) BB

C)
Statin

206) Man came from traveling presented with abdominal pain show hepatic mass,
Diagnosis A- amoeba

B- hydatid cyst

207) Man came from traveling

With fever, diagnosed with amoeba Management

A- Metronidazole
208) Pregnant lady during labor, CTG show fetal persistent bradycardia:

A- placenta insufficiency
B- congenital heart disease
209) New born seizure after delivery with hypoglycemia, management -
vit. K A- glucose

B- diazepam
C- antibiotic

209) Patient presented with confusion and loss of sensation only in lower limp:

A- MCA
B- ACA

210) 89-RT side Pleural effusion no shift of trachea and apex, hx of hemoptysis

A)Rupture esophagus

B)Heart heart failure

C) TB < bec no trachea shift + hx of hemoptysis

D)cancer obstructing ipsilateral bronchus

211) PT have HTN, heavy bleeding and anemia what contraceptive she can use?

A) tube ligation
B) IUD mirena
C) condom
D) OCP

212) Pt with high grade fever, tender cervical lymphadenopathy, exudate tonsils, next
mx?

A. Swab culture
B. Rapid antigen detection test

213) Lactose intolerance case, which of the following best indicates the diagnosis?

A. hydrogen test after giving lactose


B. upper endoscopy
214) 15 month old brought by his parents, they were concerned about his development
that only babbles and cant say 2-3 words yet, 2 weeks back he has normal auditory
examination, what should you do?

A. Reassurance and come 24months


B. Refer to ENT
C. normalvariantdevelopment
D. arrangeforaudiometry

215) pt dm2 on lifestyle modification what medication give ?

A-sulfonylureas

B- biguanides (metformin
C- ddp4 inhib
216) Victim of RTA came with pelvic fracture and theres blood in the meatus, next step?

A- Folley catheter
B- Ureteroscope urethrogram
C- Cant remember
Retrograde urothogram✅ first diagnostic test before catheter
217) Which of the following indicates compensated of shock?

A) Anuria
B) Confusion
C) Hypotension
D) pale peripheries

218) Patient did hernia surgery years ago. He has Cirrhosis with progressive ascites.
Hernia is present now with cough test. What to do:

A) Lap Hernia with Mesh


B) Control of ascites first

219) Which of the following indicates Worse prognosis of Schizophrenia

A) Onset at adolescence
B) Family Hx

220) old prim gravida woman just delivered spontaneously baby. is delivered
complete and intact. Massaging of the uterine is performed along with 20 units of
oxytocin in 1000 of lactated Ringers fast drip. inspection of the genital tract, there s
second degree laceration 2-cm left lateral vaginal wall, suturing is difficult because of
bleeding from above the site of laceration. a soft, boggy uterine fundus Blood
pressure 164/92 mmHg Heart rate 130 /min

Which of the following is the best step in management ?


Amboss

A .prostaglandin f 2
B. methylergonovine
C. manual exploration
D. oxytocin 10 units again

Oxytocin - > ergots if no HTN or prostaglandin analogues if there was HTN as in this case - >
uterine compression and massage-> b lynch - > embolization /catheters - >hysterectomy
: Obgyne consultant said the same thing
: The max dose of oxytocin won't help here
Especially that she is unstable
HTN so prostaglandin F2 is the next step

221) Camping in the community to decrease risk of hypertension is considered as? A)


Primary B) Primordial

C) Secondary
D) Tertiary Primary prevention

222) An 18 or 25 yo F. worried about cervical cancer. She took her first dose of HPV
vaccine 3 months ago. What s the best thing to do at this visit today?

A) Schedule appointment after 3 months


B) No need to do anything at this visit
C) Give 2nd dose at this visit D) Repeat 1st dose

223) Patient with sjogren syndrome which type of RTA: + Labs:

Shows low K

Normal Na

A) RTA I
B) RTA II
C) RTA III
D) RTA IV

224)Full term baby will be discharge from hospital what will you give him?

A) BCG, HBV
B) HBV, MMR

225) which type of RTA comes with Hyperkalemia? A) RTA type 1

B) RTA type 2
C) RTA type 3
D) RTA type 4

226) woman had a C-Section. What is the best way to prevent adhesions?
A. Perform the C-section pre-term NOT APPROVED
B. Closure of the peritoneum
C. Add an adhesion barrier before closing the wound

227) Correction of fluid I m hypernatremia is over:

A. Fluid deficit correction over 48 hours


B. Fluid deficit correction over 24 hours NOT APPROVED
C. Fluid deficit correction over 12 hours
D. Fluid deficit correction over 6 hours

228) Pt with 2 weeks history of watery diarrhea, vitals were stable What is the
expected acidbase abnormality? A. Metabolic acidosis

B. Metabolic alkalosis
C. Compensated metabolic acidosis (she is stable)
D. Compensated metabolic alkalosis

229)Confusion and neck stiffness, CT showing hypo lesion in frontotemporal:


A. Herpes encephalitis

230 )years old Pt came to ER with shortness of breath, wheezing and non-productive
cough by examination there s difficulty in breathing with wheeze all over the chest O2
sat is 90 ( didn t mentioned that he is known case of BA) What s the most important
management at this time ? A. epinephrine

B. inhaled steroid
C. inhaled b2 agonist D. iv mg

231) 66 Y.O Pt heavy smoker with symptoms of GERD, upper endoscopic examination
showed Medical student came to you in the clinic wants you to give him sick leave to his
final exam tomorrow because he was with his sick mother in hospital last night and
couldn t study for the test, what is your action?

A- Refuse and tell him he was supposed to study earlier NOT APPROVED
B-Refer him to another colleague who will accept this C- Give him a sick leave and save
your future colleague
D- Call the hospital and make sure he s saying the truth and his mother is sleeping there

232) squamous cell with high-grade dysplasia, what is the most appropriate
next step? A. Council him about smoking cessation. B. Refer for possible
esophagectomy

C. PPI and follow up after 6 months.


D. Follow up after 3 months.

My answer : according to current surgical there should be endoscopic mucosa resection over
6 weeks
233) Female patient complaining of pain before menses and resolved in the third day of
menses, how do you diagnose it?

A) Hysteroscopy
B) Abd US
C) clinical symptom

234) pt came with his shoulder flat, his arm adducted and internally rotated, what is the
diagnosis?

A) Anterior dislocation
B) Inferior dislocation
C) Posterior dislocation

235) Pt sustained injury to the left chest now there is sucking wound between 3rd and
4th ICS how will you manage : A) Chest tube

B) three sides dressing


C) Analgesics

236) Pregnant women 32 weeks complaining with vaginal bleeding:

A) Early postpartum bleeding


B) Late postpartum bleeding
C) Antepartum bleeding

237) Symptoms of Crohn's then ask which drug should you try it first to see Is it useful or
not?

A) Steroid
B) 5-aminosalicylic acid.
C) Azathioprine
D) Abx

238) 4 years old pediatric with recurrent UTI infection the doctor wants to do
investigation to see complications like renal scaring, Whats the investigation?

A) US
B) MCUS
C) DMSA
D) technetium 99 something

239)common SE of lidocaine

A) drowsiness
B) Nystagmus

240) 65-years old female, with Asymptomatic femoral hernia. What is the most
appropriate management?

A. Observation
B. Open repair with mesh
C. Laparoscopic repair (female)

241) What is the most common cause of DIC ?


A-placenta abruptio

B-placenta previa

C-congenital blood disorder

D-uterine rupture

242) High K what will you do?


A- Insulin
B- bicarbonate

243) PT with Hx of skin rash with discharge with is you action?

A-blood sample

B-no need for investigation

C- swab
244) Old age PT hx of DM, HTN, dyslipidemia
His ECG I.II, aVF (MI) what is the
highest risk factor for his condition?

A) AGE
B) DM
C) HTN

245) PT with only morning headache, N/V, nothing else… what investigation you will
order? A-brain MRI

246) calculate the parkland formula wight 70:


A-5L

B-2.5L

4ml x BSA% x weight (kg) (1/2 8 hrs ½ 16 hrs)

247) pediatric milestones


***hop in 2 legs = 4Y

***copy his dad behavior, draw his mam and dad with hand = 5Y

248) female HCV no hx of blood transfusion or illegal drugs, lab shows +ive HCV
antibody and ive RNA. what to do?
A- nothing

249) Hx of ectopic pregnancy , ask about the next pregnancy


management? A- early F/U with US

Ethics
250) You attended the surgery theater as an intern (some renal surgery) and the
surgeon was preparing to do the surgery on the right side but you knew from the
history that the diseases kidney is the left one. What should you do?
A-tell the surgeon ✅

251) Couples asking surrogate pregnancy?


A- F/U and read✅
252) Patient want to take hormonal therapy, but you are sure that it is not suitable for
her ?

A- refuse✅

253) to give him stronger pain killers you respect theirs wishes?

A- principles of double effects✅

254) A child with spinal muscular atrophy, needs intubation but the family refuse,
what to do? A- Intubate✅

255) Parents are believing that vaccines are bad for the child brain, what to do: A-
Explain to them✅

256) Female in her 50s asking about the most effective way to treat stress
incontinence A- Burch procedure
B- Pessary✅
C- Kelly plication

257)Most recommended vaccine during pregnancy

A- Influenza✅
B- dtap

258) Physiological changes in pregnancy that can lead to heart failure in patient with
mitral stenosis

A- increase minute ventilation


B- increase RBC mass
C- increase renal plasma flow

259) What is the time interval between ovulation and cleavage in dichorionic diamniotic
twin

A- 1 to 3 days✅

260) Patient developed circumferential right forearm burn. O2 sat in right digit is 86%
(I can t remember if they mentioned anything about the pulse) What is the
appropriate action.

A- bedside escharotomy
B- measure intra compartmental pressure
261) A long senario of a young female (in her 20s) she is medically free, now she has very
high liver enzymes 900+ no signs of chronic liver disease and vitally stable. What is the
best action? A- order PT and INR

B- IV dextrose
C- refer for liver transplantation✅

262) Child with bilateral cataract

A- galactocemia✅

263) What study design we can use to assess the burden of hypertention in Saudi
Arabia A- Ecological study

B- Observational
C- RCT
D- population survey

264) 3 hours newborn with low birth weight , had episodes of seizure ( looks like a
kernicterue case ) asking about management?

Lab( low RBC , low glucose and other things I can t remember)

A- bolus glucose ✅
B- normal saline
C- steroids

265) X-ray : obese child with minor trauma presented with left pelvic pain and inability
to bear weight ?

A- slipped capital femoral epiphysis✅


B- Neck of femur fracture

267) Hypothyroid pt will go for surgery what you will do ?

A- give thyroxine and procede


B- give thyroxine after surgery Until be euthyroid ***
C-delay surgery until Euthyroid
268) When to do urine test for pregnant women:

A) 12✅
B) 15
C) 20
D) 24

269) Tumour lysis syndrome:

A) Hypocalcemia hypokalemia
B) Hypocalcemia hyperkalemia

270) Patient post hernia repair, when can she get pregnant:

A) 3
B) 6
C) 9
D) 12✅

271)Pt k/c of systemic sclerosis, presented with renal symptoms, what is the ttt?

A) ACEI✅
272)4 years old pediatric with recurrent UTI infection the doctor wants to do
investigation to see complications like renal scaring, Whats the investigation?

A) US
B) MCUS
C) DMSA✅
D) technetium 99 something

273) Target of hba1c in diabetic patient:

A- <7
B- < 6.5✅
C- < 6
D- < 5

274) Comman cause of polyhidroamnios?

A- DM
B- Rh isoimmunization
C- renal agenisis

275) Acute COPD exacerbation what to give:

A- iv steroids
B- saba
C- saba &sama
We start with saba Sama and oral glucocorticoids. Iv steroids are used in patients with
severe exacerbations.
276) Ctg pic of prolonged deceleration what s the cause:
A- magnesium
B- oxytocin
C- epidural

277) 80 yrs women with pelvic organ prolapse grade 3, what s the management?

A- passery
B- surgery (Sacrohysteropexy)
Start by conservative and if the conservative did not work or if the patient was unstable then
start with surgical.
278)When to do urine test for pregnant women:

A) 12
B) 15
C) 20
D) 24

279) Tumour lysis syndrome:

A) Hypocalcemia hypokalemia
B) Hypocalcemia hyperkalemia

278) Patient post hernia repair, when can she get pregnant:

A) 3 No
B) 6
C) 9
D) 12

A question about a pediatric patient who ingested acetaminophen every 4 hours and came
with jaundice and increased bilirubin and liver enzymes. He had a viral illness two weeks
back.
The answer would be viral induced jaundice not paracetamol toxicity.

31 year old pregnant 4 previous first trimester abortions What is the highest diagnostic?
A) Karyotype
B) Hysterctomy
C) Hsg
D) Pelvic us
Both should be done but karyotype is the most important here.

: Women deliver baby with Down syndrome and she wants to know about future pregnancy.
Which of the following is BEST choice of her?

A. Karyotype if infant

B. Karyotype of infant and mother.


C. U/S in next pregnancy

D. Aminocenthesis in next

: year-old male came after 12 month on war, he saw dead friends and remembers difficult situations, he
is low mode and poor sleep. What is the most appropriate treatment?

A. Sertraline

B. Lorazepam C. Amitriptyline D. Lithium

I had many questions about asthma and COPD on when to intubate and when to use non-invasive

Patient has heavy bleeding and usually has to change tampon 15-30 times/day, recently she has been
engaged in intercourse with multiple partners to conceive, on examination: retroverted uterus,
decreased uterine mobility, and tender uterosacral ligament with nodularity. What’s the diagnosis?

A. Endometriosis.

B. PID

Patient delivered her baby in the same day she was discovered in the morning round to be -o and the
father is +A, her fetus is A+ and was given anti-d (I think 3000 not sure of the does). What amount of
fetal blood will be protected?

- 60 ml of fetal blood

- 30 ml of fetal blood

- 15 ml of fetal blood

Patient with sore throat fatigue and shortness of breath and productive cough with whitish sputum,
sometimes there's bloody streaks.

X-ray: normal with no conselidation. What's the diagnosis?

Hereditary telangtesia

Good pasture syndrome

Pleural edema

Bronchitis

Symptoms of cystic fibrosis (they provided sweat chloride test higher than normal in the question) and
asked what is the most common complication?
Bronchectasis

Empyema

17 year old female, medically free, gymnast in her cass, developed breasts later and never

menstruated, on examination she is tanner stage 5 but no menstruation, diagnosis?

Hypothalamic hypogonadism

Gonadal aeenesis

Imporferated hymen

WHAT is the best indicator of the severity of post-partum hemorrhage!?

Hct

Hg

Amount of blood loss (Hacker and Moore)

Pt presented with jaundice fatigue intermittent fever hx of traveling to India?

hydatid cyst

amoebic cyst of liver

Drug abuser agitated and has palpitation for a couple of days, what’s the cause of his symptoms:

1-Cocain withdrawal

2-Amphetamine toxicity

Patient with sjogren syndrome which type of RTA:

+ Labs:

Shows low K

Normal Na

A) RTA |

B) RTA II

C) RTA III
D) RTA IV

6 years old child who’s toilet trained and complaint of bedwetting recently with labs

UTI

Child presented to the ER with his parents as they were saying that their child ingested potentially toxic
disease of some medication. What is the next step in management?

A- Activated charcoal

B- Gastric lavage

C- Ipecac syrup

35 y/o female presented with chest pain and shortness of breath, chest x-ray normal and O2 sat were
88% PFT done for her all were normal except for DCOL it was 40% normal range is ( 60-120 ). What is the
Dx

A- PE

B- Pneumonia

C- Heart failure

D- Acute bronchitis

Child with sandpaper rash, what is the organism causing it?

Staphylococcus aureus

Group A streptococcus

Group B streptococcus

A very long case of SLE (they gave symptoms), ANA is high and they asked what is the confirmatory
test?

Anti-dsDNA

Anti-CCP
Anti-smith

Which lymph node will be affected in female patient have something in uterus and going for surgery?

intrailiac

Para aortic

Patient post-esophageal dilatation, came to the ER 8 hours after the operation complaining of chest pain
and change in voice?

Hemorrhage

Aspiration

Patient with hemorrhoids that is reduced on its own. Whats the grade?

Patient with previous history of PE, what's the recommended contraception method?

IUD

OCP
A 50 year old patient with umbilical abdominal pain, vomiting on examination there is increased high
pitch bowel sounds and abdominal distention labs show high amylase and ABC count. What is the most
likely diagnosis?

A) Pancreatitis
B) Small bowel obstruction
C) Mesentric occlusive ischemia

What is the mechanism of steroids in preterm labor?

Increases fetal lung maturity : hacker and Moore

A patient with post-partum hemorrhage who was resuscitated then they found that there is persistent
bleeding at several puncture sites what is the next important step?

A) Reversal of coagulopathy
B) Oxytocin
C) Prostaglandin

What is the best indicator of severity of post-partum hemorrhage?

A) Hematocrit
B) Hemoglobin
C) Amount of blood loss

Hacker and Moore

A patient at 32 weeks GA presented to the obstetrics clinic with cervical dilatation station – 3
effacement 70% and mild abdominal pain and contractions What is the next most appropriate step?

A) Call neonatoligist, tocolytics, observe


B) Call neonatoligist, antibiotics, steroids
C) Call neonatoligist, steroids, deliver
D) Call neonatoligist

Obgyne consultants approval

A case of dysmenorrhea and and menorrhagia


What is the condition that typically causes it?
A) Ovarian thecoma
B) Ovarian fibroma
C) Endometriosis
A case of dysmenorrhea and menorrhagia, what is the initial most appropriate step in this case?

A) Medroxyprogesterone
B) Oral contraception

A case of dysmenorrhea. What contraceptive should be started?

A) COC
B) Progestin pills
Obgyne consultant ‘S approval

A 2 month old infant who spits between meals and after meals. What is the diagnosis?

GERD
Physiological reflux

A 4 months old patient with watery diarrhea diarrhea what is the next step?
A) Give all
B) Delay dtap
C) Delay bcg
D) Delay MMR

A 6 month old patient with a recent infection and immunodeficiency. What is the next most
appropriate step?

A) Delay DTAP
B) Delay BCG
C) Delay MMR
D) Give All

A patient with a pelvic fracture who was resuscitated and is currentlu unstable BP 86/50 HR 110

What is the next step?

A) Surgical intervention
B) Pelvic binder

A 32 year old patient with mastalgia two days before menses. What is the nest step?

A) Breast US
B) Breast Mammogram
C) Pelvic US

A 13 year old patient with a normal uterine bleeding for four days every two weeks to three
months. What is the next step?
A) Reassurance
B) Oral contraceptives
A 38 GA mother who who was an Rh positive baby and an Rh negative mother. What is the next
step?
A) Amniocentesis
B) Emergent delivery
C) Reassurance

A mother coming for the first visit with an Rh positive father and a baby and a mother with an
unknown RH status. WHAT IS THE NEXT most appropriate STEP?

A) TEST THE MOTHER


B) Amniocentesis

A female patient came with a swelling under the public tubercle laterally.

A) Femoral
B) Direct Inguinal
C) Indirect Inguinal

A 25 year old last LMP was 8 weeks ago presented with severe abdominal pain and bleeding. What
is the diagnosis?

Threatened abortion

Ectopic pregnancy

Complete abortion

Nothing was mentioned about the cervix or passage of tissues or labs

A 30 year old in the GA 12 presented with passage of tissues and mild bleeding the cervix is closed .
What is the most likely diagnosis?

Complete abortion

Incomplete Abortion

Threatened Abortion

A patient in GA 8 with mild bleeding only. What is the type of abortion?

Threatened

Inevitable
Incomplete

A woman in GA 38 with PROM two weeks ago presented today with cervical dilatation fever severe
abdominal pain and high wbc count. What is the next step?

Antibiotics delivery

Antibiotics observe

Tocolytics observation

Chorioamionitis Antibiotics and prompt delivery

Light’s criteria : what indicates exudative effusion!?

A) LDH more than 2/3 upper limit


B) LDH more than 1/3 upper limit
C) PF/serum protein <0.5

A patient experienced shortness of breath 6 hours after total thyroidectomy. What is the next step?

Remove the suture at the ward

Tracheostomy

A patient one day after total thyroidectomy presented with respiratory distress and shortness of
breath. Examination shows a neck swelling. No investigations or information about the stability
were provided.What is the next step?

Remove the suture at the ward

Tracheostomy

A 2 year old presented with abdominal distention only. What is the next step?

AXR

US

A patient with joint pain who is a known case of SLE. What is the best initial treatment?

Hydroxycloroquine /Methotrexate

Hydroxyclorocquine /Azathioprine

A patient who is a known case of copd presented with spo 86 low po2 high pco2 and absent gag
reflex. What is the next most appropriate step?

Mechanical ventilation /intubation

Noninvasive ventilation

Oxygen therapy

Chronic smoker low dlco fev/fvc 0.6 what is the diagnosis?


Emphysema

Copd

Asthma

A patient who presented with signs of stroke and a CT shows subarachanoid hemorrhage. He was
resuscitated but he is still hypotensive. What is the next best step?

Craniotomy and evacuation

Iv mannitol

A patient presented with NSTEMI and depression in leads 2 3 and avf. He is vitally stable and he is on
aspirin enoxaparin beta locker and lisinopril. What would you add to his medications?

Streptokinase(not indicated)

Clopidogrel

Warfarin (not indicated)

Bronchial asthma pt presents with severe exacerbation, receiving steroid, salbutamol and
ipratropium nebulization, still in distress.

What to do?

Mg sulphate is the correct answer

: Hashimoto's thyroiditis was more likely to be detected around papillary cancer

[1/12, 5:33 PM] Azzah Smle: Small cummunity doctor should report which infection to infection
control?

Clamydia

Infectious mononucleosis

.‫انا جاوبتها كالميديا‬

: Pt male 50s, alcoholic, abd pain, palpable gallbladder, weight loss, scenario suggestive of
malignancy.

Liver enzymes acceptable except for significantly high ALP. Diagnosis?

A. Periampulary Tumor

B. Hepatocelluar CA
Answer A.

: Female pregnant, doctor suspecting PE, best diagnostic measure?

A. CTA

B. D DIMER

C. perfusion scan

Answer: perfusion scan due to pregnancy

One month boy came for vaccine. His older sister (6years old) had renal transplant and now she is

on immunosuppressive medication. Which vaccine is contraindicated for boy ?

A. MMR

B. Oral polio

C. Salk polio

D. Influenz

- Pregnant 34+1 came with *slow and continuous* bleeding , CTG for fetal is completely normal

Mother vitals is normal

What is the most important thing in the management?

- admission

- CS

- betamethamzon

- Fetal monitoring
A 6 year old chronic heart failure and dilated cardiomyopathy patient came to the ER with shortness of
breath and increased heart diameter and effusion on a cxr. He is not responding to medications.

What is the next most appropriate step?

A) Increase digoxin dose


‫‪B) Stat Iv furosemide‬‬

‫علَ ْي ِه تَ َو اك ْلتُ َوإِلَ ْي ِه أُنِ ُ‬


‫يب}‬ ‫{و َما تَ ْوفِي ِقي إِ اَّل بِ ا ِ‬
‫اَّلل ۚ َ‬ ‫َ‬
Which of the following nutrients fail to get absorbed in a patient with cystic fibrosis?

A. Vitamins A, D, E, and B complex


B. Vitamins A, B, E, and K
Deficiency of fat soluble
C. Vitamins B1, B2, B12 vitamins . AMBOSS
D. Vitamins A, D, E, and C
E. Vitamins A, D, E, and K

female complaining of abnormal uterine bleeding she has an endometrial polyp, on US


endometrial lining was 19mm, what will you offer to this patient at this stage?
Should consider age , premenopausal women with symptomatic polyp
A) open hysterectomy (AUB) > polypectomy .
B) laparoscopic hysterectomy
C) hysteroscopy with polypectomy

2 year old male with very painful micturition what is the most approppriate management
1. Abx for uti
2. Foley catheter
3. Cysoscopy and turp

Patient received 5mg warfarin his inr is 7 , known case of AF ?


A. Reduce to 2.5 mg Therapeutic range of
warfarin 2.0 to 3.0
B. Same dose
C. Stop warfarin
D. hold and repeat inr
‫ﻣﻠﻒ اﺑﺮﯾﻞ ‪٢٠٢٢‬م‬

‫ﻣﻦ ﻗﺮوﺑﺎت‬
‫‪Smle b‬‬
‫‪Fair‬‬
‫‪Asma‬‬
‫دﻋﻮاﺗﻜﻢ ﻟﺠﺪﺗﻲ وﺧﺎﻟﺘﻲ ﺑﺎﻟﺮﺣﻤﮫ‬
Pediatric

Child came with anal itching mother noticed rice like, whats the organism
responsible: Pinworm infection, enterobius commonly
A-enterobius vermicularis manifests as perianal pruritus . AMBOSS

child have dry non purulent conjunctivitis, cracked red lips, erythema (I think
trunk, sole, hand)
“CRASH and BURN”: Conjunctivitis, Rash, Adenopathy, Strawberry
what’s the diagnosis? tongue, Hands and feet, and BURN (fever ≥ 5 days) are the most
- Kawasaki disease common features of Kawasaki disease. AMBOSS
- Rubella
- Measles

Child with recurrent Sinopulmonary infection ard Gi infection His brother dead 6
morth due to sepsis What is the most likely diagrosis? Pediatric with recurrent sinopulmonary
infection and recurrent gastroenteritis which is
A- Iga deficiercy confirmed to be with giardiasis. His brother
B- X-linked chronic granulomatous disease died of sepsis?
A) X-linked agamm
B- selective Iga
‫ط ﻔ ﻞ ﻛ ﺎ ن ﻋ ﻨ ﺪ و از ﻣ ﺔ ﻣ ﻦ ا ﻟ ﻄ ﻔ ﻮ ﻟ ﺔ و ﻣ ﺎ ﯾ ﺤ ﺘ ﺎ ج ﻟ ﻠ ﻌ ﻼ ج ا ﻻ ﻧ ﺎ د ر و ﺟ ﺎ ي ﯾ ﺒ ﻲ ﯾﻔﺤ ﺺ ﻋ ﺸ ﺎن ا ﻟ ﻜ ﻮ ر ة و ش اﺳ ﺎﻟﮫ‬
‫ﺗ ﻜ ﺢ ﻣ ﻊ اﻻ ﻛ ﻞ‬
‫ﺗﻜ ﺢ وﻗﺖ ﻧﻮم‬
‫ﺗﺴ ﺘﺨ ﺪم ا ﻟ ﺪ و ا ء‬
‫ﻣ ﺮ ﺗ ﺎ ح ﺑ ﺪ و ن د و اء‬

‫ اذا ام ﺟﺎﺑﺖ طﻔﻠﮭﺎ ﻟﻠﺪﻛﺘﻮر ﺗﺴﺎل ﻋﻦ ﻣﺘﻰ ﺗﺴﻮي‬screening for iRon ‫وﻗﺎﻟﮭﺎ اﻟﺸﮭﺮ اﻟﺠﺎي ﻛﻢ ﯾﻜﻮن ﻋﻤﺮ‬
‫ط ﻔﻠﮭ ﺎ ھ ﺬ ه ا ﻟ ﺰ ﯾ ﺎ ر ة ؟‬
The American Academy of Pediatrics (AAP) recommends one-time
laboratory screening (Hb level) in all infants at the age of 9–12
9-12 month iron screening months. AMBOSS
A 2 month baby came with SOB and centropreprephreal cyanosis she was
diagnosis after birth but his parents forgot diagnosis what's likely diagnosis
A -TOF
B- ASD Acynotic heart disease
C- VSD
D- coarctation of aorta

Milestone:
Dress and undressed him self
Tight his shoes
Play with group
5 years old

Postnatal which shud b treated first?


A-autism
B-dip in sacral area
C-tof
D-undesented testis

Parents discover tonic clonic abnormal sizuer of their child during sleep
What is the Dd? Rolandic epilepsy, benign childhood epilepsy with centro-temporal spikes. Seizure type:
focal motor seizures with retain awareness, with possible 2ry GTC mostly accurs in sleep .
Nocturnal Sz . From my lectures
Mother concerned about her boy with nocturnal enuresis. Until what year it
consider normal? A) 5 B) 6 C) 7 D) 8 AMBOSS

Baby underwent forceps delivery and the physician noticed crushing the left
sternomastoid foramen what the baby expected to have

Left side cant close his eye Facial palsy ?


Lost of sensation in the anterior 2/3 of the toung

6yrs with inability to bear weight on exam there was bilateral knee pain.
Labs showed positive ANA. Juvenile onset arthritis is suspected what type
could he have? NB:Oligoarthritis JIA
AMBOSS

Pediatric with seizure and other signs, labs showed metabolic


acidosis, urine analysis: aminoaciduria and glycosuria, and
cataract what’s the diagnosis?
A. Galactosemia
B. Homocystinuria
C. Methylmalonic Academia
D. Maple syrup urine syndrome

Galactosemia..
this acute neonatal toxicity syndrome may include the development of sepsis,
cataracts, and even pseudotumor cerebri ( which may cause a bulging
of fontanelle).[6]

15 month boy presented with refusal of feeding vomiting febrile was suspected
to have meningitis so CSF was done showed normal glucose high protein high
cells
Mainly neutrophils
Culture showed gram positive diplococci
Which of the following is the most appropriate management :
A) ampicillin Streptococcus pneumoniae common in 1-24 months
B) ampicillin and gentamycin
C) Vancomycin
D) ceftriaxone and Vancomycin

agree with N notes but need better recall 3- 9 y/o pt his mom
complaining about he is wetting his bed since one month. He used to be
dry since 7years The urine is foul smelling His lab showing high wbs and
turbid in colour and showed nitrate Diagnosis ? A- UTI B- normal for his
age C- nocturnal enuresis

Pediatric patient diagnosed as croup in the ER, he was given the proper
management, but after 1 hour he did not improve. What is the
appropriate management? A-exposure to cool humidified air B-prednisilone
(with dose) C-dexamethasone (with dose) D-racemic epinephrine AMBOSS

A baby with noisy breathing and wheezing that improves when prone and
increases when supine. What is the diagnosis? A- Laryngomalacia B-
Tracheomalacia

1- child came with nasal congestion on examination and there is pale nasal
polyp and darkperiorbital swelling A- allergic rhinitis
B- rhinitis medicamentosa C- Acute bronchitis D- foreign body
Pediatric pt have all UTI Sx and ask what’s the most diagnostic value? A.
Two mixed organisms from clean catch B. Single organism from any
colony from mid stream urine sample C- Single organism (3x10^5) from
suprapubic
AMBOSS

6 years old boy, history of brief loss of awareness, EGG shows: 3-hz with spikes
wave.

What’s the diagnosis? 3hz spikes and wave= Absence seizure

Newborn, abdominal distention and palpable bilateral kidney.

Distended proximal urethra, ask about Dx? Posterior urethral valve

Tracheomalacia highst daognostic value?

Child with staccato cough_inspiration between cough and post tusive vomitiyn_
there is history of conjectivitis and eosinophilia in investigations what is the
diagnosis: Machine-gun like cough, chlamydia
1/pertussis trachomatis
2/ clamydia pneumonia
3/ mycoplasma pneumonia

Baby Preterm diliver (didn't specify CS or SVD). 4 hours later complaining from
SOB , Tachypnea, Tachycardia with grunting

Hyaline membrane disease is called also respiratory distress syndrome preterm


2- pulmonary "something" Pulmonary surfactant insufficiency
3- Meconium Aspiration syndrome post term so no
4- Transient Tachypnea of the New born full term so no

Mother brings her infant to family medicine clinic for regular check, the doctor
advice her to start iron supplement after 1 month from this visit, from this
scenario her child in which age now (in months)?
A.1
B.2

C.3
D.6
Child with sore throat..toxic with drooling..sitting upright with mouth open
What is the causative organism:
1/ hemophilus influenza type B epiglottis AMBOSS
2/ parainfluenza
3/ Rhinovirus
4/adenovirus

Parents bring their child with acute sever asthma


What counseling about bronchial asthma to tell:
1/ keep O2 humidity more than 50% wrong 30-50
2/cold wash..?
3/keep carpet in playing and eating room

Child point to red colour..ride tricycle..dress himself what is his age?? 4y/o

RF organism? group A β-hemolytic streptococcus

Pt bring her 7 yrs old child worried about short stature


There is delayed bone growth
What is management:
1/reassure he will gain adult stature with puberty
2/ do basic investigations cbc.TFT.bone age.

...Q: parents came with their 4 year old child who has not moved their right arm
for 2 days after falling on a stretched out arm. They claim on the first day they
saw no bruise or bleeding but thought its normal child not moving the arm after
falling. On the second day they got worried. On examination the child looked
well and healthy with no other injuries or signs of trauma. On x-ray both ulnar
and radial fractures. In the patient file you saw regular visits with no concerns
about the family?
A- Contact child services
B- contact police
C- treat the injury only (this option was worded differently but it suggested no
sign of child abuse so focus on injury)
D- i dont remember
Case of female child who lives in poor family with distended abdomen and
pitting edema there are lab results but I don’t remember them:
A - marasmus AMBOSS
B - Kwashiorkor

Child had type 1 diabetes and consulting the family regarding celiac screening ,
according to UpToDate its A .... "Asymptomatic
which of the following is true ? members of a high-risk group — For children at
increased risk for celiac disease (eg, those with Down
— A- Screen at diagnosis then every 5 years syndrome, type 1 diabetes, or with a first-degree
relative with celiac disease) who are asymptomatic, we
— B- Screen at diagnosis then every 2 years use a slightly different approach (algorithm 1). We
— C- Screen annually initiate screening around three years of age and repeat
— D- Screen at diagnosis then annually for 5 the screening every three to five years during
years
childhood."

~7 years child , systolic murmur heard in lower sternal area with no thrill, ( more
noticeable when supine)

ASD
VSD
AORTIC STENOSIS
STILL MURMUR

8 YO female presents to the clinic after finding of hilar lymphadenopathy


on CXR. She has on and off cough, but denies any fever, headache, n/v,
weight loss all negative. Labs insignificant except for X-ray shows
confirmed bilateral hilar lymphadenopathy CT guided biopsy shows
noncaseating granuloma What is the best next step in management
Sarcoidosis ,
AMBOSS
A-Observe
B-Start prednisolone sacrdosis
C-Start azithromycin
D-Start anti TB medication

Rocker bottom foot, prominent occipital, waiting for genetic testing, what’s the
possible syndrome?

- Edward Syndrome AMBOSS

3 y old child diagnosed with malaria


What is type of malaria?
A) P falciparum,
B)P vivax,
C)P ovale
D) P malariae

1New mom bring her her (2month) boy to neonatal well-being clinc, mother
noticed the baby had inter-meal spitting of milk he is cow milk formula
Abdominal examination normal
Growth parmeter normal
Next appropriate mangment:
-Pyloric ultrasound
-Reassuring and educate about reflexes
- change formula

Newborn with hypoglycemia and hypotonia , what is the intial management ? :


A-Iv fluid
B-Intubation
C-Dextrose

Child with Sx of tracheomalacia


How to confirm the diagnosis?
A) chest XR
B) bronchoscopy
C) Fluoroscopy

pediatric with chronic diarrhea and recurrent streptococcal AOM and


streptococcal pneumonia what to order :
1- C1 inhibitor
2- Immunoglubulin level
2- Anti hemolytic CH 50

A child starts to develop an awareness for strangers 6 month and separation


anxiety is 9 month . How old is the child in months?
a. 6 months
b. 7 months
c. 12 months
d. 24 months
Pt Dx grand mall seizure from child hood on NA valoprate develop. Pt c/o
generalized tonic clobic seizure which drug IV use in management???
1.phenytion
2.diazepam
3.phenobarbitone
4. Not remember

Introduction of soild food to children? 6 month

5 years old girl presents to the Emergency Room with a history of severe
abdominal pain, which is central and colicky, and repeated vomiting. The
pain is getting worse by movement. One examination, there is
generalized guarding in abdomen, with more tenderness toward the right
iliac fossa , Tachycardia + WBcs= 15 + T=38 + ESR = 12 Which of the
following is the most appropriate counseling regarding this condition ?

A. Occurs 20% individual x


B. it's uncommon below 3 years old
Intussupted
segment C. Sausage shaped mass often palpable
D. Passage of redcurrant jelly stool confirmed the diagnos

Child with unilateral wheezing ???

Foreign body

Pediatric age pt presented with his mom to ED she said that her child ingested
large amount of paracetamol (i think) the pt was healthy no signs of any toxicity
on presentation, they observed the pt for 4 hours with no any change in health
no signs and symptoms of toxicity, the mom then mentioned that she found
some of the drug was spilled on his clothes and in the floor, what is next?
A. Reassure
B. Watch for another 4 hours
C. Give antidote

Mother brought her 9yrs child for short stature Physical exam was
normal GH normal FSH normal TSH NORMAL T3-4normal Insulin like growth
factor decreased X ray: bone age of 6yrs Dx?

Familial short stature


GH deficincy
Constitutional growth delay

4 month milestone:
A. Sit without support
B. Head control
C. Start trying to crawl
D. Follow objects ects

g scenarios about a child 2 years old An upper RTI followed by symptoms of


with SOB and Expiratory wheeze and other respiratory distress and wheezing in a child < 2
years of age should prompt evaluation for
respiratory symptoms he had a viral illness bronchiolitis.
2 days ago
A. Respiratory monitoring for hypoxia
B. Inhaled epinephrine and steroids
Recent guidelines recommend using such
therapies mainly in sever cases
C. Intubation and antibiotics
D. SABA
Pt with hepatitis C and cirrhos

7 months old with fever, inspiratory crackles and respiratory crepitus. What does
he have?
Asthma
Bronchiolitis
Pneumonioa

Child with strep throat, has a brother 2 years old without symptoms. What to
do?
Strep antigen
Observe
Antibiotics

Child 6 month complain of drooling of saliva ... surgery done hiatal repair...
Most complication of this opration:::
1..GASTRIOSPHAGEAL REFLUX
2:PHERNIC NERVE INJURY
3;

6years loud s2 bluish skin.sob.???


1.aortic stenosis
2.tof
3.vsd

Mother concerned about her child how have abnormal face and mouth
movements during sleep
mid temporal spikes in ecg? ? Benign Rolandic epilepsy
Shows centero temporal spike
Lecture
Pediatrics patient with abnormal movement, tea coloured urine, odeama around
eyelid, HTN, low C3, mother mentioned that child has preceding sling infection

1-PSGN Post streptococcal glomeruloniphritis ,


nephritic syndrome with hematuria. AMBOSS
2-HUS
3-IgA

Child with joint pain and rash


Urine ..red cell Dx ..
Sle
AMBOS
RA
Lupus nephritis

3day infant develop rash erythromatus on red base all inv..normal ???1refer to
dermatology
2.reasure and this is not a serious condition can resolve
3.skin biopsy

Pedia come withher parent after seizure Hx of cough .nasal congestion flu like
symptoms temp38.5 Febrile
Next step ??
Phyniton
Ampiciln
Paracetmol

Kid with cough coryza conjunctivitis macuopapular Rash that started on face
then to hand and feet?

A Gonoccocal bactremia
B Rubella
C Toxic shock syndrome AMBOSS
D Rotair syndrome’s
• Measles ttt: Symptomatic treatment [4]

• Vitamin A supplementation

...
• Child throws ball and draws straight lines what age?
• 9 months
• 15 months
• 18 months
2 years

Child able to run, dress and undress himself, tell stories, tie his shoes:
A- 5 yrs
B- 4 yrs

New born after ventose delivery swelling limited to suture

A. Cephalohematoma
Lecture
B. Carbuncle

Which of the following is the highest risk factor for


cerebral palsy
• A-Hypoxia AMBOSS
• B-Preterm
• C-Gestational diabetes
• D-Neonatal sepsis

9 m old child brought by his family to the ER complain of cough that is barky
(like seal), the family is woring about there son, audiable sound was heard, what
you will hear in the pulmonary ascultation?

- expiratory wheez with proloned expirarory phase


AMBOSS
- decreaes bilateral air entery
- increase inspiratory phase due to mucus plug within the large bronchus
- increase inspiratory ohase due to subglottis narrowing

4 years. old girl with decrease head growth, decrease social interaction,
decrease in language , what is the closest ddx?

— Autisim
— mental retardation AMBOSS
— Rett's disorder
— aSPERGER syndrome
A female brought her baby to the well baby clinic ,he is still on formula /breast
fed the doctor advised her to give the baby iron supplement after the
appointment
How old is the infant now?

A- 1 month
B- 2 months
C- 3 months
D- 6 months

Child 4 years old with asymptomatic umbilical hernia what's the treatment?

A. Reassurance AMBOSS
B. wait until 5 years
C. Surgical repair

At what age baby talk 6-10 word and recognize two body part

A-17 month
B-19month
C-24 month
D-12month
15-year-old girl has sparse pubic hair not reaching the mons pubis, breast
budding with areolar enlargement, but with no clear distinction between breast
and areola. No contour of breast was seen. What Tanner stage is she?

a. Tanner I
b. Tanner II
c. Tanner III
d. Tanner IV

When does bedwetting becomes alarming?

A. 5 years AMBOSS
B. 6 years
C. 7 years
D. 8 years

which vaccine can be given in immune deficient?

A. Dtap
B. MMR
C. Polio
2 y/o pt on chemo for ALL, contacted a chicken pox pt, what to do?

A. Acyclovir 7days
B. Give vaccine now
C. Stop chemo
D. Give varicella zoster Immunoglobulin

Child came with seizure, diarrhea with blood & mucus. What is the causative
organism?
- Salmonella
- campylobacter
- Shegilla

A child presenting with croup and received the usual treatment with no
improvement. What is the important investigation to be done?

A- Lateral neck X- ray Steeple sign


B- Chest inspiration and expiration X-ray
C- Visualization by laryngoscopy
D- Chest CT
Patient diagnosed as tracheomalacia , what is the best diagnostic tool ?

A- Bronchoscopy
B- CT chest
C- X- ray

A child with widening of his joints , Ca 2.2 (normal 2.2- 2.7) P 2.1 (1.3- 2.3),
alkaline phosphatase > 1000 very high , No PTH is given. What is the most likely
diagnosis ?

A- Hypophosphatasia
B- Renal osteodystrophy
C- Vit D deficiency rickets
D- Familial hypophasphatemic rickets

Mother strictly vegan and she is breastfeeding her baby asking what deficiency
baby will have?

A- Vit B12
B- VitD
C- iron
D- vit A

infant case of a child drooling, unstable ?

A. Ct chest Epiglottis
B. X ray
C. Laryngoscope in OR
OBGYN
Patient on oxytocin, epidural and MgS04, preeclampsia. Her CTG: absence
variability (or non-reactive). What's the cause?

A. MgS04 toxicity Wafa


B. Oxytocin
C. Epiduralanalgesia

!Note to Remember
- Magnesium toxicity -> Deep tendon reflexes are lost, respiratory depression,
and cardiac arrest
- When toxicity occurs? The infusion should be stopped emergency correction
with calcium gluconate

Bacterial vaginosis scenario with fishy smell no itching What cells will you see?
A- Granular epithelial cells B- Atypical cells
Clue cells ( one of four As el criteria for
C- Budding yeast Dx of bacterial vaginosis, AMBOSS

Whiff’s test positive. What is your diagnosis


A. Candidiasis
AMBOSS
B. Trichomoniasis
C. Bacterial vaginosis and one other option

100-A pregnant lady complains of vaginal bleeding, shes a confirmed case of


low lying placenta, no
active bleeding now and no pain. Next step in management?

A- Ultrasound
B- Biophysical profile
C- CTG
D- Emergency delivery

GA 39 weeks having prom of 24 hours..ctg normal nd no contraction not in


labour what to do??
A-c/s
B-iol induc of labour
C-observation
D-augmentation of labour
A postmenopausal female complaining of vaginal itching and irritation. There is
vaginal discharge
which she described as watery and odorless. On examination, there were
multiple scratches, and
it had a scaly appearance, and it bleeds with touch. What is your diagnosis?
A) Trichomoniasis
B) Atrophic vaginitis
C) Candida

CTG shows transient episode of bradycardia more than 12 min (prolonged


deceleration)

with good variability =


category 2,

Causes of bradycardia:
Idiopathic
Arrhythmia
Drugs
Hypothermia
Cord compression

patient presenting with severe bleeding in 9th week of pregnancy, Os is open,


doctor saw some tissue on the cervix. What is your management?
A-Expectant management
B-D&C
C-Oxytocin

!
- D&C is indicated if the patient has bleeding!
- I would go for expectant if she’s not bleeding!
1. Pt vith dysmenorrhea what is the important to ask?
A) Family hx
(B)MehstrualI HE
(C)Medical hx

(D)Surgical hx

Pt known HTN and DM found out she's pregnant what is your management?
A) folic acid supplements.
B) ANC labs.
C) US.
D) review her medications

Pregnant woman found to be non-immune to rubella, when to give


vaccine? 3rd trimester After delivery MMR live vaccine
contraindicated in pregnancy.

Pt pregnant on 20 week known case of SLE controlled on Plaquenil what to do

Refer to rheumatologist to stop medication


Stop medication
Continue medication Hydroxychloroquine is not teratogenic

Woman with hx of miscarriage, now in 7 weeks with vaginal spotting, no


hx of passingtissue , on examination os is closed and no bleeding A.
Threatened abortion B. Normal delivery C. Incomplete abortion D. Inevitable
abortion

Patient came with vaginal discharge she have done CS with episiotomy
10 days ago , the obstetrician diagnosed her with UTI and described Abx
, but she did not improve then she went to another obstetrician and he
found infected vaginal swab , What is the medical error done by the first
obstetrician? A. let the midwife assist him and depend on her B. doctor
failed to follow the surgical safety protocol in the OR C. no
communication between the second and first obstetrician D. genuine
differente assessment or diagnosis of patient case

Dysmenorrhea, how to best diagnose endometriosis: A. laparoscopically B.


USC. MRI
Patient takes retinoic acid asking when she can get pregnant A- Tell her
to stop retinoic acid now and try to get pregnant B- She can get
pregnant after she finish the course C- get pregnant during the course no
problem D- She can get pregnant after 3 months of stopping retinoic acid
some reference say 1 month

Last mensural cycle was (?/?/2019) , what’s the estimated date of confinement
by Naegele rule? D+7/M-3/Yrs +1 or 0

What of the following will cause oligohydramnios?


A-Placental insufficiency AMBOSS
B- DM
C- Chriongioma
D-Dudenal atresia

Pregnant case of epilepsy for 2 years and poor control despite medication ,
what will you do?
- review the medication

What is the most common cause of bleeding in postmenopausal women ?


- endometrial atrophy
-endometrial cancer

Senario post deliver pph then ask about management of PPH:


1/ misoprostol
2/ propafol
3/ dexamethasone
No oxytocin in answers

according to Dr Wafa
Medication used in PPH:
1. Oxytocin
2. Methylergonovine (2nd line contraindicated in HTN and preeclampsia)
3. Carboprost (hemabate) PG F2a (3th line contraindicated in asthma)
4. Misoprostol

Younge female presented with suprapubic pain and heavy vaginal discharge
O/E tender fornices what diagnosis:
1/ cervicitis
2/ vaginitis
3/ endometritis

Female 15 days post c/s presented with suprapubic pain ,fever and vaginal
discharge O/E wound clean
Diagnosis:
1/ endomeritis
....

Younge patient complain of suprapubic pain and dysuria


No fever
Urine analysis given=
Urine colour yellow
Turbid
Leukocytes 10
Other normal
What is the next step:
1/ empirical antibiotics
2/ do culutre and wait for result
3/ no need for treatment
...

Mother 2 monthes post delivery .asymptomatic. culture screening of urine more


than100000 Ecoli sensitive to ciprofloxacin_nitrofurantoin anf trimethoprime
sulphamethoxazole
Ask about treatment:
1/ciprofloxacin
2/ nitrofurantoin
3/trimethoprim sulph...
4/ no need

14 years old female with infrequent vomiting during her period_came this time
with vomiting associated with smal amount of blood then symptoms releived.no
vomiting..soft non tender abdomin
Most apropriate next step:
Could be mallory-wiess tear due frequnt vomiting
1/Admit for observation
2/Discharge home and tell to come if symptoms recur
3/Prepare for urgent endoscopy
...

CTG sinusoidal pattern


Management??
Cord prolapse. Cs

Pregnant at 3rd trimester with asymptomatic bacteriuria:


-amoxicllin
-nitrofourantion Nitrofourntion increase risk of neonatal hemolysis

25 years old primigravida 20 weeks GA history of mitral stenosis due to


rheumatic fever
What physiological change makes her at high risk for heart failure?
- increase RBC mass
- increase stroke volume
- increase minute ventilation
- increase Renal plasma volume
Causes CVS defect
22 years female known case of severe depression ،suicidal attempts, she is
controlled on paroxetine Now, she is pregnant. What to do:
A. Stop paroxetine because of fetal malformation
B. Continue paroxetine and control her depression
-Not sure of other options
C. Switch to others drug i think Switch to sertraline or citalopram
D. Stop paroxiten b/c of prematurity

4- Polyhydramnios cause ?
A/ Anencephaly My lecture
B/ Post term pregnancy
C/ Maternal ingested NSAIDs
D/ Posterior urethral valve

case of PCOS asking about treatment.


Letrizole to pregnant Letrozole , ovulation induction
tamoxifin agen . My lectures

Progesterone
4th one was wrong

27 years old women come to the ER complain of mild vaginal bleeding, LMP
was before 9 w, Ex revelead soft uterus felt above the symphysis pubic, what is
the most appropriate next step?

- pelvic US to locat placenta


- pelvic MRI
- CT abdomen
- CT chest abdomen pelvic

35-year-old pregnant women, classified as low risk pregnancy prior, came today
with vaginal bleeding and abdominal pain of which she was diagnosed as
placenta abruption.. What is the next step?
A. continue same management.
B. classified her as high risk patients and do U/S.
……………

...
30 something female has amenorrhea for 7 months & want to conceive. test ?
LH ,FSH first
FSH/LH RATIO 3:1 means PCOS

Patient has complain about change of mood 3 days befor menstruation That
affects her
work and life
What is the case :
A- PMDD: , Premenstrual dysphoric disorder (PMDD)
B- PMS ?

Normal pregnant women at 12 weeks ,Indication of sever preeclampsia :


Increase cr
Increase urea
Increase Na level
No platelets in choice

Lady 29 week pregnant Came with bleeding and Vesicle Structure came out
what is your Dx ?
- partial mole
- complete mole
- threatened abortion
- missed abortion

Pregnant with candida?


Oral anti fungal
Topical anti fungal
Oral abx
Megablastc anemia if nurology symptom b if not A

which valvular heart disease has the worst outcome in pregnancy?

A)Mitral regurgitation
B)Mitral stenosis UTD
C)Aortic stenosis
D)Aortic Regurgitation

female patient came to the clinic complaining of a mass on a vagina she has a
history of
repeated unprotected intercourse with multiple partners, upon examination. she
has a wart in
the vagina, the causative agent is:
A- Herpes simples
B- Neisseria Gonorrhoea
C- Treponemma pallidum
D- Molluscum contagiosum

Female 27 years old, she is asymptomatic, her last pap smear was 3 years ago
and it showed unconcerned squamous cells. What is the most appropriate thing
to do?
A-Repeat pap with cytology
B-No need and reassure
C-Colposcopy
D-Cervical swab

-postmenopausal woman compaling of 1 year hx of recurrent vulvar itching


associated sometime with blood streak secretions , recently devolop pea size
mass in the labia?
Lecture
a) bartholin cyst
b )cystic adenosis
c) bartholin gland cancer
d) squamous cell cancer of vulva

$pt with HCV ,, developed abd pain, CT done and show HCC which is 6cm =
A. Radiotherapy B. chemotherapy
C. excision

Pregnant woman about to deliver, past history significant for Asthma and E.Coli.
What to give during delivery?
Betamethasone
Ampecilline

What anti htn contraindication in preg??


Hydralazine
Labetlol
Captopril

Role of acyclovir in pregnancy


A decrease viral sheeding and infection duration
B decrease placental transmission
C decrease infection sevirty

After d and c uterine perforation occur risk of placenta accreta


A increase
B decrease
C no change
D unknown
Female 43 years want to do screening for cancer what best at her age
A Colonoscopy
B mamogram
C clonoscopy and mamogram

Lady inferty amonnorhea for 1year .do curettage because up normal utrine
bleeding what cause of infert???
1.Asherman Intrauterine adhesions (Asherman syndrome)
2.shehan Lectures
3.pco

117. The uterus is inverse after delivery of the placenta what is the site of
insertion to be like this shape?
A. Fundal
B. Anterior
C. Posterior
D. Lateral
1-Picture of female genital with multiple lesions all over, history of multiple
sexual partners

A- condlyma lata Could be warts due HPV


B- basal cell carcinoma
C- molluscum contagisum
D- cant rmmbr

2-Picture of female genital with ulcerated lesion on labia majora

A- syphilis
B- herpes simplex
C- bartholin gland abscess

3- picture of something, done in a female patient that has a history of bleeding

A- Hysterscopy
B- laproscopy
C- cystoscopy

Ectopic pregnancy
Bhcg
1st 1000
2nd 1200
3rd 800
Expectant?
What to do next ?

41 yrs old woman presents with postcoital bleeding her cycles are becoming
more heavy and irrigulae than usual, she has type 2 diabetes and BMI 38 us
showed endometrial thickness 18mm
What is the most appropriate next step in her management?
A) pelvic ct
B) hystroscopy
C) clinical endometrial sample

Urine culture in pregnancy ?

Pregnant came to the ER today with moderate bleeding. What's the inext step?
(Not mentioned if there’s Hx of placenta previa)

A- transfer to Us
B- insert 2 large IV cannula
C- Emergency delivery
D- pelvic examination

Which is most contraindication in pregnancy ?


- thyrodctomy
- radicoactve idoine
- beta blocker

Woman in delivery bleeding not stop, she wants to conceive in the future, which
structure you should ligate?
A. Uterosacral ligament
B. Externaliliacartery
C. Internaliliacartery D. Uterinevein

Pregnant, twins one cephalic and another is breech presentation, how to


deliver?
A- Cesarean section B- Normal delivery

Patient at 29 weeks, didn't feel fetal movement for 1 day, CTG was reactive,
Biophysical profile was 8. What to do next?
A. Steroid and repeat Biophysical profile after 24 hours
B. Repeat Biophysical profile at 1 week Lecture
C. IOL
D. Urgent CS

40 years old female, P4 with a history of tubal ligation 4 years ago after her last
CS delivery. Her last Menstrual period was 6weeks ago.
Presented to ER with vaginal spotting. NO abdominal pain
O/E: OS is closed. Tenderness and fullness in her abdomen in RLQ. What is the
best Initial investigation?
A- Pregnancy test
B- US
C-CT Abdomen

12 gestation with fundal hight 19 wks and bhcg270000 ( very high) most likely
dx:
‫ ﺟﺎ ﺳﺆال ﻣﺸﺎﺑﮫ ﻟﮫ ﺑﺲ ﺑﺴﯿﻨﺎرﯾﻮ ﻛﺎﻣﻞ و ﻓﯿﮫ‬bhcg ‫ ﻣﺮه ﻋﺎﻟﻲ واﻟـ‬fundal high ‫ﻋﺎﻟﻲ و ﯾﺴﺄل ﻧﻮع اﻟﺤﻤﻞ‬
Ectopic pregnancy
Molar pregnancy

A 39-week pregnant patient, history of caesarean section, due to breech


presentations, now she is in labor, with regular contractions 4 every 5 min,
cervix fully dilated, full effacement, station +3, What is your management?

A. Ventouse delivery
B. caesarian section
C. examine her after 2 hours

Primigravid come with labor for 4h Dilated 5 cm, effaced 80%, station +1 after
5h there is no change in cervix, and contraction occur every 3 min. and stay for
60 sec. What to do?

A. Instrument use
B. C/S
C. IV oxytocin
D. Wait for 2h
"!Note to remember
She is still 5 cm dilated (latent phase) no need for intervention she can stay in
the latent phase up to 20 hours.
When to choose IV oxytocin? -> if rupture of membrane is mentioned in the
question. To avoid infections, otherwise? NO INTERVENTION IN THE LATENT
PHASE AT ALL.

41YO P5 +3 presented to the clinic complaining of abnormal uterine bleeding


her Menstrual period is regular, associated with blood clots and pain that is not
relieved by analgesic she had previous myomectomy, she is a known case of
PCOS&her BMI is 40?

A-Adenomyosis
B-Endometriosis
C-Uterine fibroid
D-Endometrial hyperplasia

Normal vaginal delivery, Baby weight 4.2kg, Laceration which degree:

A-First
B-Second
C-Third
D-Forth
Female with Pelvic pain increases with menses. On examination: uterus normal
And there is uterosacral nodularity and tenderness dx?

A. Endometriosis Lecture
B. Adenomyosis
C. PID
D. Fibroids

Female with previous 2 preterm labour, now she is in 20 weeks of gestation and
her cervix opened 30 mm, what you will do?

A- Immediate cervical cerclage


B- Give tocolytic & wait
C- Strict bed rest
D- progesterone supplement
How to know fetal weight intrapartum in 37w?

— Femur length
— head circumflex
— Biparietal diameter
— Abdominal circumference
Patient with intermenstraul bleeding. What is the most appropriate investigation

A) CBC
B) TFT
C) B-HCG
D)US

"!Note to remember (ACOG)

Intermenstrual Bleeding is mainly caused by uterine and cervical etiologies.

Lets Exclude!!
-CBC would be initial
-TFH would be initial (Thyroid dysfuction causes oligo- or amenorrhea or
menorrhagia and NOT intermenstrual bleeding)
-B-HCG its intermenstrual bleeding (between menstruations!!)
-US is the primary imaging is the primary imaging test of the uterus for the
evaluation of AUB (ACOG)
Pregnant with warts in vulva ttt?

A. Cryotherapy
B. electrophoresis

A 30 weeks gestation , she has lower abdominal pain with no bleeding or


leaking if we suspect preterm labor , which of the following will confirm preterm
labor ?

A- History will be enough to confirm preterm labor


B- Presence of contractions in the CTG
C- Pelvic examination to see the cervix

22 years old female pregnant, with pregnancy test positive at home, came with
sever abdominal pain, in examination cervical is closed, there is fluid collection
(didn’t mention the site). Thin endometrium, intact both tubes. what is the
diagnosis?

A. Intact ectopic pregnancy


B. Ruptured ectopic pregnancy
C. Luteal phase
Severe abdominal pain and fluid collection indicates ruptured ectopic.
Intact tubes ?? we cant tell if the tubes are intact on US?? ‫ﺣﺮﻓﯿﺎ ﻣﺎﻋﺮف وش ﻗﺼﺪه ﻟﻤﺎ‬
‫ ﻗﺎل‬intact

Female 25 years old, presented to the ED with vaginal bleeding, nausea, and
right lower abdominal pain and tenderness. History of open appendectomy due
to perforated appendix. Her BP:90/50 HR:120 RR:25 What is the possible site of
bleeding
A. Cervix
B. Uterus
C. Fallopian tube Ectopic

Pregnant 8 weeks GA come to hospital with spontaneous abortion , and ask you
what is the most common cause of that leads to spontaneous abortion ?

A- Incomptant cervix
B- Chromosomal abnormalities
C- smoking
D- intersive exercise
A 9 weeks gestation, presented with minimal vaginal bleeding and abdominal
cramps on transvaginal ultrasound : crown rump length equals 9 weeks
gestation and fetal heart beat is positive. What is the most appropriate next step
?

A- Pelvic assessment
B- Admit for observation
C- IV progesterone
D- US for placental localization
INTERNAL MEDICINE
Patient came with right-sided pleural effusion and a positive history of
hemoptysis. There was no shift of the trachea nor the cardiac apex. What is the
diagnosis?
A. Rupture of the esophagus
B. Heart failure
C. Tuberculosis
D. Cancer obstructing the ipsilateral bronchus

Increase cardiac out put , increase... , decrease ... resistance what type of shock
A- cardiogenic
B- Neurogenic
C- Septic
D- Hypovolemic

Pt with hepatitis C and cirrhosis came to the ER drowsy with abdomen


distention, on exam abdomen is tense with lower bilateral limb edema, what is
the management?

1- perform paracentesis
2- start diuretic and spironolacton
3- TIPS

Patient after 2 days post mi complain of chest pain radiate to back with st
segment elevation in leads II, III, and AVF present now with a new holsystolic
murmur and bilateral basal crackles. Diagnosis?? Papillary muscle rupture

A- Right ventricular infarction

B- papillary muscle rupture MR


C-VSD
D-cardiac temponade
ld pt DM- HTn cardiac dis, long senario. And all treatment given and
stable on thrombolytic, What is the most common cause of death during
his hospital stay:
A. bleeding
B. stroke
C. PE
D.MI

Pt in ICU received 15 units of blood, now blood coming out from NGT, incision,
and cannula site
1. Transfusion reaction
2. Thrombocytopenia
3. Hypocalcemia
4. Von wilbrand

Old patient hx of HF with dialated cardiomyopathy and A.fib . what is the most
appropriate management to control his heart rate? A.propranolol
B.diltiazim/ verapaiml
C. Digoxin

- case of female audotry halocentation


And delusional
For 1 month after that she recover without treatment?

Brief psychotic episode. <1month


Schizophreniform >1monfh but less than 6month
Schizophrenia >6months

Case about baby ingest asprin large dose!! Toxicity Q what abnormality in acid
base balance
Respiratory alkalosis early effect
Metabolic acidosis late effect
Different bw SISS SEPSIS SEVER SEPSIS SEPTIC SHOCK

Definitiv Diagnossis of tb
Treatment of tb
Diff b/w tb & sarcoidosis

TB : Caseating granuloma / Treatment: AKT

Sarcoidosis : Non Caseating Granuloma/ Treatment: Steroids

What is the structure having abnormality in cardiac tamponad :


A) pericardum
2) mayocardium
3) coronary artery
4) endocardium

Pt post some cholangio intervention few hours later he devolps chills and rigor
only no fever mentioned
TWBs was 9 normal till 10
RR 18
HR 90
Bp 115/88
Temp :38 degree exactly
What is the most likely diagnosis :
A) sepsis
B) SIRS
C) bactermia
D) septic shock
IAm sure of the figures

14- RT leg pale & absence of peripheral pulse + irregular irregular radial pulse,
What is the Source of thrombus?
A/ RT atrium
B/ LT ventricle
C/ Thoracic Aorta
D/ Abdominal Aorta

They suspect hypovolemic AKI what you’ll see ?


A-BUN/Cr >20
B-FENA >2%
femal 72 year
C/o knee pain,morning stiffness, wake her at night
Diagnosis
Osteoarthritis
Osteoporosis
Rematoid

Female with bloating and GI symptoms. Started gluten free diet by herself
and now she is better and visiting the GP. What to do? A* Dietician
referral B* GI referral C* No need to continue on gluten free diet D* Stop
gluten free diet and then do celiac serology

Blood loss of 25%, what to expect to be decreased?


- RR
- Pulse pressure
- GCS scale
- Urine output
55 years old male, no history of dm or any other disease
Bp: 159/75 What will give him ?
1/ lisinopril
2/ amlodipine
3/ hydrochlorothiazide
4 / lisinopril + amlodipine

Child has fatigue and splenomegaly , Hb low,RBC low ,MCV low ,Retic normal
,Iron normal , What to replace in this case ? Erythrocyte
A Iron
B. B12
C. Folate
D. Erythrocytes

Most Causative organism in case of valvular replaced patient, vegetation seen?


Depends on duration in the Q ,
less 2M = staph epi ,
more than 2M = Staph aure

Long case: Most significant risk factor for MI?


-smoking
-age
-hypertension

Child-Pugh Score for Cirrhosis Mortality I forgot the case but the questions
about calculating the risk so study it

Pt 64 Male , HT , Diabetes, HF , history of stroke the CHADS2 is ?


A3
B4
C5

Risk factor
Points
Congestive heart failure
1
Hypertension
1
Age ≥ 75
2
Diabetes mellitus
1
Stroke, TIA, or thromboembolism history
See the definition of primary and primordial

Educate people about low sodium diet to prevent hypertension? A. PrimaryB.


Secondary C. Primordial

A 39 y/o Patient with symptoms of SLE, arthritis and malar rash, what
medication should be started?
• A- Hydroxychloroquine and Mycophenolate mofetil
• B- Hydroxychloroquine and Methotrexate
• C- Hydroxychloroquine and Steroid By dr Amer

• D-Methotrexate

Pt with K/C CKD , ... at the end asking what is Most common cause of death in
CKD?
Dr. Amer
A- CVD
B- hyperkalemia
C- Uremia
D- coagulation disorders

COPD patient came to clinic with bilateral lower limb edema and pulmonary
hypertension. O2 sat 86% ,PO2 8.6, PCO2 7.5, pH above normal range Which
of the
following the appropriate management to give for the patient now

A) Start oral furosemide


B) Start oxygen therapy
C) Prednisolone therapy

Meningococcal prophylaxis: Single dose of ciprofloxacine or 2 dose of


rifampicine Uqu

★SLE
● LABs
➔ Most sensitive lab > ANA
➔ Most specific lab > anti-dsDNA, Anti-smith is more specific than ant-dsDNA

anti-smith
● Management.
# ARTHRITIS :
➔ Arthritis & malar rash: hydroxychloroquine It depends on the scenario

➔ Extremely severe flare up of polyarthritis: IV methylprednisolone If SLE case yes

# RENAL Involvement
Dr.Amer >>

➔ SLE with mild nephritis: corticosteroids


➔ SLE with severe nephritis (rapidly
progressive, diffuse proliferative, or severe proteinuria and active urine
sediment) :
corticosteroid + cyclophosphamide
# CNS involvement:
➔ SLE with central nervous system manifestations (seizure, organic brain
syndrome or coma):
IV cyclophosphamide & IV methylprednisolone
➔ Methotrexate or azathioprine : steroid-sparing drugs
# Drugs induced lupus: (which diagnosed by Anti-Histone AB)
● Hydrazine
● Isoniazid
● Chlorpromazine ● Procainamide

65 y male heavy smoker medicaly and surgycally free


What screen for him Other recall

65 heavly smoker came for check up what screening inv to do:

A) DM
A. Osteoporosis
B. Colon cancer
C. Abdomina aortic aneurysm✅

B) HTN Smoking is associated more with vascular disease, and it’s one
of the criteria they take it into consideration when it comes to

C)AAA AAA screening.


If he’s not smoker then colon cancer is the answer

In order to eliminate dengue fever from saudi arabia with witch you should start
- central
- East
- West

⭐ Patient known hypertensive on lisinopril and amlodipine, recently diagnosed as


TB
What you will do
1.discountinue lisinopril bec stim cough
2.discontinue amlodipine

A 54 yo female medically free comes for routine checkup ,Her cardiac exam
revealed grade 4 pansystolic murmur heard best at the apex and radiated to the
axilla,she is asymptomatic and the rest of her exam is normal,best next step to
confirm the diagnosis?

A. TTE Amer
B. TEE

C. ASO TITER

D. CHEST X ray

⭐ Patient diabetic admitted to hospital on Metformin , which is best during


hospitalisation :
Metformin
Sliding scale insulin
Pre and post prandial insulin

70 y old male K/O HTN, DM, hyperlipidemia on madication ( lesinopril,


amlodipine, statin)
came to reguler check and by examination you find congestive heart with
bilaterly lung cripitation and mild LL edema
BB 155/ 85
HR 110
Spo2 94%
What drug to add to his medication?
A) spironolactone
B) fursemide
C) doxazosin
D) atenolol

A female presented with back pain and fever, she was given NSAIDs and
advised for bed rest at home, then she presented with inability to move her
lower limbs, O/E there was tenderness on the midback.
Spinal MRI: Diffuse densities on T6,
⭐⭐⭐
What is the most appropriate next step?
A- Reassurance
B- Specimen from T6 vertebra
C- Brucella titer
D- Bone marrow aspiration for culture
The diagnosis of vertebral osteomyelitis or discitis is established based on
positive culture obtained from image guided biopsy (via computed tomography
[CT] or fluoroscopic guidance) of the involved vertebra(e) and/or disc space
19 y male with history of headach for 5 days and neck stiffness for two
days With low grade fever for one month CSF analysis Turbid Glucose
normal range WBCs neut: 27 Lymph: 87 or 78 Protein not sure Diagnosis:

A bacterial meningitis
B viral meningioencephlitis
C TB meningitis
D fungal meningitis

Female pt with 2 months jaundice fatigue and pruiritis


Obese Medically free
High AST ALT ALP
Viral serology negative
+ANA
CT Liver
Plasma cell
Management :
Vit E and weigh loss
Methotrexate
Steroid prednisone

Aggressive behaviour, what is the cause ? A. low serotonin Google

B. high serotonin
C. low endorphin
D. high endorphin
Low serotonin and high testosterone.
Take note plese.

In poor sleep - low Gmma amino butyric acid (GABA)

⭐⭐⭐
45 y/o female complete mammogram screening when to do it again ?
-after 1 year
-after 2 years
-no need
-forgot but i think after 6 months

Guidline in Saudi 2 years but other 1 years


Pregnant lady 30 wk with letharg and fatiuqe tremor i think c/o neck swelling
Investigation
TST. 0.1 ONLY
US diffuse multi nodular swelling
Most appropriate management
A. FNA
B. RA iodine
C. Thyroid surgry
D. Anti thyroid drug

Patient hear aliens


Could be 4 ⭐ ⭐

‫ﯾﺴ ﻤ ﻊ أ ﺻ ﻮ ا ت ﻛ ﺎ ﺋ ﻨ ﺎ ت ﻓ ﻀ ﺎ ﺋ ﯿ ﮫ ﺗ ﺘ ﻜ ﻠ ﻢ ﻣ ﻌ ﮫ‬
Dx.
1.anxeity disorder
2. Severe deprresion
3. bearevmenbt disorder
4. severe bipolar +not remember

71-Patient diabetic with pneumonia came with parapneumonic effusion


what will made poor prognosis:

A- DM
B- high WBCs
C- pleural effusion
D- Age above 65
Cystic lesion, thickened content , ddx could be galactocele, abcess, complicated cyst for correlation
Same Q is solved by Thawabah course in
What next May 2021

1.I&D ✅ Also, according to Dr. Abdulmuhssen


If it is abscess I&D
If simple cyst aspiration
2.repeated aspiration If complicated cyst or complicated mass
needs biopsy.
3.Exisional biopsy
The Q is not clear
.4. I fogot...could be reassurance or antibiotic

...

19- 6 m boy presented with abnormal movement and yellowish discoloration for 1 week .

He is exclusively breast feeding

2 week ago his mother started to interduce fruit and fruit juice, since that , pt started vomiting, irritable,

On examination:

Hepatospleenomegaly

Distended abdomin

Generalised jaundice

* Urine: positive reducing substances( normal is negative)

Dx?

1.Tyrosinemia

2.galactosemia

3.Alpha 1 anti trypsin def

4.conginital fructose intolerance ✅

20-Sever Preeclampsia which will be significantly decrease

A-Uric acid

B-Plasma

( should be low platelet)

: 43-Meconium aspiration question?

Surfactant lavage
Solved bu Dr. Safdar :
If just Meconium aspiration alone then surfactant lavage.
If Meconium aspiration with pulmonary HTN then Nitric oxide is the answer.

nitric oxide ✅

44-mother with polyhydrominos what is the comp I chose cord prolapse.

‫ لقيته كامل‬، ‫السؤال ناقص‬

Pregnant 38 wk with polyhydramnios, presents with rupture of membrane and severe abdominal pain,
CTG show fetal bradycardia, whats the diagnosis :
See similar Qs
- vasa previa Not complete Q

If it is not painful >> cord prolapse


- cord prolapse✅ If it is painful >> placenta abruption,

Sometimes vaginal bleeding does not occur (concealed


- placental abruption abruptio placentae)

- can't remember

45-genatic disorder

Tall , thin , long hand feet , no mental retardation

- marfan ✅

- klinefilter

46- Acut Mx of megrain

I think there was triptan in the choices

47- fibrail sizer Mx . Control the tembrtur or give diazepam or phenytoin

Conservative management by acetaminophen


48-) risk factor for preeclampsia

Matrnal age

Gastional age

Multipl pregnancy

If it is just Miscarriage NOT Recurrent , according to UTD A,C and D are correct. And I think Maternal age is
the most correct answer.
49) risk for miscoreg.
If Recurrent Miscarriage the answer will A also

Maternal age

Gastional age

Pre existing conduction htn and

daiabitis

50) iron replacement in the next month what is the baby age 4 months.

All other options1-4 and 6 month ( no 5 month )

Choose 5 month

51) gun shot in left leg hematoma in medial aspect of the thaig what’s next step : ther was angiography
(second step) Not complete Q see another recall

Gun shot wound in the thigh, he has weak pulse in the affected
limb and there’s hematoma and he cant move and there’s

ABC, Direct pressure first paraesthesia


What to do ?
1- exploration in the Or
2- exploration now
3- angiography

52) lady with multipul abortions and throbovaclar event : anti phospholipids syndrome
Not complete..

53: when baby start to have social smail in weeks ? 6 week , 12 week

54) full term baby vacation befor discharge:


Another recall:
Bcg hib Full term delivered healthy baby
Vacation should give before discharge
HBV & BCG
HBV& DTAP
MMR & BCG
DTAP & BCG

Or just HepB ‫على حسب الخيارات الي تجي في السؤال واذا هم‬
‫ على جدول التطعيمات السعودي الجديد‬B‫ماشي‬
Hemochromatosis: bronze skin, typical
presentation 40s in men/
postmenopausal women, cirrhosis, DM

Wilson: Kayser-fleicher ring, liver dx sx,


neurological sx

55) on slit lamp greenish , brawenish ring , and hepatic abnormality : Wilson , hemochromatosis

See another recall

- 2 days old baby brought to the hospital because of reluctant to feed and seizure since last
56) infant with sizer and hypotonia : it seems like tenitus night.
On exam the baby weight is 3Kg, inactive, has a generlized increased tone. CSF: clear,
Glucose 3 mmol (normal 2.8-5), protein 0.22 (normal 0.22-0.33), cells 10 (normal up to 10).
Which of the following is the most likely diagnosis?
Tetanus ِA. neonatal sepsis
B. neonatal tetanus
C. Hypoxic ischemic encephalopathy
D. Pyogenic infection
57) strok provfilaxisis

Doac

58)PE. S/s and Mx and profilaxsis even in case of renal abnormalities


UFH in case of renal impairment. “Dr. Amer”

59) araund 3 Quastion Abut hemothorax and pneumothorax how to defrantiait and how to maneg
See this from Dr. Abdulmohsen course
Hemothorax - >thoractomy (hematoma in the pleural areas and parenchyma)

Tension pneumothorax - >×start needle decompression (signs of shock and ARDs)

Spontaneous,, underlying pathology such as emphysema so you start by chest tube

60) haughty raiding testes Mx : surgical See another recall

805-Teen came because of scrotal pain. On examination, the scrotum was


high riding, and tender to palpate.
exploration Next?
A. Explore
B. Elevate testes & IV Abx
C. Doppler
D. ?

Answer is : A
N.B ( indication of doppler is when the clinical findings are INCONCLUSIVE)

61) type of study to increase sample size ? Not clear Q ..

No choices given but it could be cross sectional

62) lead pip sing ( ulcerative or chrons ) cobblestone >> Chron’s


lead pipe >> UC

AMBOSS

63) pt post thyroidictomy and had bilatral vocal cord paralesis and after removing the intubation she
developed straidor and dytryurated : cricothydictomy , reentubat

: 1-pregnant lady has an epilepsy and smoke then she got premature labor asking about what was the
big risk factor

A-smoking ✅ AMBOSS

patient was extubated after thyroid surgery and


B-epilepsy immediately developed shortness of breath and
respiratory distress. Examination reveals that both vocal
cords are in a semi-closed position. How will you secure
the airway?
A. Cricothyroidotomy
B. Tracheostomy tube
C. Re-intubate the patient
2-what gestational age you

screen for gestational diabetes ?


It depends on the Q
( 24 - 28 ) ✅
Treatment of ACS initial you give :

3-treatment of inferior MI ? Aspirin + Streptokinase


ACEI
BB
High intensity statin
Next : heparin & streptokinase ✅ Heparin
Nitroglycerin

If asked you about the definitive for ST-Elevation MI go with PCI.


While best : PCI ✅ And If asked you about the Investigation in inferior MI >> RT side
ECG to R/O RT Venticular infraction.

Most important next inv : RT sided ECG Dr.Amer & UQU

ACS is Very IMP TOPIC


4-vaccination for 1 year old

( MMR , MCV4 , OPV , PCV ) ✅

5- child tell a short story and undresses himself, which age?

4 years ✅

51) gun shot in left leg hematoma in medial aspect of the thaih what’s next step :

Apply direct pressure first

ther was angiography

Actually I think we start imagining then angiography

52) lady with multipul abortions and thrombosis event :

anti phospholipids syndrome

53: when baby start to have social smail in weeks ?

6 week , 12 week

Amboss 2 month so 8 weeks

54) full term baby vacation befor discharge:


Hepatitis B only

BCG given at 6 m

55) on slit lamp greenish , brawenish ring , and hepatic abnormality :

A-Wilson ✅

hemochromatosis

Kayser–Fleischer ring (KF rings) are dark rings that appear to encircle the iris of eye

36- impitigo diagonsis and management

Honey crusted lesions Generally diagnosed based on clinical presentation

Caustive organism Staph aureus

Management abx

37- common cause of AOM


AOM : Coinfection with both bacterial and viral pathogens
Virus: RSV✅, if not there choose rhinovirus is common.

Virus : RSV

Bacterial :
S. pneumoniae
Bacteria: Streptococcus pneumoniae✅ Haemophilus influenzae
Moraxella catarrhalis

AMBOSS and Abdulmehsen.

38- GERD not responding to PPI what’s next? ‫هذا السؤال جانا كلنا السنة الي راحت وبصيغ مختلة واجوبة‬
‫مختلفة‬
So know the rule

If taking 20mg << so increase it to 40mg

If taking 40mg << so 24h monitoring PH

Other recall:

Elderly with GERD not respond to PPI what is the next step ?

A. manometry

B. 24h monitoring PH✅.

C. lifestyle modification.

D. nissen fundoplication D & C B‫هذاعليه جدل كبير من السنة الي راحت ب‬

Some Drs went with D as the pt already


has esophagitis and symptomatic.

Others went with C as :

needs double check


not clear q

Definition
Temporary displacement of the testicle in the inguinal canal by the
cremasteric reflex
The testis may be easily repositioned back into the scrotal pouch.
Treatment: No treatment is necessary.

AMBOSS

39-Retractile testicle is the answer,, long history but they will tell u that testes can be manipulated the
inguinal sac or something like this (other choices were in descended testes, torsion I think)

(A retractile testicle is a testicle that may move back and forth between the scrotum and the groin ✅,
Undescended testicles testes have not moved down into their proper place in the scrotum.
See different recallsb

A 2 year old child presented with fever jaundice and a (third thing).
Family history: his brother had neonatal jaundice and required multiple blood

40-mediterranean transfusion. They are from the Mediterranean.


temp 38
RBC 4
Hbg 9
Reticulocyte 3%
What is the drug contraindicated (paracetamol, ibuprofen, amoxicillin, aspirin) Which medication should be avoided in the management?
-Aspirin
-amoxicillin
-ibuprofin
-acetamenophin

Answer is Asprin, G6PD case


______________________________

recalls: Which drug is not used in medetrian fever What is the drug contraindicated in familial Mediterranean fever?

A)paracetamol
B) ibuprofen
A. Aspirin C) amoxicillin
D) aspirin

Answer is Amoxicillin.

B. Amoxicillin So amoxicillin is CI in Mediterranean fever


And Asprin is CI in G6PD

Answer is : B

A paent from the Mediterranean region with anemia and fever What medicaon should not be given?

A. aspirin Needs more info if G6PD then A

B. acetaminophen

C. ibuprofen
See other recall:

D. glucocorcoids primi came for antenatal visitis she twice had small for gestational age on
abdomianl examination which indicated IUGR. Which has the most highest
diagnostic valvue?

Answer is: A Non stress test


Fetal kick chart
Serial ultrasounds

31-Highest diagnostic test IUGR

The diagnosis of FGR is based on discrepancies between actual and expected sonographic biometric
measurements for a given gestational age. Traditionally, it has been defined as <10th✅

Sonographic parameters
Intrauterine growth restriction (IUGR) is defined as lower than normal fetal growth
1-Biparietal diameter characterized by an estimated fetal weight below the 10th percentile for a given
gestational age. AMBOSS

2-head circumference

3-abdominopelvic

4-femoral length

5-amniotic fluid volume

6-calculated fetal weight

7-umbilical circumfere is the most effective parameter for determining fetal weight because it is
reduced in IUGR!!!✅
32-healthy medically free came for vaccines

A) meningeococcal
B
B) pneumococcal✅ According to WHO
Shingles: recommended for >= 50 yr
Pneumococcal polysaccharide: >= 65, & <65 w/ certain health conditions
Pneumococcal conjugate: for all adult w/ weakened immune.

C) MMr

E) Hepatitis B

D) dtap

33-post splenectomy, what type of vaccine should be given and after how many weeks?
See other recall
33-Pneumococcal vaccine after two weeks✅ What is the optimal time after splenectomy to get the vaccine?
A -2 weeks.
B -3 weeks.
C -4 weeks.
D -6 weeks.
Answer is A

S. Pneoumonie ✅

Meningococcal

35-HSP when to vaccine again? Not clear q ! Maybe it asks about the HSP pt who takes steroid and when to give
vaccines ? I don’t know

American academy of pediatrics : no difference was found in children already on HSP who received
vaccinations on time

But MMr yellow fever typhoid influenza vaccines are known to trigger IgA vasculitis

I would defer vaccines till the resolution of the illness.

34-Prosthetic valve undergoing dental procedure should receive which antibiotic prophylaxis?
IE
Yes, For
Amoxiciliin ✅

26-Pt presented with jaundice fatigue intermioent fever hx of


traveling to India ?

.A-hydat liver cyst


Abdulmohsen

B- amoebic liver cyst ✅

.C-tuberculosis liver
27-Newborn delivered with meconium aspira,on he was assessed and
managed pt was suffering to breathe (Surfactant nor surfactant
were available) what is the next step?

A-NO ✅

43-Meconium aspiration question?

Surfactant lavage

nitric oxide ✅

44-mother with polyhydrominos what is the comp I chose cord prolapse.

‫ لقيته كامل‬، ‫السؤال ناقص‬

Pregnant 38 wk with polyhydramnios, presents with rupture of membrane and severe abdominal pain,
CTG show fetal bradycardia, whats the diagnosis :

- vasa previa

- cord prolapse✅

- placental abruption

- can't remember

45-genatic disorder

Tall , thin , long hand feet , no mental retardation

- marfan ✅

- klinefilter
46- Acut Mx of megrain

47- fibrail sizer Mx . Control the tembrtur or give diazepam or phenytoin

48-) risk factor for preeclampsia

Matrnal age

Gastional age

Multipl pregnancy

49) risk for miscoreg.

Maternal age

Gastional age

Pre existing conduction htn and

daiabitis

50) iron replacement in the next month what is the baby age

All other options1-4 and 6 month ( no 5 month )

51) gun shot in left leg hematoma in medial aspect of the thaig what’s next step : ther was angiography
52) lady with multipul abortions and throbovaclar event : anti phospholipids syndrome

53: when baby start to have social smail in weeks ? 6 week , 12 week

54) full term baby vacation befor discharge:

55) on slit lamp greenish , brawenish ring , and hepatic abnormality : Wilson , hemochromatosis

56) infant with sizer and hypotonia : it seems like tenitus

Antithrombotic therapy with antiplatelet or anticoagulant agents is an important part


of secondary stroke prevention for patients with ischemic stroke or transient
ischemic attack (TIA) .
57) strok provfilaxisis If it was non cardiogenic strike >> anti platelet ( Asprin or clopedogril)
If cardiogenic stroke >> Long-term anticoagulation with warfarin or a direct oral
anticoagulant (dabigatran, apixaban..)

UTD

58)PE. S/s and Mx and profilaxsis even in case of renal abnormalities

59) araund 3 Quastion Abut hemothorax and pneumothorax how to defrantiait and how to maneg

60) haughty raiding testes Mx : surgical

exploration

I found this on telegram

61) type of study to increase sample size

Cross sectional

62) lead pip sing ( ulcerative or chrons )


63) pt post thyroidictomy and had bilatral vocal cord paralesis and after removing the intubation she
developed straidor and dytryurated : cricothydictomy , reentubat

28- pregnant low hg refused before enter to cs consent for blood transfusion during cs she confused
and need to give her blood transfusion what do ‫؟‬ Other recalls

A pregnant lady tells the nurse not to transfuse blood, during surgery she had
sever bleeding and become hypotensive. What to do?

A- Ask her to consent before she confused and loss of conscience -Get blood and Transfuse
-Inform the ethics
-Take consent from her before she lose conciosness
-Take consent from husband
2- Give her blood transfusion without consent
Woman do a CS, in the oberation room she told the nurse that she refuse to
do a blood transfusion Whatever the reason, and during the operation she
3- consult the medical ethical commioee had a severe bleeding and getting fainting

A: Call her husband and take the consent from him


B: Apply a pressure on the bleeding source and contact with ethical
This is controversial Q, I would go with 3.. committee
C: Do as the she want
D: Take the consent from her now before she loses consciousness

?? pregnant woman in labour with fetal distress. You counselled her for surgery
but she is
refusing bluntly. What is the appropriate action?
A. Refer to another hospital,
B. Inform the hospital ethics committee,
C. Get consent from husband,
D. Allow vaginal delivery based on her wishes
Answer is: D
29-2y , uncontrolled asthma , foul smeling stool exit
I think they mean CF case

A-Cl test

B-Chest ct

C-U L endoscopy

D-Bronchscpic lavag...

??
Patient on tocolytic beta-mimetics (terbutaline) what is the most side effect?
30-toucoltic SE A-Oliguria
B-Palptation
C-Abdominal pain
D- Vaginal bleeding

According to ACOG Terbutaline side effects (beta-adrenergic receptor agonists)

Maternal: - Tachycardia, hypotension, tremor, palpitation, SOB, chest discomfort, pulmonary

A-Oliguria edema, hypokalemia, andhyperglycemia

Fetal: - Fetal tachycardia

Wafaa OB
B-Palptation ✅

C-Abdominal pain

D- vaginal bleeding

21-Forgot Y/o presents with left lower quadrant pain with nausea and vomiting ,
on exam there is rebound tenderness?

.A-diverticulitis Not clear


B- Ovarian torsion

In the exam will be more clear (diverticulitis present with fever leukocytes and LLQ pain) & ovarian
torsion ( Sudden unilateral lower abdomen or pelvic pain) I agree

22-Female broughther baby to the well baby clinic, the doctor


advised her baby on iron supplement 1 month after this appointment.
How old is the infant now?

A-1

B-2

C-3 ✅

D-6

23-Child post chemotherapy day 2 for leukemia, played with other


children and got infected with Varicella and he never
vaccinated before‫؟‬

A-Give vaccines now

B-Acyclovir

C-.Give varicella

D-IVIG ✅

24- Mother of Down syndrome ask about risk of having other


children dow?? 1%

(A woman who has a child with Down syndrome is more likely to have another child with the same
condition by 1 in 100.)
# Mother of Down syndrome ask about risk of having other children down?
Depend in mother age ✅
Not depend in father
Depend on child and parent genetic factor
*25- 2days old with low oral intake,low activities , fever, ketonuria, fever,
characteristics smell of urine
235-Pediatric with seizure and other signs, labs showed metabolic acidosis, urine analysis:
aminoaciduria and glycosuria, what’s the diagnosis?

.A-Maple Syrup Syndrome ✅ A. Galactosemia

B. Homocystinuria

C. Methylmalonic Academia
.B-phenylketoneuria D. Maple syrup urine syndrome v

46- Acut Mx of megrain “incomplete qs” ??

For All patients

Limit stimuli (i.e., light, loud noises) and activity.

Treat nausea/vomiting, if present.

1- IV fluids

2- Parenteral antiemetics like:

Metoclopramide or Prochlorperazine

#Acute headache:

Ibuprofen at standard doses is effective for acute migraine treatment.

Also Intravenous metoclopramide is recognized as an effective therapy for acute migraine.

#Mild to sever headache:

Start a migraine-specific agent :triptans (e.g., sumatriptan) or ergotamine (do not combine these
agents!)

47- fibrail s Mx . Control the tembrtur or give diazepam or phenytoin “incomplete qs” ???

Febrile Seizure:

It’s seizures Secondary to Fever and Infection outside CNS “not meningitis or encephalitis” , Happed at
age of 6month-6years , have 2 types:

1- simple febrile seizure:

Last 15 min - Generlaized- once per 24h

Can be discharged no need for EEG neither MRI


2- Complex febrile seizure:

Last 15 min - Focal - more than once per 24h

Need referral to neurologist to do EEG + MRI

#Tt :

No Seizure currently: Antipyrtics “Paracetamol”

Currently Seizure : Treat the seizure with :

* Treatment of choice: Lorazepam

* Alternative: diazepam

‫بس اذا ماعنده تشنجات االوليه تكون انك تعالج الفيفر ألنها هي‬, ‫اذا فيه طفل عنده تشنجات الحين ومعاها فيفر االوليه انك توقف التشنج‬
‫ السبب بـ‬Paracetamol.

#Indications of Lumber puncture in febrile seizure:

U do it to exclude meningitis for these people:

1- Pt with sign of meningitis

2-Pt less than 12-18 month of age espi if not immunized. “Routinely”

‫الن هذوال االطفال ماتوضح عليهم ساين المننجايتس‬.

48- risk factor for preeclampsia

A-Matrnal age ✅ If <20 or > 40 years.

B-Gastional age ❎

C-Multipl pregnancy ❎

#Other risk factors:

DM - Chronic HTN - BMI>30 -Multiple gestation “twins” - nulliparity.

49) Risk for miscarriage ?

A- Maternal age

B- Gastional age
C- Pre existing conduction HTN and DM ✅

#Other maternal risk factors:

Advanced age - Heavy smoking - Uterine repture

50) iron replacement in the next month what is the baby age now ?

All options

A- 1 month

B- 2 month

C- 3 month ✅

D- 6 month

( no 5 month )

There’s a depate about this qs, and there’s a alot of wrong answers chosing 6 month as a first month to
introduce iron supplement to infant which’s wrong, the righ answer is C

General recommendations:

* Full-term infants. Start giving your baby an iron supplement at age 4 months. Continue giving your
baby the supplement until he or she is eating two or more servings a day of iron-rich foods,

* Premature infants. Start giving your baby an iron supplement at age 2 weeks. Continue giving your
baby the supplement until age 1.

Questions from 46-50 were answered by

Dr Mohammed Adel.

157) Typic Copd case ((SYMPTOMES was not severe ))

The mMRC (Modified Medical Research Council) was 2

A- Combined LABA + ICS

B- Phosphodiesterase-4 (PDE₄) inhibitors + LABA + ICS

C- SABA as needed + LABA+ pulmonary rehabitilization

D- SABA as needed + ICS


1) Which of the following indicates Worse prognosis of Schizophrenia :

A) Onset at adolescence

B) Family Hx

2) Which of the following indicates compensated of shock?

A) Anuria

B) Confusion

C) Hypotension

D) pale peripheries

****Infant Mortality Rate has decreased to 4.28 in 2020 in Saudi Arabia.***


158-MOH reported the infant mortality rate in 2020 which was 4.81 .
3) Which of the following figures involved in This: Which of the following help in calculating this ratio

A-Knowing the childbearing women in the middle of 2020


B- Population percentage in Saudi Arabia
C-Knowing percent of those died before the first year of their life.
A) Number of women in childbirths age answer: C

B) Number of deaths in saudi arabia in 2020

C) children who died in 2020 and not completed their first year

4) Patient did hernia surgery years ago. He has Cirrhosis with progressive

ascites. Hernia is present now with cough test.


Answered by dr al Abeidi
What to do:

A) Lap Hernia with Mesh

B) Control of ascites first

5) Fetus with Cystic hygroma. What to do

A) Amniocentesis

B) Chrionic Villous Sampling

<15 weeks cvs

>15 weeks amniocentesis

6) Baby product of illegal pregnancy, abandoned by his mother. Pediatric

surgeon says baby have to do Hernia surgery. What to do

A) Inform the case to police

B) inform the case to social worker

C) inform the case to health ethics committee


D) Do the surgery without consent

1) Which of the following indicates Worse prognosis of


Schizophrenia :
A) Onset at adolescence
B) Family Hx

2) Which of the following indicates compensated of shock?


A) Anuria
B) Confusion
C) Hypotension
D) pale peripheries

****Infant Mortality Rate has decreased to 4.28 in 2020 in Saudi


Arabia.***

3) Which of the following figures involved in This:


A) Number of women in childbirths age
B) Number of deaths in saudi arabia in 2020
C) children who died in 2020 and not completed their first year

4) Patient did hernia surgery years ago. He has Cirrhosis with


progressive ascites. Hernia is present now with cough test.
What to do:
A) Lap Hernia with Mesh
B) Control of ascites first

5) Fetus with Cystic hygroma. What to do


A) Amniocentesis
B) Chrionic Villous Sampling

6) Baby product of illegal pregnancy, abandoned by his mother.


Pediatric surgeon says baby have to do Hernia surgery. What to do
A) Inform the case to police
B) inform the case to social worker
C) inform the case to health ethics committee
D) Do the surgery without consent

7) Pelvic Fracture with injury to membranous part of urethra.


Blood seen in external meatus. What is the most appropriate
action. 315-Pelvic Fracture with injury to membranous part of urethra. Blood seen in
external meatus. What is the most appropriate action.

A) Cystoscopy A) Cystoscopy
B) Folly’s Catheter
C) Suprapubic catheter

B) Folly s Catheter D) CT pelvis

Answer: C

C) Suprapubic catheter Abdulmohesen course

D) CT pelvis

8) Patient with Anterior chest trauma with bruising in sternum.


Patient vitally stable, clear cardiac and respiratory exam, except
for Pounding pulse. ECG: Arrhythmia
X-ray: Sternal Fracture.
Echo: Normal

What is the Diagnosis:


A) Pneumothorax
B) Cardiac contusion
C) Cardiac Tamponade
D) Ventricular rupture (not sure if this was the choice or not)
Answered by thawabah

9) Lymph Node swelling, and was clear lymphatic fluid in this


swelling. What
to do: 3 years old with 10 cm mass lateral neck
A) Surgery mass on aspiration ear

B) Sclerotherapy Bad recall lymphatic fluid what to do? Chemo✅

C) Radiotherapy Radiation D) chemotherapy Surgery


Sclerotherapy

(This question is not formulated wright, but this is main and whole
needed information in this question)
10) most recommended tool for checking prognosis after asthma
attack?
A- Chest x-ray
Which of the following is a recommended tool for checking
prognosis after asthma attack?

A Chest x-ray

B- ABG B ABG
C Pulse oximetry
D Peak flow rate

C- Pulse oximetry Answer is D by Amer

D- Peak flow rate

11)Female came with white breast discharge and high prolactin,


what radiology you will do?
Female came with white breast discharge and high prolactin, what radiology
you will do?
Pelvic MRI
Chest MRI

A- Pelvic MRI Pelvic MRI

Abeidies’s Answer ; D, Brain MRI

B- Chest MRI Other recall

Secondary amenorrhea , bilateral breast pain and viscous yellowish

C- Sella turcica MRI discharge without mass or lymph nodes


Prolactin high , next:
sella turcica imaging
CT pelvis and abdomen
TSH checking

12) 0 s male diabetic BMI: 41, diagnosed with BPH. What is the
most important risk factor?
A- Age 60 y/o male diabetic BMI: 41, diagnosed with BPH. What is the most
important

B- Diabetes
risk factor?
A A- Age
B B- Diabetes
C C- Obesity

C- Obesity D D- Hypergonadisim (something like that)

D- Hypergonadisim (something like that) Age due to androgen-estrogen imbalance; as men age, testosterone levels
decline, but estrogen level remains the same, which results in a higher
estrogen/ testosterone ratio
So, A✅
Source: amboss

13) 10-year-old boy, diagnosed with TB. On rifampin. What is the


side effect?
Other recall

A- discoloration in body fluid Pt with TB started treatment then presented with


hematuria in fx there’s urethral stones, in lab low platelet
and hemoglobin what’s the dx
A- discoloration in body A-urethral stone
B-Medication side effect✅
C-Hematological abnormality related to TB
D-Renal TB

Side effects of rifampin


B- changing behavior
C- decrease immunity

14) Miscarriage in an old lady (~45yrs) she asked if her age had
anything to do with her miscarriage:
A- risk of miscarriage is 3% at this age in some other resources B-
from 10 to 50% ‫موجودة في هاكر‬ Answered by dr Wafaa

C- 80% UpToDate
D- no risk

15) An 18 or 25 yo F. Worried about cervical cancer. She took her


first dose of HPV vaccine 3 months ago. What the best thing to do
at this visit today?
A. Schedule app after 3 months
B. No need to do anything at this visit
C. Give 2nd dose at this visit
D. Repeat 1st dose

16) A pediatric patient came in RTA. Has splenic rupture and thus
splenectomy was done.
What s an appropriate vaccine later on?
A. Pneumococcal
A. Meningococcal
C. MMR

17) 10 week GA pregnent lady with UTI witch of the following


drug is contraindicated: A- nitrofurantoin
B- ciprofloxacin
C- amoxicillin 17) 10 week GA pregnent lady with UTI $ witch of the
following drug is contra- indicated:

D- Ceftriaxon A- nitrofurantoin
B- ciprofloxacin
C- amoxicillin
D- Ceftriaxon

Answered by Amer B
18 ) Screening for pregnant for asymptotic Bacteriuria according
to U.S. Preventive Services Task Force (USPSTF) :
A- 12 weeks 12 week

B- 20 weeks
C- 26 weeks
D- 32 weeks

19) camping in the community to educate people about health


determents considerd as? A-health education
B-active surveillance

20) pt with fever and chest pain and they mentioned an ECG
finding they ask about which part will be affected
A- Pericardium
B- Myocardium

21) a 42 y.o male, with rectal bleeding, a biopsy from sigmoid


showed adenoma , at what interval shall you do colonoscopy
screening?
A- 3 and 6 months
B- 3 years
B- 10 years
C- No need
22) Case of stroke and complain of weakness in the arm leg and
face, where s the damage? Case of stroke ( patient has other co- morbidity ) and

A. Middle cerebral artery


complain of weakness in the arm leg
and face but the sensation is intact , where’s the
damage?
A. Middle cerebral artery

B. Anterior cerebral artery B. Anterior cerebral artery


C. Mid basilar

C. Mid basilar artery


Needs more informations , but more with A

24) old patient came with subdural hematoma with signs of 1-old patient came with subdural hematoma with
signs of lateralization imaging revealed

lateralization imaging revealed 13 mm shifting. his GCS 7/15 13 mm shifting. his GCS 7/15 then was intubated and
resuscitated what to do next
A- iv mannitol

then was intubated and resuscitated what to do next A- iv


B- admit ICU and observe
C- craniotomy
D- insert intracranial monitor

mannitol Next step you have to lower the ICP if definitive go


with craniotomy
Answer is : A , by Dr, Abdulmuhsen .. see %

B- admit ICU
and observe C-
craniotomy
D- insert intracranial monitor

25) Old patient hx of HF with dialated cardiomyopathy and A.fib ..


what is the most appropriate management to control his heart
rate? Old patient hx of HF with dialated cardiomyopathy and A.fib .
what is the most appropriate management to control his heart

A- Digoxin rate?

A.propranolol

B- amlore
B.diltiazim/ verapaiml
C. Digoxin

Answer is A, by Dr. Amer.

know that CCB is CI in every HF case and BB is CI when the


patient is in Decompensated HF. If he stable you can use BB.

26) Pregnant lady everything was normal but she complain of


itching and had elevated liver enzyme, what is the dx?
A- Viral hepatitis
B- Pregnancy cholestasis

It s incomplete Q

27) Victim of RTA came with pelvic fracture and there s blood in
the meatus, next step? A- Folley catheter
B- Ureteroscope urethrogram
C- Pelvic CT
28) RTA pt with maltipule mandpular fracuter with
sever bleeding, uncontious, no vitales menstioned How
would you mange his Airway?
A. Laryngial mask
B. Orotracheal Answer is D.Cricothyrotomy

C. nasotracheal
By thawabah and abdulmuhsen

A- D.Cricothyrotomy

29)What of the following decrease the risk of preeclampsia?


A- Antibiotic
B- Aspirin

30) The time for introducing solid food for baby;


A-
14
mon
ths
B-
18
mon
ths
C- 22 months
D- 6 months

31) Patient post thyroidectomy had arm spasm during blood


pressure measurement, what is your next step?
1-Give analgesia
2-Take blood pressure again Patient post thyroidectomy had
arm spasm during blood pressure
measurement,
what is your next step?
A A- Give analgesia
B B- Take blood pressure again
C C- Check calcium level

& #FAITH_2021_IM&
3-Check calcium level(this is due to hypocalcemia most likely ,
check the calcium and correct it then measure BP again after the
patient is not spasmodic in this case)
Child had Type1 diabetes and was consulting the
family regarding celiac screening, which of the
following is true:
A. Screen at diagnosis then every 5 years
32) Child had Type1 diabetes and was consulting the family
B. Screen at diagnosis then every 2 years
C. Screen annually regarding celiac screening, which of the following is true: A-
UpToDate
o Soon after diagnosis Screen at diagnosis then every 5 years B- Screen at diagnosis then
o If negative, then after 2 years
o Then every 5 years

Anyhow, answered by Safdar B


every 2 years
C- Screen annually

33) Milestones of a child that can ride a tricycle but can t draw
a square? A- 3
B- 4 Milestones of a child that can ride
a tricycle but can’t draw a
square?

C- 5 A-3 yo

Answer: A

34) Cervical cancer screening for a 35 year old woman:


A- Not indicated
B- Every 5 years
C- Annually

35) Menopausal woman complaining of insomnia and flushing:


Menopause women complain of insomnia and flushing
give?

A. Local hormonal replacement Ocp


Ssri

B- Systemic hormonal replacement


Local hormone replacement
V Systemic hormone replacement

36) Pneumonia case asking about the appropriate abx regimen:


A- Ceftriaxone + azithro By Amer

B- Vanco + azithro course

C- Meropenem + azithro

37) Vaccines for a 9 month old:

A- Measles - MCV ✅✅
Measles - MCV

38) Which of the following can cause an increase in BNP:


BNP

Increase with :
COPD
Renal dis
Advanced age
A.Feb

A- COPD Decrease with


Obesity

B- Furosemide Dr. Amer

C- Obesity

39) Which antipsychotic is associated with weight gain:


A- Olanzapin
UTD

40) Long case of a medullary thyroid cancer (diagnosis given)


what is the appropriate management?
Abdulmuhsen

A- Sub total thyroidectomy


B- Total thyroidectomy
C- Hemithyroidectomy

41) Case of a patient who had itchy skin in scalp and multiple
areas (eczema), with recurrent infections, and thrombocytopenia.
It is a triad eczema , recurrent
infection and thrombocytopenia =

A- Wiskot Aldrich Syndrome ✅✅✅


Wiskott-Aldrich syndrome

UQU , ‫زم يجي‬V ‫كل سنه‬

42) Case of a pregnant woman in 39 week of gestation who had


severe headache, with hypertension and proteinuria.

A- Plasma exchange
B- Delivery

43) 6 year old child who s toilet trained and complaint of


bedwetting recently A- UTI Toilet training children are at great risk for UTI since they learn how to hold their urine
and not to pass it which can lead to “ voiding dysfunction “ .

44 )Patient underwent a surgery for uterine firboid. It was written in the surgery note
that it has enterned uterine cavity, what s the percentage of placenta accreta?
A-Increase
Yes , increased , UTD

B-Decrease

C-Not affected

(Answers were written like this)


45 )Woman asked not to get pregnant until 2 months of reciving rubella , But she got
pregnant, what s the most likely pregnancy outcome?
A-not affected

B-associated with congintal malformations

If she received the vaccine one. Month beforehand there would be no effect but if she did
not receive it there would be a risk of malformation

46 )7 yo child had vascular malformation on her left cheek?


A-hemangioma
Not clear in other recall
A ‫ على‬B‫بس كلهم متفق‬

B-AV fistula

C-malformation?

47 )Patient fell on outstretched hand, pain at anatomical sniff box, which fracture is it?
A-colles
B-scaphoid - A young man fell on an outstretched hand and there was pain at the anatomic snuff
box:
-Hamate fracture
- Scaphoid fracture
- Colle’s fracture

48) Newborn suddenly developed severe dyspnea and bluish discoloration, dx?
A-transient tachypnea of newborn

49) Newborn has yellowish disoloration that is not faded, started at the 3rd day.
Previously was dx as physiological jaundice and so phototherapy was done. What is
the cause?
A-ABO incombatibility

B-Rh incombatibility

C-G6PD

D-Duodenal atresia

50 )45 y.o. asymptomatic pateint, on pelvic ultrasound that was 2*3(not sure about the
size) uterine fibroid at the fundus. What is the next best appropraite plan of
managment?

A-urgent hysterectomy

B-urgent myomectomy
C-follow up with yealy U/S

D-follow up with serial U/S and CBC every 2 month

51) 55 y.o patient C/O low mood, loss of appetite, insomnia, suisidal ideation, dx?
A-minor depression
It could be minor , ‫السؤال قد يكون ناقص وفيه اعراض زيادة غير كذا مافي ديوريشن‬
d‫ختبار بيكون اوضح ان شاء ا‬V‫في ا‬

B-major depression

C-bipolar disorder

52) Patient C/O indigestion, upper endoscopy revelead an intestinal-type metaplasia at


the lower esophagus, dx? A-barret esophagus Definition: intestinal metaplasia of the esophageal mucosa induced by chronic reflux.
Histopathological examination of the mucosa shows a columnar epithelium instead
B-plummer vinson of the normal squamous epithelium.

AMBOSS

53) Pic of baby with abdominal distention, PR exam empty rectum but when pulled
finger there was stool,, dx? A-hirschpurg disease

54) Patient had mandibular fracture, best way to establish an airway? A-oropharyngeal
mask
B-cricothyrodotomy

55) Most common HCV genotype in Saudi Arabia:


A-1

B-2

C-3

D-4

56 )Couple came to infertility clinic, husband is healthy and atheltic, wife is DM and BMI 35,
what is the best management?
A-investigations for wife only
2 couple came to infertility clinic because of 2 years no pregnancy the
husband look healthy and well while wife Dm obese BMI 30?
What to do
B-investigations for both and lifestyle modifications for wife A- infertility test for wife only
B-lifestyle change for wife as she get pregnant
C- test for both Test for both + lifestyle modification to the wife
57 )Patient was perscribed angiotensin receptor 2 blocker, what is the drug?
A-lisinopril Other recall

Long scenario of then they say they put him on angiotensin receptor ll
blocker, which is the most probably drug that the pt was given?

B-valsartan(was not valsartan but from the same family ~sartan) 1. Cardivol
2. Irbesartan
3. Amidarone
4. Lisinopril (I think)

58 )Child post chemo came in contact with a child with chickenpox, he did not recieve varicella
vaccine before, what to do?
A-give varicella vaccine

B-give varicella immunoglubulins

C-postpond chemotherapy for 4 weeks

59) Scenario of a child with celiac disease, +ve anti-endomysial antibodies and
Antitransglutaminase antibodies (ATA), biopsy shows villous atrophy what to do? A-
gluten free diet
B-gluten free diet for 6 months and then do biopsy again

--------------------------------------

60) Cord prolapse CTG:


A-accelerations Kaplan

B-late decelaration

C-variable deceleration with bradycrdia

61) Patient had normal vaginal delivery, developed massive hemorrhage due to uterine
atony, which of the following is considered the standard of care?
A-oxytocin

B-ergotamine

64 )Which of the following cause patent ductus arteriosus?


A - indomethacin

G
A indomethacin tocolysis has been reported to increase the incidence of
persistent PDA in premature infants.

65 )Patient has a hx of PE, most appropriate contraceptive method?


A-IUD

B-Depoprovera injection

C-combined oral pills

66) 17 y.o female developed weakness and


visual disturbances, MRI report was given
and it showed hyperdense areas in white
mattar, dx?
A-Multiple sclerosis

67) Tumor lysis syndrome(dx was given), findings?


A-hypercalcemia and hypokalemia
In Timor lysis syndrome:

B-hypocalcemia and hyperkalemia Hyperkalemia


Hyperphosphotemia
Hyperurecemia

Hypooooooocalcemia

Dr. Amer

68 ) Ebola patient wants to escape hospital, what to do?


A-call security

69 )Child had multiple bruises, parents said that its because of a fall. You suspected child
abuse , what will you do? Mother with face bruises came to the ER with her child multiple
21 November
‫جاني هالسؤال وعدلت عليه كذا الريكول أفضل‬
bruises all over the body what will you do :
A-call child protection team A - call the father to take consent
Child known case of asthma present to ER with an asthma attack, his
father is heavy smoker, he was told many time to avoid smoking near
the child, but he seems to be careless about it, what is the most
B-confront parents B - Ask the mother to call the police
appropriate action?
A-inform child protection team
B-call hospital ethical committe
C - admit the child for treatment and call child protection services C-Explain to father importance of immunization
D-Ignore father refusal and focus on treating the child

70 )14 y o presents with painless vaginal bleeding at interval from 3 weeks to 2 months.
Her last period was 9 weeks ago(not sure about this info, i might confuse it with
another Q) anyway, her secondary characteristics are normal, what to do?
A-reassure?
14 years old girl complaining from painless bleeding 2 weeks to 3
months
interval that is not related to her period with no abdominal symptoms
Not clear needs more info What is the dx?
A- Bleeding disorder
B- Endocrine disorder
C- Ovulatory dysfunction
71 )Patient (forgot her age), P4 (i m not sure), K/C/O PCO and a smoker, which of the
following increases her chance of endometrial
cancer?
A-PCO

B-Age

C-Smoker

D-Parity

Normal physiology

What are Montgomery's tubercles? Montgomery's tubercles 72 ) Pregnant lady with C/O areolar swelling, o/e there was multiple small well defined
are sebaceous (oil) glands that appear as small bumps around
the dark area of the nipple. Studies have found between 30
and 50 percent of pregnant women notice Montgomery's swellings , limited to the areolar area , dx? A-Montgomery's tubercles
tubercles. Their primary function is lubricating and keeping
germs away from the breasts.

2 73 ) 55 y.o patient was dx with autoimmune thyroiditis, with a hx of progressive enlargement

. of right lobe of the thyroid. FNA report: malignant cells.which of the following is most
likely the type of thyroid malignancy?

A-papillary

B-
medullary
C-
anaplastic
D. Lymphoma ?

74 )Patient with down syndrome came with a hx of irritability and chronic constipation
of 2 months, what is the next best investigation to be done?
A-TSH

75 )Child inhaled forigen body, where will it resides?


A-right bronchus Child with foreign body aspiration (peanut) and doctor prepare
him to do endoscopy, where did you expect its location?
B-left bronchus
A- larynx
B- trachea
C- right main bronchus
D- left main bronchus

76 )60 years old lady presented with lower genital bleeding, she described it as scanty
and barely stains the pads, what is the source of bleeding?
A-fallopian tubes

B-ovaries

C-uterus
D-genital tract

(This is the exact question and choices)

77 )Pic of a baby with stepping reflex and asked when it disappears? Age in months.
A-2 months
B-3 months
Rr C-4 months

78) Patient underwent a surgery for uterine firboid. It was written in the
surgery note that it has enterned uterine cavity, what s the percentage of
placenta accreta? A-Increase
B-Decrease

C-Not affected

(Answers were written like this)

79) Woman asked not to get pregnant until 2 months of reciving rubella ,
But she got pregnant, what s the most likely pregnancy outcome?
A-not affected

B-associated with congintal malformations

80 )7 yo child had vascular malformation on her left cheek?


A-hemangioma
B-AV fistula

C-malformation?

81 )Patient fell on outstretched hand, pain at anatomical sniff box, which fracture is it?
A-colles
B-scaphoid
82 )Newborn suddenly developed severe dyspnea and bluish discoloration, dx?
A-transient tachypnea of newborn
Needs more details

dx as physiological jaundice and so phototherapy was done. What is the cause?


A - ABO incombatibility
B - Rh incombatibility -
C - G6PD
D - Duodenal atresi a ⑦
biliary

83 )Newborn has yellowish disoloration that is not faded, started at the 3rd day.
Previously was
84 )45 y.o. asymptomatic pateint, on pelvic ultrasound that was 2*3(not sure about the
size) uterine fibroid at the fundus. What is the next best appropraite plan of
managment?
A-urgent hysterectomy

B-urgent myomectomy

C-follow up with yealy U/S


D-follow up with serial U/S and CBC every 2 months

85 )55 y.o patient C/O low mood, loss of appetite, insomnia, suisidal ideation, dx?
A-minor depression

B-major depression

C-bipolar disorder

86 )Patient C/O indigestion, upper endoscopy revelead an intestinal-type metaplasia at


the lower esophagus, dx? B-barret esophagus
A-plummer vinson

87 )Pic of baby with abdominal distention, PR exam empty rectum but when pulled
finger there was stool,, dx? A-hirschpurg disease

88 ) Patient had mandibular fracture, best way to establish an airway?


A-oropharyngeal mask

B-cricothyrodotomy

89) Most common HCV genotype in Saudi Arabia:

A-1

B-2

C-3

D-4

90 )Couple came to infertility clinic, husband is healthy and atheltic, wife is DM and BMI
35, what is the best management?
A-investigations for wife only

B-investigations for both and lifestyle modifications for wife

91 ) Patient was perscribed angiotensin receptor 2 blocker, what is the drug?


A-lisinopril

B-valsartan(was not valsartan but from the same family ~sartan)

92) Child post chemo came in contact with a child with chickenpox, he did not
recieve varicella vaccine before, what to do?
A-give varicella vaccine

B-give varicella immunoglubulins

C-postpond chemotherapy for 4 weeks

93) Scenario of a child with celiac disease, +ve anti-endomysial antibodies and
Antitransglutaminase antibodies (ATA), biopsy shows villous atrophy what to do?
A-gluten free diet
B-gluten free diet for 6 months and then do biopsy again
94 )Mother worried about her asthmatic child, and how asthma will affect him in
adulthood? A-most children will come out of it
B-it will get worse

95 )Patient has lower back pain and hyposethesia of left calf and foot, which lumbar
vertebrae is most likely affected?
A-L4-L5

B-S1-S2

C-L5-S1 laterally

96) Patient had normal vaginal delivery, developed massive hemorrhage due to uterine
atony, which of the following is considered the standard of care? A-oxytocin

B-ergotamine

97 )Which of the following cause patent ductus arteriosus?


A-indomethacin

98 ) Patient has a hx of PE, most appropriate contraceptive method? -IUD


A-Depoprovera injection

B-combined oral pills

99 )Patient delivered her baby in the same day she was discovered in the morning round
to be o and the father is +A, her fetus is A+ and was given anti-d (I think 3000 not
sure of the does). What amount of fetal blood will be protected?
A- 60 ml of fetal blood
B- 30 ml of fetal blood
C- 15 ml of fetal blood

100) Patient with sore throat fatigue and shortness of breath and productive
cough with whitish sputum, sometimes there's bloody streaks. X-ray: normal
with no conselidation. What's the diagnosis?

A-Hereditary telangtesia

B-Good pasture syndrome


C-Pleural edema

D-Bronchitis

102) Symptoms of cystic fibrosis (they provided sweat chloride test higher than normal
in the question) and asked what is the most common complication? A-Bronchectasis
B-Empyema By Google

103) 17 year old female, medically free, gymnast in her cass, developed breasts later
and never menstruated, on examination she is tanner stage 5 but no menstruation,
diagnosis?
A- Hypothalamic hypogonadism
B- Gonadal aeenesis
C- Imporferated hymen

104) Pt diagnosed with hepatic benign tumor dx is hemangioma (mentioned), they


decided that
the patient will be managed conservatively. What should you advice the patient
A-Avoid contact sport and truma

B-Lose weight

C-Smoking cessation

105) Pt presented with jaundice fatigue intermittent fever hx of traveling to India?


A-hydatid cyst

B-amoebic cyst of liver

106) Drug abuser agitated and has palpitation for a couple of days, what s the cause of
his symptoms: A-Cocain withdrawal 18 year old male presenting with 1 week history of euphoria,

B-Amphetamine toxicity nausea vomiting, weight loss, agitation, hallucinations. What is


the diagnosis?:

A. cannabis intoxication
B. Amphetamine intoxication
C. Cocaine withdrawal
D. Schizophrenia
Answer: B

108) Camping in the community to decrease risk of hypertension is considered as?

A) Primary
B) Primordial C) Secondary
D) Tertiary
109) Child presented to the ER with his parents as they were saying that their child
ingested potentially toxic disease of some medication. What is the next step in
management? A- Activated charcoal See other recall

B- Gastric lavage Paracetamol toxicity in a child after 3 hours what to give?


N acetylecestine@
Activated charcoal

C- Ipecac syrup According to UTD and Dr.Sfdar it depends on the duration


< 4 h use Activated charcoal
> 4 h N-actylecystine the anti dote

111) 35 y/o female presented with chest pain and shortness of breath, chest x-ray
normal and

O2 sat were 88% PFT done for her all were normal except for DCOL it was 40% normal
range is ( 60-120 ). What is the Dx

A- PE By dr. Amer course

B- Pneumonia
C- Heart failure
D- Acute bronchitis

112) Child with sandpaper rash, what is the organism causing it?
A-Staphylococcus aureus

B-Group A streptococcus

C-Group B streptococcus

113) A very long case of SLE (they gave symptoms), ANA is high and they asked what is
the confirmatory test?
A-Anti-dsDNA

B-Anti-CCP
By UQU
C-Anti-smith

114) Pt with face laceration ,repair was done and they use lidocaine Most common
complications of lidocaine?
A-Nystagmus

B-Vventricular tachycardia

C-Drowsiness By medscape

115) Most common type of fibroids?


A-Intramural
fibroid :
Bleeding = Submucosal
Most common = Intramural
B-Subserosal See other recall

C-Submucosal Patient with menorragia, which type of fibroid does she have:
A. Intramural
B. Subserosal
C. Cervical
D. Submucosal
116) Which lymph node will be affected in female patient have something in uterus and
Old lady with uterine fundal mass. (Uterine CA) underwent
going for surgery? A-intrailiac surgery. What lymph nodes to resect?
a. External iliac lymph node

B-Para aortic b. Internal iliac lymph node


c. Deep inguinal lymph node
d. Para-aortic lymph node✅

Wafaa

117) Patient post-esophageal dilatation, came to the ER 8 hours after the operation
complaining of chest pain and change in voice? 68 y/o male presented to ER 8 (days or hours) post esophageal

A-Hemorrhage stricture dilatation c/o sever chest pain and change in voice, dx
?
- haemorrhage
-perforation ✅
- aspiration
B-Aspiration ?

118) 8years old boy. Mother complained of poor training to bath room which muscles is
targeted in therapy?
A-Perianal

B-pelvic floor C

C-Rectus muscle

D-Detrusor Dr.Safdar

119 )Female patient she had pulmonary embolism 3 years ago she want effective
contraceptives method :

A-IUD

B-condom

C-Dermal patch

D-OCPs

120) Patient with hemorrhoids that is reduced on its own. Whats the grade? A-1
B-2

C-3

D-4

121) Patient with previous history of PE, what's the recommended contraception
method? A-IUD
B-OCP

122) Elderly came to the family medicine clinic, what vaccine you should give?
A-Pneumococcal
UTD Elderly healthy medically free came for vaccines

There are five vaccines adults age 65 and older should A) meningeococcal
consider to prevent certain diseases: B) pneumococcal ✅
C) MMr
Influenza (flu) vaccine
D hepatitis
Pneumonia vaccine

Herpes zoster vaccine

Tetanus, diphtheria and pertussis (Tdap) vaccine

COVID-19 vaccin
B-Meningococcal

127 Ha
-

123) Screening for pregnant for asymptotic Bacteriuria according to U.S. Preventive
Services Task Force (USPSTF):
A- weeks 12
B- weeks 20
C- weeks 26
D-weeks 32

124 )patient did colonoscopy 1 polyps size 1.5cm and biopsy showed tubular adenoma.
Interval of screening? A-No need

B-6 months

C -3 y

D- 5-10 y

125) Female post partum 2 weeks , she is breastfeeding, what contraception method ?
A-POP See other recall
Mother just gave birth, exclusively breastfeeding, want
contraception for 2 years?
B-medroxyprogesterone acetate A- Depo provera injections
B- POP
C- OCP

Wafaa
C-COC

D-IUD

Pop for the first six weeks then iud Mirena


126) patient came with chest pain (+ECG) he doesn t know his medication, he also
complains of yellow vision. Diagnosis?
A-Digoxin toxicity Dr.Amer

127) how to measure fetal weight?


A. Head circumference
B. Abdominal circumference Wafaa

C. Biparital diameter
D. Femur length
Hacker says umbilical circumference
128) Patient came with chest pain (muscle) increased with movement, started after
heavy training See complete a with Dr. Amer
A. Nitroglycerin
B. Reassure
My answer is in green
129) febrile neutropenia empirical
antibiotic

A. Meropenum Dr. Amer ; Meropenum, or Cifepime or tazocine


If there is central line add Vanco.

B. Vancomycin
High risk meropenem low risk
ciprofloxacin and Amoxiciliin

130)COPD on PFT ?

B.A. increasedecrease TLC TLC , ,decrease decreased


FEV1/FEVC FEV1/FEVC <0.7 <0.7, decrease, increase

VC VC in emphysema

132) Case of bronchiolitis (severe) with chest recessions what is the management?
A-Admit for fluids hydration and oxygen

133) type of inheritance of Cystic fibrosis"=?:


A-autosomal recessive Dr.safdar

134) 10 m old infant with pneumococcal infection and repeated infections. His brother
died from severe sepsis. on studies he has few B cells but normal T cells diagnosis:
A. xlinked agammaglobinemia

135) girl with diarrhea and dehydration , splenomegaly , Hb is low , direct and indirect
coomb is +ve , Dx
? A. Autoimmune hemolytic anemia Since Coombs test is positive

136) unvaccinated kid and his dad said he doesn t believe in vaccinations, what to do:
A.Inform child protection
151-Unvaccinated boy and the mother said the vaccine not
good. What to do!
A. Explain to the mother about the importance of vaccine
B. Talk to the father about the importance of vaccinations B. Ignore and treat
C. Inform child protection

Answer: A

137) 30-min aged baby preterm of 27 weeks, developed tachypnea and grunting , Dx:
A. Respiratory distress syndrome → isrisk factor
By safdar
138) 25 yo F. Worried about cervical cancer. She took her first dose of HPV vaccine 3
months

ago. interval
What the best thing to do at this visit today? Gmto 12 the
m

A. Schedule app after 3 months u doses BT Two B. No need to do anything at


this visit C. Give 2nd dose at this visit -
-

D. Repeat 1st dose

My answer is in green
139) 10 week GA pregnent lady with UTI witch of the following drug is contra-
indicated : A- nitrofurantoin

B- ciprofloxacin

455-The World Health Organization (WHO) published the WHO Surgical Safety Checklist in 2008 in order to
increase the safety of patients undergoing surgery. The Checklist consists of three phases of surgical

140) Surgery safety checklist : procedure :

A. Before admission , Before skin incision , at discharge from hospital.

B. Before admission , before induction of anaesthesia , at discharge from hospital.


A- before entering OR C. Before induction of anaesthesia , Before skin incision , Before patient leaves operating room.

D. Before admission , Before induction of anaesthesia ,Before patient leaves operating room.

Answer: C

141) vaccinations for elderly: a- pnmoccal ???


142) Warfarin did not stop by patient regardless of
clear instructions
, the nurse in OR noticed that the patients didnt stop the drug as instructed and
informed the surgeon , So the surgeon postponed the operation:

A- near miss
B- adverse event
C- medical neglicance

143) Case of stroke and complain of weakness in the arm leg and face, where s the
damage?
1- Middle cerebral artery readit in
2- Anterior cerebral artery → legs only
3- Mid basilar artery Toronto → Andreplasia and dysarthria
144) placenta abruption case management:
A-multidisciplinary team Waffaa

145) When to diagnose pedia as hypertension: A - bp >


120/80 B - bp > 150/.. When to diagnose pedia as hypertension?

C - >90% depends age and sex A- bp > 120/80


B- bp > 150
C- >90% depends age and sex
D- >95% depends age and sex
UTD
D - >95% depends age and sex

146) Man after trauma presented complaining that he can t hold fork to his mouth,
where is the lesion? A. Cerebellum See other recall

A patient who has a tremor in his right hand,


seen when reaching for an object and with the
B. Ventricles finger-nose test. Were is the lesion?
-Right cerebellum✅
-left cerebellum
C. Cerebrum -left basal ganglia
-left motor cortex

147) research s are trying to collect data from multiple studies, how to apply
that? A. Meta analysis

B. Case control
C. Cohort study

148) child had pertussis and has 2 other siblings in home, what to do regarding them?

A. Give Dtap immediately


B. Give dtap if high risk
C. Observe for signs and symptoms
D. Give prophylactics treatment

149) child 5 y/o with recurrent viral infection, dry cough increase at night, he is on LABA
with no improvement, what to add to tx?
Is it asthma case !

A. Cough syrup
B. ICS
C. Oral beta agonist

150) female with multiple liver masses, k/c of hepatitis c, what is your further
management? A. Liver biopsy

B. Us
C. CT

151) y/o female pregnant, polyhydraminos had ruptured membrane, on CTG


persistent fetal bradycardia? A. Rapid fetal descend

B. Cord prolapse By Wafaa

C. Anomaly

152) child presented to you with history of coin ingestion 6 hours ago, child is stable,
on X-ray it s in the stomach, Mx? A. Immediate Endoscopy
See other recall from Earth channel

B. Wait till it passes by it s own 794-A child swallowed a coin. And on imaging it was on stomach. What to do (I had the same scenario but with drooling and fatigue)
A) observe
B) remove by endoscopy

C. Perform lavage C) go home and wait till it comes out in stool


D) don’t remember

Answer: B

The NASPGHAN Endoscopy Committee recommends button battery removal within 2 hours in a symptomatic children regardless of size [9].
A button battery ≥20 mm located in the stomach of an asymptomatic children aged <5 years should be removed within 24 to 48 hours. If
serial X-rays do not show progressive movement of an ingested FB in asymptomatic children, it can be observed for 24 hours. Magnets
retained in the stomach in symptomatic children require removal within 2 hours. In asymptomatic children, they should be removed within 24
hours. Coins in the stomach of symptomatic children should be removed within 24 hours. In asymptomatic children, these can be observed
for 24 hours. Long or large FBs in the stomach necessitate removal within 24 hours

Patients with drooling, marked emesis, or altered mental status (likely from excess vagal stimulation) may require supportive measures to
protect the airway.
Corrected 252-280
Smle April
‫منال‬
، ‫ عبارة عن مانع حمل هرموني على شكل لولب ويتدخل بالرحم‬iud merina A‫او‬
heavy bleeding and anemia ‫ريضة تعاني من‬G‫ ا‬،‫بحيث انه يفرز بروجستيرون‬
‫لم ايضا عكس‬A‫رينا انه يقلل كمية الدورة ويخفف ا‬G‫ حق ا‬SI‫ومعروف من ال‬
‫لم‬A‫ اللي يزيد كمية الدورة وا‬copper iud ‫اللولب النحاسي‬

‫رينا راح يساعدنا في حل مشكلة الدورة الغزيره ومشكلة فقر الدم‬G‫فبذلك ا‬


B) IUD mirena
Mirena IUDs work to prevent pregnancy by thickening the mucus in the
cervix, which stops sperm from fertilizing an egg, and thinning the lining
C) condom of the uterus, which suppresses menstrual bleeding.

D) OCP foods and liquids other than breast milk


or infant formula) during the first year
of life has varied over time and across
: Child 9 months introduced to solid food late, still breast feed. What will you give: cultures. The American Academy of
Pediatrics and World Health
From 7 to 12 months of age, iron intake should be 11 mg/day. In general, an average of Organization recommend that
two servings (a total of 30 g or one-half of a cup of dry cereal) of iron-fortified cereal complementary foods be introduced
A) Iron in combination with human milk is sufficient to meet the daily iron requirement. around six months of age

After six months of age, the volume of human milk ingested by exclusively breastfed infants generally becomes insufficient to meet the
B) Vit D infant's requirements for energy, protein, iron, zinc, and some fat-soluble vitamins

: neck injury in one CTA sho s essel inj r hat s o r ne t step

A) ligation Only if stable

B) primary suture
Because patient
C) embolization unstable

: Family physician have approved FDA drug for something and Share holders To community

A) conflict of interest

B) neglected something

: foul smelling watery diarrhea, green in color, what is the causative organism?

A) shigella bloody ‫ذولي يجون‬

B) salmonella

C) Rota Watery diarrhea https://www.amboss.com/us/knowledge/diarrhea

: Woman takes OCP with strong androgenic progestin found out she is pregnant what do you expect to
find in female fetus

A) External genitalia masculine ( virilization)

B) Feminine

C) Adrenache

D) No effect
‫يتابولزم يجي معها ميتابولك‬G‫كل امراض ا‬
‫اسيدوسز‬
: child hypoglycemia & seizure & metabolic acidosis & characteristics smell & positive ketone Diagnosis?

A) Phenylketonuria

B) maple syrup urine disease

Pt sustained injury to the left chest now there is sucking wound between 3rd and 4th ICS how will you
manage :
Uptodate

https://pro.uptodatefree.ir/show/13862
A) Chest tube

B) three sides dressing

C) analgesics

: 14 y/o boy ingested bottle of acetaminophen tablets brought by his parents 20 hour later, asking about
which stage of toxicity?
24 ‫ماتجاوزت‬
A) 1
‫ساعة‬

B) 2

C) 3

D) 4

: 35 year old lady presents with left nipple bloody discharge, diagnosed as Intraductal papilloma, how
will you treat her?

A) duct excision

B) observation
image guided biopsy ‫هذا الجواب من‬
D)mastectomy
C)
‫دورة ثوابة‬

: A pregnant lady came for. Antenatal screening. All within normal except for Zero Rubella antibody
titers. What s the next thing to do?

A) Give MMR at this visit

B) Schedule app after 3 months for .......

: Victim in RTA who did Ureteroscope urethrogram and found injury in the bulbar what do you want to
do next?

A) pubic catheter

B) Foley catheter

pt dm2 on lifestyle modification what medication give ?

A) sulfonylureas
‫هو‬
B) biguanides 7‫ا;يتفورم‬

C) ddp4 inhib

: Pedia with systolic murmur and palpable liver

A) cystic fibrosis

B) pda
C) vsd

Which of the following is true in necrotizing pancreatitis.

A) increased lipolysis

B) Hypoglycemia

C) Decreased gluconeogenesis

2 month baby with fever, cough, fine crackles, on xray puffy infiltration

A) bronchopneumonia

B) cystic fibrosis

: Man who fell in marathon and came to ER with something like (dark urine or blood) But no microscopic
RBC Dx??

A) Hemorrhagic cyst
https://medlineplus.gov/ency/article/000473.htm

B) Rhabdomyolysis

: Screening test for a disease in low prevalence area will result in increase in:

A- False negative

B- False positive

C- True negative

D- True positive

: COPD patient presenting with bilateral lower limb edema and pulmonary

hypertension. PO2 8.6, PCO2 7.5, pH within normal range Which of the following

the appropriate management?

A) start furosemide https://next.amboss.com/us/article/TI06Xh?


q=management%20of%20copd%20exacerbation#Zc6d0fbd53f7d50d
b917a571d31f1dd6e
B) Start oxygen therapy

C) Prednisolone therapy

: Deep wound in anterior thigh 10 cm , bleeding , how to contol bleeding

A) direct pressure on the wound https://pro.uptodatefree.ir/show/15912

B) apply torniquete

C) pressure on the femoral vessle above the wound

Kawasaki case - what is the modialty of choice to assess coronary arteries?

A)Echo
‫ا بحثت قوقل وامبوس يقولون غلط‬5
‫ بحطه‬angiography ‫بس مدري انا لو جاني خيار‬
B) Coronary cath

: Long Cardiac case discharged on meds, What drug causes high uric acid?

A) Lasix

B) plavix

C) acei

D) asprin

: Alternative tx for severe depression?

A) elctrocovusive therapy
https://next.amboss.com/us/article/
PP0WUT?
B) TCA q=Depression#Z1f104e7daf14d39fb
270b5b75766553d
C) Elctro somthing graph therapy

: Paragravedous in labour, When Latent phase consedrid prolonged?

A) 2 H

B) 4 h

C) 8 h

D) 18 h

: Pt sustained injury to the left chest now there is sucking wound between 3rd and 4th ICS how will you
manage :

A) Chest tube

B) three sides dressing

C) analgesics

: 14 y/o boy ingested bottle of acetaminophen tablets brought by his parents 20 hour later, asking about
which stage of toxicity?

: Deep wound in anterior thigh 10 cm , bleeding , how to contol bleeding

A) direct pressure on the wound

B) apply torniquete

C) pressure on the femoral vessle above the wound

Kawasaki case - what is the modialty of choice to assess coronary arteries?

A)Echo
B) Coronary cath

Long Cardiac case discharged on meds, What drug causes high uric acid?

A) Lasix

B) plavix

C) acei

D) asprin

7 yo female last pregnancy 15 years ago, amenorrhea for 7 months, negative

pregnancy test,

wishes to get pregnant, what is the most appropriate investigation:

-FSH, LH

-Hysterosalpiogram

-Endometrial sampling

: Alternative tx for severe depression?

A) elctrocovusive therapy

B) TCA

C) Elctro somthing graph therapy

: Paragravedous in labour, When Latent phase consedrid prolonged?

A) 2 H

B) 4 h

C) 8 h

D) 18 h

Submucosal fibroid removal in a female wants to preserve her fertility?

A) hystroscopic resection by myomectomy

B) Lap myomectomy

C) Lap hystrectomy

Endometriosis most acceptable dx method?

A) MRI

B) Laparoscopic
C) US

: K/c of nephrotic Long hx with abdominal pain, What is the possible complication?

A) Gastritis
‫احس السيناريو ناقص ؟‬
B) Peritonitis

: 35 year old female, smokes 12-14/day. Came to antenatal clinic .. what is the greatest risk factor for
sub fertility?

A) smoking
‫ومن مراجع اخرى ومن دكاتره برضوا‬
B) Maternal age A ‫يقولوا ان الجواب بي وليس‬

Case long hx newborn. I only remember low plts, there are hb and hct in labs ( but needs unit conversion
) & baby was in respiratory distress and cyanosis ?

A) polycythemia

B) Primary pulmonarry htn

C) Heart failure

Pregnant in 27 GA, came with minimal bleeding, us showed Placenta totalis. What is the most imp mx ?

A) abx

B) Tocolytics
‫ حقت‬lung maturity ‫ليش ستيرويد ؟ علشان‬
C) Steroid ‫ اسبوع‬34 ‫ن عمره اقل من‬O ‫البيبي‬

Medial ulcer RF?

A) burger disease

B) Dm

C) Atherosclerosis

D) Venous HTN

ea old bo da of ja ndice and abdominal ain didn men ion fever)

Labs :

Indirect and total bilirubin ( high )

AST 1000

ALT
A) infectious hepatitis

B) Gilbert
Has normal liver enzymes according to amboss

. Bilateral hydronephrosis in baby. What is the best diagnostic test ?

A) MCUG

) Other scan type B) DMSA

C) IV pyelogram

Investigation and management of antenatally detected hydronephrosis


https://www.ncbi.nlm.nih.gov/pmc/articles/PMC2645869/

: Case in 2nd trimester came with minimal bleeding i think, gestational age in us lesser than GA, no fetal
heart detected and no movement, no hx of passing tissues?

A) Fetal demise

B) missed abortion

If it was after 20 GA Fetal demise

Before missed abortion

dm female otherwise all normal full term in delivery fetus had tachycardia how to prevent this.?

O tocin s

Change mother position ‫مالقيت الجواب اتوقع 'نه يضغط‬


‫على ا'ورتا‬
Mg glucanate

5)Dm pt what in labs can determine néphropathies ?

serum creatinin

Albumin creatinin ratio

: 6)chest xray of newborn labs all normal vital signs low oxygen saturation no fever using esocssory
muscles whats the diagnosis? TTN

pt with ascites huge ascities already on 40mg forusmide and dose if hydrothazide hydrochlorothiazide

How to manage ?

increase durtic dose uptodate


Give IV durtics
‫السوال مو كامل‬
Tapping< but médical term ‫بس هذا مصدر‬

https://next.amboss.com/us/article/
how to determine fluid replacement in 15 year old burn victim? Urine output fM0kLg?
q=fluid%20replacement#Z9ac7aecb4f5181
15 year old must do csf sample take consent from who? Options the boy only 55cdf950f1ab272b88

Parents only
https://www.moh.gov.sa/Documents/2019-12-09-001.pdf
Both?

known of a sickle cell anemia, you want to give her folic

acid, what is the right prescription for her?‫اذا بيشنت عاديه مافيها انيميا منجلية توقف‬
‫ اسبوع‬١٢ ‫عند‬
A- 5 mg folic acid till 12 week
‫'نها سيكلد سيل انيميا تاخذ الفوليك اسيد للو'دة وحتى‬
‫بد يعطونهم ا?صدر امبوس‬E‫ل‬
B- 5 mg folic acid till she gave birth

C- μg folic acid till weekg folic acid till week

D- μg folic acid till weekg folic acid till

child came with potential toxic dose of paracetamol The physician requests an Acetaminophen

level The lab technician calls to report results He says two stops and pauses and then says one,

three The nurse mistakenly writes it as while the real result was The patient went into

irreversible liver damage, What was wrong here?

A. ................

B. Poor communication

C. Nurse who failed to repeat the result and double check she got the accurate result.

D Lab technician who didn t repeat or double check that the nurse heard him and recorded correctly.

Hot thyroid nodule 3x3 was removed by thyroid lobectomy , 8mm papillary will defined focus was found
distant to the leison, what is the appropriate?

A. Complete thyroidectomy

B. Follow up 3 months https://www.cancer.gov/types/thyroid/patient/thyroid-treatment-pdq

C .RAI

2 Cases of vaginal discharge one with frgellated motile tx metronidazole the other fishy odor tx:
metronidazole

Infant Mortality Rate has decreased to 4.28 in 2020 in Saudi Arabia.


Which of the following figures involved in This:

A) Number of women in childbirths age

B) Number of deaths in saudi arabia in 2020

C) children who died in 2020 and not completed their first year

- new Q pt with sle, anti phospholipids, milumisscarges > what to give

*LMWH bridging with warfarin BMJ /uptodate https://next.amboss.com/us/article/Tm06Ug?


q=anticoagulant#Z868d6a2028d5c175da17ef6f1dc97b5a
*asprin

* warfrin

-new Q baby term dilvered with macrosomnia and ARDS what next step:

*ovserve and glucose

*observe in icu with glucose

*intubate only if po less than 60 with low 9 sat

common SE of lidocone

*drowsiness

*Nastigmus

-new Q pt with anterior neck and devaible tissue what to do?

*intubate

*CT

-it depends unstable intubate stable ct

new Q how to know ventrcular if abnormal

*ck? Or BNP if it was there

*troponin

*.......

Symptoms of Crohn's then ask which drug should you try it first to see Is it useful or not?

- steroid

- 5-aminosalicylic acid.
- Azathioprine

- Abx

Pan sustolic murmur


https://next.amboss.com/us/article/rM0fJg?
*vsd q=heart%20murmurs#Za94cbf56c65ba879048421c95766464c

*asd

Female have dysmenorrhea what is the important to ask her in history

Medical

Family

Surgical

Mensturl

: 38 weeks CS baby with tachycardia and grunting?

TTN https://next.amboss.com/us/article/340SQT?
q=transient%20tachypnea%20of%20the%20newborn#Z7ce062e2ac91888cce715b8ccd40533e

Pt presented with jaundice fatigue intermittent fever hx of

traveling to India , imagining show hypoechoic cyst ?

hydatid cyst

amoebic cyst of liver

Hypoechoic cyst

: Pt with increased ICP next best ?

IV manitol

Raise the bed from the head side

Another options can’t remember 3- hyperventilation

most recommended tool for checking prognosis after asthma attack?

A- Chest x-ray

B- ABG

C- Pulse oximetry

D- Peak flow rate


: RTA pt with maltipule mandpular fracuter with sever bleeding, uncontious, no vitales menstioned

How would you mange his Airway?

A. Laryngial mask
Contraindicated in
B. Orotracheal facial fracture

C. nasotracheal

A- D.Cricothyrotomy

metastasis ‫اثنينهم مرتفع) دليل ان فيه‬


‫ يكون‬Alp‫ ال‬bph‫ مرتفع وفي ال‬Alp‫ن ال‬:
‫عادي‬
‫ فقط‬Bph ‫هذا دليل انه سرطان وليس‬
elderly with dysuria, and dificulty in urination, PSA high, ALP high, what is the diagnosis?

A- Urinary bladder ca

B- Prostatic ca

C- BPH

year old lady presents with left nipple bloody discharge, diagnosed as Intraductal papilloma, how will
you treat her?

A- duct exsion

B- opservation

C- mastectomy

Ministry recommend doing campaign to educate people about decreasing high carbs and fatty diet to
decrease the risk of HTN
Type of prevention

- primary

- secondary

- tertiary

- primordial

: Female came with white breast discharge and high prolactin, what radiology you will do?

A- Pelvic MRI

B- Chest MRI https://pro.uptodatefree.ir/show/802

C- Sella turcica MRI

: 0’s male diabetic BMI: , diagnosed with BPH. What is the most important risk factor?

A- Age

B- Diabetes

C- Obesity

D- Hypergonadisim (something like that)


: An 18 or 25 yo F. Worried about cervical cancer. She took her first dose of HPV vaccine 3 months ago.
What the best thing to do at this visit today?

A. Schedule app after 3 months

B. No need to do anything at this visit

C. Give 2nd dose at this visit

D. Repeat 1st dose

: A pediatric patient came in RTA. Has splenic rupture and thus splenectomy was done.

What s an appropriate vaccine later on?

A. Pneumococcal

A. Meningococcal

C. MMR

: 10 week GA pregnent lady with UTI witch of the following drug is contra-indicated:
During pregnancy, you may be screened for urinary tract
A- nitrofurantoin infections (UTIs). Even if you don't have symptoms,
you'll likely be prescribed an antibiotic to treat it.
B- ciprofloxacin Amoxicillin and cephalexin (Keflex) are usually the first-
choice antibiotics for a UTI during pregnancy.
C- amoxicillin

D- Ceftriaxon
Screening for pregnant for asymptotic Bacteriuria according to U.S. Preventive Services Task Force
(USPSTF) :

A- 12 weeks

B- 20 weeks

C- 26 weeks

D- 32 weeks

camping in the community to educate people about health determents considerd as?

A-health education

B-active surveillance

pt with fever and chest pain and they mentioned an ECG finding they ask about which part will be
affected

A- Pericardium MedLine
B- Myocardium

a o male i h rec al bleeding a biop from igmoid ho ed adenoma a ha in er al hall


you do colonoscopy screening?

A- 3 and 6 months

B- 3 years

B- 10 years

C- No need
Ca e of oke and complain of eakne in he a m leg and face he e he damage

A. Middle cerebral artery

B. Anterior cerebral artery

C. Mid basilar artery

Old patient hx of HF with dialated cardiomyopathy and A.fib .. what is the most appropriate
management to control his heart rate?

A- Digoxin

B- amlore

RTA pt with maltipule mandpular fracuter with sever bleeding, uncontious, no vitales menstioned

How would you mange his Airway?

A. Laryngial mask

B. Orotracheal

C. nasotracheal
A- D.Cricothyrotomy

What of the following decrease the risk of preeclampsia?

A- Antibiotic
Amboss and
B- Aspirin UTD ‫مكرر‬

The time for introducing solid food for baby;

A- 14 months

B- 18 months

C- 22 months

D- 6 months

Pt found to ha e anofistula on o clock

M.c Anofistula intenal openning ?

Lt lateral

Medial posterior

Medial anterior

Immune competence with aspergillosis infx

Treatment:

Flocanazol my choice

Pt was on malaria medication doctor didint now about it and started him on digoxin

What should done to avoid this complication:

Pt should bring his medication

Nurse should know what pt is on

Computerized system medication


1- Surgery safety checklist:

A-Before anesthesia, before skin incision, before leave OR.

B-before admission, before anesthesia, before discharge

C- before OR, during operation, at recovery room✅

Pt sustained injury to the left chest now there is sucking wound between 3rd and 4th ICS how will you
manage :

A- Chest tube

B- three sides dressing

C- analgesics

Pt had previous ectopic pregnancy.

The percentage of recurrence?

A- 5%

B- 10%
‫سوال مهم برضوا تكرر باغلب اسئلة الهيئة ابحثو عنه‬
C- 30%

E- 50%

Hacker and Moore says 30%

clinical pic of opioid toxicity, asking about the antidote?

Naloxone✅

: elderly with dysuria, and dificulty in urination, PSA high, ALP high, what is the diagnosis?

A- Urinary bladder ca

B- Prostatic ca

C- BPH

8- Risk factor for esophageal ca?

A- esophageal stricture

B- Barrett's esophagus

C- GERD
: patient with renal failure presents with low calcium, high alkaline phosphatase, and high PTH. What is
the best form of vitamin D to give this patient?

A- Calcitriol

B- Vitamin D2

C- Vitamin D3

Pt came 3 days after roux-Y surgery complaining of fever chill and left shoulder pain, best diagnostic
investigation?

A- CT with contrast

B- Endoscopy

C- Laparoscopy

D- exploratory laparotomy

: year old lady presents with left nipple bloody discharge, diagnosed as Intraductal papilloma, how will
you treat her?

A- duct exsion

B- opservation

C- mastectomy

child i h S inal M c la A h The edia ician deemed he need in ba i n B h a en d n


want intubation as they had a son with the same illness who died. What do you do?

A- call child protection


‫صدر فوق من وزارة‬-‫مكرر حطيت ا‬
B- Search the internet for solutions of similar cases ‫ت الخطرة والوفاة‬8‫ في حا‬، ‫الصحة‬
‫م او اي احد‬8‫ب وا‬8‫ يهم راي ا‬8 ‫والحياة‬
C- Discuss with hospital ethics committee ‫بالحياة‬

D- Ignore both parents and intubate the child as this is an emergency

What treatment will you give for moderate asthma?

A- SABA
Amboss
B- LABA

C- LABA + Inhelar corticosteroids

F- LABA + Anticholinergic
SMLE: 16- pt came with his shoulder flat, his arm addcted and internally rotated, what is the
diagnosis?

A- Anterior dislocation

B- Inferior dislocation

C- Posterior dislocation

: Most common type of fibroid:

Submucosal

Subdural

Intramural

38 wk Pregnant with preeclampsia ( headache, high BP, ) management

Delivery Amboss
Pregnant with bleeding, placenta abruption

- Rapid response team

- ICU admission under OBs care

Ministry recommend doing campaign to educate people about decreasing high carbs and fatty diet to
decrease the risk of HTN

Type of prevention

- primary

- secondary

- tertiary

- primordial

Case of cirrhosis with ascites

Presenting with abdominal pain

Management:

- Antibiotic ‫السؤال مو واضح‬


- digonostic laparoscopy

- exploratory laparoscopy

Women with Breast benign phylloid tumor, management

- localize excision
Amboss
- mastectomy

- localized excision + radiotherapy

Pediatric patient presented with lower limp numbness and muscle weakness

Has hx of fever and sneezing, diagnosis:

- poliomyelitis

- polymyositis

- Guillain-Barre syndrome

: Patient presented with confusion and loss of sensation only in lower limp:

- MCA

- ACA

Patient with rest tumer, mask face, shuffling steps:

Pa kin n di ea e

Shaima SMLE: Alternative treatment for sever depression:

TCA

Psychotherapy

Electrical

Asthmatic on Short acting B blocker

With increase symptoms with exertion

What to add: Amboss asthma


- Inhaler corticosteroid

- inhaler corticosteroid + long acting B blocker

Patient with cough and inspiratory strider, But croup rolled out

What is best management


‫فقط بكروب لكن مدري هل ينطبق‬
- warm humidifier ‫ت؟ اذا‬:‫ في كل الحا‬stridor‫على ال‬
‫ايه معناه هو الجواب‬
- repeat epinephrine dose

New born seizure after delivery with hypoglycemia, management

- vit. K
First treat the cause
- glucose
- diazepam

- antibiotic

: According to my references it is B

[: 36. Pt 30s with irregular period for 6 month, she uses pills to get the menstruation , he has hx of
infertility for 2 years , underwent many investigations:

TSH normal

Prolactin normal

Progesterone and estorgen were low

FSH, LH are high

What to give for fertility:

A) GHRN
Amboss + Wafaa
B) ovulation induction with clomiphene

C) ovulation induction with Gonadotropins

[22 month infant with right inflamed red hemi scrotum, on examination there was red firm irreducible
painful scrotal swelling which is extended to left inguinal region. Left testis couldn't be palpated. What is
the diagnosis?

A. Torsion appendix testis.

B. Testicular torsion

C. Epididymo orchitis.

D. Incarcerated inguinal hernia

: 1-

6-two questions about mutiple miscarriages asking about the probable diagnosis?

A-antiphospholipid syndrome ✅

7-what is used for "short term" treatment of migrane ?NSAIDS ✅

8-17 yr old male presented with severe asthma exacerbation on albuterol and long acting glucocorticoid,
hx of admission 2 yrs ago
On exam: resp distress and diffuse expiratory wheezing

..

He Received systemic glucocorticoid and b2 agonist inhaler but no improvemnt..

..

Cxr : hyper inflation


this is the closet answer according to MOH guideline
Ph 7.3

What next;

1, admission to icu

2.discharge and reassurance

3.disscharge and follow up next day

4. Admit to medical word

9-50 yr old , with IHD and dm

Admitted to icu with severe pneumonia and was treated with abx ..

After 3 days of admission, he developed hypotension and treated with hydration and inotrope

,On admission lab was normal

After 3 days, LFT was abnormal

Total bil 20(increased)

Very high ast and alt( 1000) From Journal of Clinical Gastroenterology;
Mild increase in LDH

Us : unremarkable

What is dx:

1) ischemic hepatitis

2. Intravascular hemolysis

3. ICU related jaundice

4. aclculus cholycystitis

......

10- what is the Symptom ssuggest severity of dementia


1. Increase irritability

2. Difficulty finding word

3. Forget appointment

4. Difficulty in reading

....

41-

A patient who was diagnosed with pneumonia was treated with antibiotics. On day 3, they
developed severe bilateral crackles with respiratory failure which needed intubation. What is the
most likely diagnosis? UPtoDate: ARDS should be suspected in patients with; progressive symptoms of ;
dyspnea, an increasing requirement for oxygen, and alveolar infiltrates on
chest imaging within 6 to 72 hours of an inciting event (table 1).

a. Pleural effusion On examination patients may have tachypnea, tachycardia, and diffuse crackles.
When severe, acute confusion, respiratory distress, cyanosis, and diaphoresis may
be evident.

b. ARDS

patient presents with a temperature of 39 degrees celsius. X-Ray shows air under the
diaphragm. What is the next step?

a. Exploratory Laparotomy Fever = sign of infection


b. Diagnostic Laparoscopy

c. CT Scan

d. Ultrasound

child presents with hip and groin pain. An US was done to reveal hip joint effusion. He is
afebrile. What is the most likely diagnosis?
Toxic synovitis = Transient synovitis
a. Toxic synovitis

b. Benign acute myositis

c. Osteomyelitis

d. Septic arthritis

31. A 25 year old healthy female presents with a systolic murmur best heard at the apex
radiating to the axilla. She has no past medical history. She has a recent history of dental tooth
extraction 2 months ago and sore throat 10 days ago. What is the most likely diagnosis
case of Rheumatic herat disease presneting with Mitral valve
regurgitation. source; mayoclinic.org

a. Infective endocarditis

b. Group A streptococcus

Ans: B

A baby was admitted for circumcision. During the surgery, the baby could not stop bleeding.
What is your action?

a. Continue the surgery ABC of surgery.


source; UTD
b. Apply direct pressure

c. Investigate the cause

Ans: B

Trauma Pt with extra peritoneal bladder injury ? Suprapubic catheter

Urgent exploration and repair

Catheter repair and assess after 2 weeks

Catheter us after 2 weeks

woman had a C-Section. What is the best way to prevent adhesions?

a. Perform the C-section pre-term this is the most likely answer


source; https://journals.sagepub.com/doi/full/10.2217/WHE.13.45
b. Closure of the peritoneum

c. Add an adhesion barrier before closing the wound

70 year old woman presents with a protruding mass and very little muscle on examination. What
is your management?

a. Surgery

b. Passerie Ans: B

woma
next step?
UTD
a. Colposcopy with biopsy

b. Cone biopsy

c. Repeat pap smear


185. A patient who was diagnosed with pneumonia was treated with antibiotics. On day 3, they
developed severe bilateral crackles with respiratory failure which needed intubation. What is the most
likely diagnosis?

a. Pleural effusion

b. ARDS
Repeated

lady who is smoking 20 cigarettes a day comes with IUFD. What is the factor that led to fetal demise?

a. Chromosomal anomaly
UTD
b. 20 cigarettes a day

c. There is no identifiable cause

Pt has amenorrhea pupic hair and axilary hair and high testosterone:
Repeated
A. Mullerian agenesis ( another name Mayer Rokitansky Kuster Hauser syndrome )

B. Androgen insensitivity

Type of heart rate control in atrial fibrillation (4 drugs)) beta blocker correct, source; UTD:
Smoking most common risk factor for:-

COPD

LUNG CANCER
UTD
hypertension

Patient complaining of burning chest pain and unpleasant taste when lifting heavy objects:-

Gastritis

Esophagitis

Perforated peptic ulcer

Mumps in pedia affect which organ:-

Lung

Skin correct, source; UTD:


Parotid

Screening in pregnant women detecting HBV consider:-

Primary

Repeated
Secondary

Tertiary

: Female patient complaining of pain before menses and resolved in the third day of menses, how do
you diagnose it?

Hysteroscopy
Repeated
Abd US

clinical symptoms

Most common cause of postpartum bleeding:-

Uterine atony Uterine atony (ie, a lack of effective uterine contraction after birth) prevents mechanical hemostasis from occurring
and is responsible for at least 80 percent of cases of PPH
Retained tissue Source: UTD

Laceration

-
Crohn's Dermatologic manifestations occur in approximately 10 percent of patients and include;
Erythema nodusum erythema nodosum and pyoderma gangrenosum

Source: UTD
When to do urine test for pregnant women:-

12

15 Repeated
20

24

Tumour lysis syndrome:-

Hypocalcemia hypokalemia

Hypocalcemia hyperkalemia

....

Patient post hernia repair, when can she get pregnant:-

6
Repeated
9

12

Pt , k/c of sle , she is on prednisolone, hydroxuchlorcqune , MMF , she want to pregnant, what to do?

Stop MMF start MTX

source: UTD
Stop MMF start azathoprine

sle

Chlild has 2 rash in lower limb and one in eyelids, what to do?

Refer to Optha

Refer surgical

Abx

Conservative pediatrics consultant

Pt came with RUQ pain and jundice , what is the next investigation?

HbA IgG

HbA IgM correct, source; AMBOSS


HbB

Goodpasture correct, source; AMBOSS


28 yo patient came with hip fracture, orthopedic surgeon wants to give her prophylactic for DVT?

A. Enoxaparin

B. Heparin correct, source; AMBOSS


C. LMWH

Shaima SMLE: Patient came wi

A. Parkinson

B. Alzheimer

C. Dementia with lowey body

Female Patient came with typical presentation of PCOS asking how dx Source: Mayoclinic.org

Rotterdam criteria (preferred) Most expert groups use Rotterdam criteria to make the
diagnosis of PCOS [19,23].

Two out of three of the following criteria are required to make the diagnosis [24]:

Oligo- and/or anovulation


Clinical and/or biochemical signs of hyperandrogenism
Polycystic ovaries (by ultrasound)
New born Baby came with right inguinal hernia + rifht undescended testicles and no left inguinal hernia,

A. Observation

B. Wait till 6 years and do surgery

define which procedure?

A. Barium enema

B. CT abdomen with contrast

C. UGIE

4 years old pediatric with recurrent uti infection the doctor wants to do investigation to see
complications like renal scaring

Whats the investigation?

US

MCUS

DMSA

technetium 99 something

: - rta with wound in the thigh then bacame infected with greenish or yellowish discharge with gas whats
the causative organism;

Staph aureus

Clostridium perfringens

1-Cocain withdrawal

2-Amphetamine toxicity

I forgot the other 2

: Patient with sjogren syndrome which type of RTA:

+ Labs:

Shows low K

Normal Na

A) RTA |

B) RTA II
C) RTA III

D) RTA IV

: which type of RTA comes with Hyperkalemia?

A) RTA type 1

B) RTA type 2

C) RTA type 3

D) RTA type 4

Full term baby will be discharge from hospital what will you give him?

A) BCG, HBV

B) HBV, MMR

: old prim gravida woman just delivered spontaneously baby. is

delivered complete and intact. Massaging of the uterine is performed along with 20 units of oxytocin in
-cm
left lateral vaginal wall, suturing is difficult because of bleeding from above the site of laceration. a soft,
boggy uterine fundus Blood pressure 164/92 mmHg Heart rate 130 /min

Which of the following is the best step in management ?

Amboss

A .prostaglandin f 2

B. methylergonovine

C. manual exploration

D. oxytocin 10 units again

If there was no response to the maximum dose oxytocin the next step is prostaglandin hacker and
obgyne consultant

: 1- 28 y/o female k/c of SLE presenting with neuropsychiatric symptoms. What you will give for her ?

A- Corticosteroid+Cyclophosphamide

B- cyclophosphamide

C- IV corticosteroid

D- Corticosteroid+Rituximab

You might also like